Family Medicine Board Review

अब Quizwiz के साथ अपने होमवर्क और परीक्षाओं को एस करें!

A 45-year-old female returns to your clinic after pulmonary function testing for dyspnea. Her prebronchodilator FEV1/FVC ratio was 0.65, which improved to 0.9 post bronchodilator. These findings are most consistent with which one of the following? A) Asthma B) COPD C) Interstitial lung disease D) Pulmonary hypertension

ANSWER: A A diagnosis of COPD is established by an FEV1/FVC that is consistent with obstruction and is not significantly reversible with bronchodilator treatment. The American Thoracic Society/European Respiratory Society guidelines define reversibility as an improvement of more than 12% in adults. This patient's FEV1/FVC increased by more than that with bronchodilation, so her results are most consistent with asthma. Spirometry is not used to diagnose interstitial lung disease or pulmonary hypertension. Interstitial lung disease is diagnosed using high-resolution CT. Echocardiography is the recommended first step in the evaluation of suspected pulmonary hypertension, but confirmation by right heart catheterization is often required.

Which one of the following U-100 insulin products has the longest duration of action? A) Degludec (Tresiba) B) Glargine (Lantus) C) Isophane NPH (Humulin N) D) Lispro (Humalog) E) Regular (Humulin R)

ANSWER: A Among the available U-100 insulin products, the one with the longest duration of action is ultralong-acting degludec, which lasts 42 hours. The duration of action of rapid-acting lispro is 3-6.5 hours, short-acting regular is 5-8 hours, intermediate-acting isophane is 12-16 hours, and long-acting glargine is 11-24 hours.

A 21-year-old female sees you for a medical evaluation prior to admission to a treatment program for anorexia nervosa. The effects of anorexia on the hypothalamic-pituitary axis can cause which one of the following? A) Bone loss B) Elevated testosterone C) Hyperglycemia D) Hypothyroidism E) Menorrhagia

ANSWER: A Anorexia has multiple effects on the hypothalamic-pituitary axis. Bone loss can be significant. In a study of 130 women, bone mineral density was reduced by at least 1.0 standard deviation at one or more skeletal sites in 92% of patients. Testosterone levels are often low, contributing to bone loss. Hypoglycemia, not hyperglycemia, can occur but this is not common. Anorexia often results in amenorrhea and infertility, and TSH and T4 levels may be normal or low.

A 62-year-old female sees you for a routine health maintenance examination. She has a history of breast cancer diagnosed 6 years ago that was treated with lumpectomy, radiation, and endocrine therapy. She is feeling well today and has no symptoms of concern. There is no family history of breast, ovarian, colon, or prostate cancers. In addition to mammography, which one of the following annual tests would improve this patient's chance of survival? A) No tests B) CT of the chest C) MRI of the breast D) Breast ultrasonography E) A bone scan

ANSWER: A Breast cancer, the most common noncutaneous malignancy among women, has a 5-year survival rate of almost 90%, so medical care of such patients is increasingly common. To help provide guidance to primary care physicians, the American Cancer Society and the American Society of Clinical Oncology published their joint Breast Cancer Survivorship Care Guideline in 2016. This guideline includes a recommendation for annual mammography for women with prior treatment for breast cancer to screen for local recurrence or a new primary breast cancer. MRI is not recommended in the absence of specific high-risk criteria such as a BRCA mutation. Similarly, other imaging modalities such as ultrasonography are not recommended in the absence of symptoms. Imaging is not indicated to screen for metastatic disease. Though breast cancer most commonly metastasizes to the lung, bone, and liver, there is no evidence that screening CT or a bone scan provides mortality or quality-of-life benefits.

A 52-year-old gravida 4 para 4 presents with bothersome incontinence, predominantly with coughing and straining. In addition to a physical examination, urinalysis, and measurement of postvoid residual volume, which one of the following tests is recommended as part of the initial evaluation? A) Cough stress testing B) Urodynamic testing C) Pelvic ultrasonography with a vaginal probe D) Cystoscopy

ANSWER: A Cough stress testing helps detect urine leakage with coughing in patients with at least 200-300 cc of urine in the bladder or with the sensation of a full bladder. This test is recommended as part of the initial evaluation of women with symptoms of stress urinary incontinence. It has excellent sensitivity when compared to urodynamic testing. The initial evaluation could also include the cotton swab test (insertion of a lubricated swab into the urethra and evaluating angle change with the Valsalva maneuver). Urodynamic testing, pelvic ultrasonography, and cystoscopy are not recommended.

The U.S. Preventive Services Task Force recommends screening for depression for A) all adults B) all women but not men C) only adults with a family history of depression D) only adults with a known personal history of depression E) only adults with a history of disability, medical illness, complicated grief, chronic sleep disturbance, and/or loneliness

ANSWER: A Currently the U.S. Preventive Services Task Force recommends that all adults should be screened for depression, as it is one of the leading causes of disability in persons older than 15 years of age (B recommendation). The optimal interval is yet to be defined. Depression is more common in women, but screening is recommended for all adults. While a family history of depression, a personal history of depression, disability, medical illness, grief, sleep disturbance, and loneliness are all risk factors for depression, these factors do not need to be present to screen.

The most common symptom of alcohol withdrawal in the elderly is A) confusion B) seizures C) tachycardia D) tremor E) vomiting

ANSWER: A In older adults the onset of alcohol withdrawal syndrome may not occur until several days after the cessation of drinking. Confusion, rather than tachycardia or tremor, is often the predominant clinical sign, and the severity and duration of withdrawal tend to increase with age. Alcohol withdrawal should be considered as a cause of confusion in older patients and may be manifested as new-onset confusion in a hospitalized older patient. Confusion is also a more common symptom of alcohol withdrawal in the elderly than seizures or vomiting.

A 70-year-old male sees you for evaluation prior to cataract surgery. He has well controlled hypertension, as well as prediabetes and erectile dysfunction. Which one of the following would be most appropriate prior to the procedure? A) No testing B) A prothrombin time and INR determination C) A CBC D) A resting EKG E) 2D echocardiography

ANSWER: A Medical testing prior to cataract surgery does not improve outcomes and is not recommended. A prothrombin time and INR determination, a CBC, a resting EKG, and 2D echocardiography would not be appropriate prior to this patient's cataract surgery.

A 49-year-old female presents to your office in northern California with a 6-week history of increasing cough, wheezing, fever, and increased shortness of breath. She has type 2 diabetes and persistent asthma uncontrolled on high-dose budesonide/formoterol (Symbicort) and montelukast (Singulair). Two weeks ago she was treated with azithromycin (Zithromax) and a 10-day taper of prednisone. Her symptoms initially improved but have worsened in the past 3 days. She has not traveled recently. On examination today she has a temperature of 37.7°C (99.9°F), a respiratory rate of 22/min, and an oxygen saturation of 92% on room air. A pulmonary examination is significant for diffuse expiratory wheezing. A chest radiograph shows some interstitial thickening and is read as concerning for bronchiectasis. A CBC is significant for eosinophilia with an eosinophil count of 960/mm3 (N 30-350). HIV testing is negative. Testing for which one of the following organisms is most likely to reveal a contributing factor to her illness? A) Aspergillus B) Coccidioides immitis C) Histoplasma D) Mycobacterium tuberculosis E) Pneumocystis jiroveci

ANSWER: A Pulmonary aspergillosis comprises a spectrum of clinical disease, from invasive, often cavity-forming disease in critically ill and profoundly immunosuppressed patients to allergic bronchopulmonary aspergillosis. This case of poorly controlled asthma associated with eosinophilia and bronchiectasis is typical of allergic bronchopulmonary aspergillosis. Aspergillus IgE titers are recommended as initial testing in patients with suspected allergic bronchopulmonary aspergillosis. Antifungal treatment can improve outcomes in these cases. Coccidioides immitis and Histoplasma typically cause more systemic symptoms such as muscle and joint pain, rather than wheezing. Coccidioides immitis is present in the desert regions of the U.S. southwest and Histoplasma is endemic to the Mississippi and Ohio River valleys of the Midwest and the South. Mycobacterium tuberculosis is less common in the United States, and this patient's symptoms and findings are not typical of active tuberculosis. Pneumocystis jiroveci causes pneumonia but is rare in patients who are not overtly immunosuppressed.

A 25-year-old female at 28 weeks gestation comes to your office for a routine prenatal visit. She received Tdap vaccine 3 years ago after she was bitten by a horse. Which one of the following would be most effective to reduce the newborn's risk of contracting pertussis? A) Tdap vaccination of the mother now B) Tdap vaccination of the mother post partum C) No Tdap vaccination of the mother now or post partum, and Tdap vaccination of other family members D) No Tdap vaccination of the mother now or post partum, and Tdap vaccination of the newborn within 72 hours of birth E) No Tdap vaccination of the mother now or post partum, and a recommendation for good hand hygiene to protect the newborn

ANSWER: A The Advisory Committee on Immunization Practices recommends Tdap vaccination during each pregnancy, regardless of the time interval since the last booster, primarily to confer immunity against pertussis to the infant. For that reason administration of the vaccine is recommended between 27 and 36 weeks gestation to maximize the concentration of pertussis antibody transferred to the fetus. Postpartum vaccination of the mother with Tdap does not provide as much protection as vaccination of the mother during pregnancy. Newborns should not receive Tdap but should begin pertussis immunization at 2 months of age with the first dose of DTaP. There is no recommendation to vaccinate infants earlier to reduce the risk of vertical transmission. Family members and close contacts should be vaccinated against pertussis at least 2 weeks prior to contact with the infant, but this strategy alone is less effective than maternal immunization. Good hand hygiene is important when caring for an infant, but on its own is not an effective strategy to reduce the risk of pertussis transmission.

An 87-year-old female comes to your office for an annual health maintenance visit. She appears cachectic and tells you that for the past 6 months she has had a decreased appetite and generalized muscle weakness. The patient is alert and oriented to person and place. She has a 10% weight loss, dry mucous membranes, and tenting of the skin on the extensor surface of her hands. While inflating the blood pressure cuff on her right arm you observe carpopedal spasms. Which one of the following is the most likely electrolyte disturbance? A) Hypercalcemia B) Hypocalcemia C) Hypokalemia D) Hypernatremia E) Hyponatremia

ANSWER: B A Trousseau sign, defined as spasmodic contraction of muscles caused by pressure on the nerves that control them, is present in up to 94% of patients with hypocalcemia. Hypercalcemia is more likely to present with hyperreflexia. Patients with hypokalemia, hypernatremia, or hyponatremia may present with weakness and confusion, but tetany is not a common sign of either sodium or potassium imbalance.

Long-term proton pump inhibitor use is associated with an increased risk for A) Barrett's esophagus B) gout C) hypertension D) pneumonia E) type 2 diabetes

ANSWER: D Acid suppression therapy is associated with an increased risk of community-acquired and health care-associated pneumonia, which is related to gastric overgrowth by gram-negative bacteria. Long-term treatment of Barrett's esophagus is an indication for chronic proton pump inhibitor (PPI) use. PPI therapy does not increase the risk of gout, hypertension, or type 2 diabetes.

Routine follow-up blood tests for colorectal cancer survivors should include A) carcinoembryonic antigen (CEA) levels only B) liver function tests only C) CBCs and CEA levels only D) CBCs and liver function tests only E) CBCs, CEA levels, and liver function tests

ANSWER: A The Choosing Wisely campaign recommends checking only carcinoembryonic antigen (CEA) levels following curative treatment for colorectal cancer (SOR C). No routine laboratory studies such as a CBC or liver function tests should be ordered for follow-up.

Which one of the following patient factors would be most likely to contraindicate use of a thiazide diuretic such as chlorthalidone? A) Chronic gout B) An ejection fraction of 45% C) Hypothyroidism D) Stage 3b chronic kidney disease E) A history of a rash when taking trimethoprim/sulfamethoxazole (Bactrim) in childhood

ANSWER: A According to American College of Cardiology/American Heart Association guidelines, first-line agents for antihypertensive drug therapy include thiazide diuretics, calcium channel blockers, ACE inhibitors, or angiotensin receptor blockers. In the largest head-to-head comparison of first-step drug therapy for hypertension, the thiazide-type diuretic chlorthalidone was superior to the calcium channel blocker amlodipine and the ACE inhibitor lisinopril in preventing heart failure. Thiazides increase serum uric acid and the risk of gouty attacks, so they should be used with caution in patients with a history of gout. The presence of a reduced ejection fraction or chronic kidney disease does not preclude the use of a thiazide diuretic, although progressive kidney disease or concomitant use of loop diuretics does increase the risk of electrolyte abnormalities. Furthermore, thiazide diuretics are ineffective in patients with severe renal disease (stage 4 or stage 5). Hypothyroidism is not a contraindication to the use of thiazide diuretics. The potential for cross reactivity between antibiotic sulfonamides and non-antibiotic sulfonamides is extremely low and may be nonexistent.

A 46-year-old male presents to your office and asks you to screen him for pancreatic cancer. He tells you that his best friend was recently diagnosed with pancreatic cancer at age 45. His friend was previously healthy and had no family history of cancer. The patient is doing well and has no symptoms. He does not have a family history of cancer. Based on U.S. Preventive Services Task Force recommendations, which one of the following would be most appropriate at this time? A) No screening B) Ultrasonography of the abdomen C) CT of the abdomen without contrast D) CT of the abdomen with contrast E) MRI of the abdomen

ANSWER: A Although an uncommon cancer, pancreatic cancer is the third most common cause of cancer death in the United States. The incidence is rising, and it is estimated that it may soon become the second-leading cause of cancer deaths. In an asymptomatic patient with no family history of pancreatic cancer or inherited genetic syndromes, the U.S. Preventive Services Task Force recommends against screening for pancreatic cancer (D recommendation).

Which one of the following clinical features is a component of the STOP-Bang questionnaire used as a screening tool for obstructive sleep apnea? A) Blood pressure B) Pulse rate C) Resting oxygen saturation D) Smoking status E) Waist circumference

ANSWER: A The STOP-Bang questionnaire is a screening tool to help identify patients with obstructive sleep apnea. In the questionnaire S = snoring, T = tiredness, O = observed apnea, P = high blood pressure, B = BMI >35 kg/m2, A = age >50 years, N = neck circumference >40 cm, and G = male gender. For each question, answering "yes" scores 1, answering "no" scores 0, and the total score can range from 0 to 8, with a higher score indicating a higher probability of obstructive sleep apnea. Pulse rate, resting oxygen saturation, smoking status, and waist circumference are not components of the STOP-Bang questionnaire.

You have assumed the care of a well established patient in your practice whose medications include chronic alprazolam (Xanax) treatment for anxiety and codeine for chronic back pain following a work accident years earlier. His Prescription Drug Monitoring Program report shows a consistent pattern of filling the medications as prescribed. You order a urine immunoassay for opioids and benzodiazepines. The results are positive for opioids but negative for benzodiazepines. Which one of the following would be the most appropriate next step? A) Perform confirmatory testing for alprazolam B) Repeat the urine immunoassay for benzodiazepines C) Investigate for possible diversion of alprazolam D) Stop prescribing alprazolam E) Stop prescribing controlled substances

ANSWER: A Because of the importance of urine drug testing and the ramifications for patients, it is essential that physicians understand and properly interpret these results. The most appropriate next step in this case is to perform confirmatory testing for alprazolam. Immunoassays can have false-positive and false-negative results, and unexpected negative results must have confirmatory testing for verification (SOR C). The immunoassay for benzodiazepines detects the metabolite nordiazepam only, which is a metabolite of diazepam, oxazepam, and temazepam but not of alprazolam, lorazepam, or clonazepam. This negative immunoassay screening test would require confirmatory testing for alprazolam. A repeat immunoassay for benzodiazepines would likely show the same negative result and would not change decision-making. The immunoassay for opioids detects only nonsynthetic opioids such as morphine and codeine, and a positive immunoassay for codeine would be expected in this case. It would be inappropriate to suspect drug diversion, or to stop prescribing alprazolam or controlled substances, based upon the negative immunoassay for benzodiazepines in this patient.

A 6-month-old female is brought to the emergency department by her parents with a 4-day history of fever, congestion, cough, shortness of breath, and decreased appetite. On examination of the lungs you note wheezes and crackles throughout. You also note subcostal retractions, an oxygen saturation of 91% on room air, and a respiratory rate of 42/min. A chest radiograph reveals peribronchial markings with no infiltrates. The parents say that the patient's teenage cousin visited recently and was having "cold symptoms." You suspect bronchiolitis. Which one of the following would you recommend next? A) Maintaining hydration and keeping oxygen saturation >90% B) Deep nasal suctioning C) Albuterol via nebulizer D) Broad-spectrum antibiotics E) Systemic corticosteroids

ANSWER: A Bronchiolitis is a common lower respiratory tract infection in young children and infants. Respiratory syncytial virus (RSV) is the most common cause. Supportive care with hydration and maintenance of oxygen saturation is important in the treatment of RSV bronchiolitis. Infants with respiratory rates >60/min are often unable to manage oral hydration due to the risk of aspiration. In these cases, intravenous or nasogastric feeds are acceptable. An oxygen saturation >90% is sufficient in RSV bronchiolitis and use of supplemental oxygen to achieve higher levels of oxygen saturation may prolong hospital stays. There is no clear advantage to deep nasal suctioning, which may also be associated with prolonged hospital stays. Routine nasal suctioning is indicated, however. Bronchodilators are not recommended in the treatment of RSV (level of evidence A). Antibiotics are only indicated with a confirmed bacterial co-infection (level of evidence B). Systemic corticosteroids have shown no benefit in the treatment of bronchiolitis.

A 45-year-old female sees you for the first time for a health maintenance visit. She has a history of chronic low and midback pain for several years, which she treats with ibuprofen, muscle relaxers, and a heating pad. She takes a combined oral contraceptive pill to control heavy menstrual cycles. She recently went to an urgent care clinic for treatment of an upper respiratory tract infection. She also mentions the gradual appearance of a rash on her back but says that it does not bother her. A skin examination is notable for reticular brown hyperpigmentation of her entire back. Which one of the following is the most likely cause of the skin changes? A) Erythema ab igne B) Henoch-Schönlein purpura C) Idiopathic guttate hypomelanosis D) Tinea versicolor

ANSWER: A Erythema ab igne is characterized by reticular brown hyperpigmented skin changes at the site of exposure to heat, which this patient has developed due to regular use of a heating pad. Exposure to excess heat and humidity can predispose an individual to this condition. Henoch-Schönlein purpura is a leukocytoclastic vasculitis and is characterized by palpable purpuric lesions on the lower extremities and buttocks, usually sparing the trunk. It may be preceded by a streptococcal or viral upper respiratory infection. Idiopathic guttate hypomelanosis consists of 2- to 5-mm white spots with sharply demarcated borders on sun-exposed areas of the arms and legs. Tinea versicolor is a fungal infection that is characterized by white, pink, or brown circular macules most commonly located on the upper trunk.

A 40-year-old runner presents with pain in the left leg. He is training for a marathon and has been increasing his running distance in recent weeks. He reports localized pain and swelling at the midpoint of the shin over the past 4 weeks that begins after a run and lasts for a few days, but now the swelling and tenderness have lasted for several days and there is severe pain when he tries to run. You suspect a tibial stress fracture. Which one of the following imaging modalities would be indicated initially? A) A plain film B) Ultrasonography C) CT D) MRI E) Bone scintigraphy

ANSWER: A For a suspected tibial stress fracture, plain radiography is indicated as the initial imaging modality due to its availability and low cost. Its sensitivity is highest when symptoms have been present for at least 3 weeks, as in this case. Ultrasonography and CT are not indicated for this patient. If plain radiography is normal and further imaging is warranted, MRI or bone scintigraphy should be considered. Both modalities have a similar sensitivity, but MRI is preferred due to the greater specificity and ability to inform alternate diagnoses.

A 68-year-old female with a history of diabetes mellitus, hypertension, and heart failure with preserved ejection fraction presents with a long-standing diabetic foot ulcer. The patient reports no signs of any gastrointestinal bleeding, no blood in her urine, no bleeding gums, and no vaginal bleeding. Her medications include metformin (Glucophage), insulin glargine (Lantus), lisinopril (Prinivil, Zestril), atorvastatin (Lipitor), and furosemide (Lasix). A CBC reveals a WBC count of 7600/mm3 (N 4500-11,000), a hemoglobin level of 9.7 g/dL (N 14.0-17.5), a mean corpuscular volume of 89 m3 (N 80-100), and a platelet count of 412,000/mm3 (N 150,000-400,000). To further assess the patient's anemia you obtain the following laboratory results: Ferritin 293 ng/mL (N 22-275) Serum iron 43 g/dL (N 50-175) Transferrin 190 mg/dL (N 177-264) Reticulocyte count 3.2% (N 0.5-1.5) Vitamin B12 564 pg/mL (N 230-1050) Haptoglobin 198 mg/dL (N 63-273) Which one of the following is the most likely cause of her anemia? A) Anemia of chronic disease B) Bone marrow suppression C) Hemolysis D) Iron deficiency E) Vitamin B12 deficiency

ANSWER: A This patient has findings most consistent with anemia of chronic disease, also known as anemia of inflammation. This condition is thought to be primarily a disorder of iron distribution in response to systemic inflammation, which also biases hematopoiesis toward myeloid cell production rather than erythropoiesis and shortens the erythrocyte lifespan. Anemia of chronic disease is a normocytic and normochromic anemia. Iron studies typically show evidence of iron restriction without systemic iron deficiency. A common challenge in diagnosis is when true iron deficiency coexists with anemia of chronic disease. This patient's normal WBC and platelet counts make bone marrow suppression less likely. The normal haptoglobin level and low reticulocyte count are not consistent with hemolysis. She has a normocytic rather than microcytic anemia and her ferritin level is elevated. These two factors make iron deficiency less likely despite her low serum iron level. The low normal transferrin level is also consistent with anemia of chronic disease rather than iron deficiency. Her normal vitamin B12 level makes a deficiency unlikely. Her history of a chronic foot ulcer and elevated inflammatory markers (ferritin and platelets) are consistent with anemia of chronic disease.

You are caring for an 84-year-old female with Alzheimer's dementia. She is disoriented to time but usually oriented to person and place. She is homebound but is frequently visited by family. The patient's daughter feels conflicted but wants her mother to go to a skilled care facility. The patient is adamant that she does not want to leave her home. During a recent home visit you noted that she smelled of urine and had a severe intertriginous rash suspicious for Candida. Her bedside Mini-Mental State Examination score was 24/30. Which one of the following ethical principles should be the primary consideration when determining whether this patient should continue to live at home? A) Autonomy B) Competence C) Harm D) Justice E) Safety

ANSWER: A Four widely recognized principles of medical ethics include respect for autonomy, beneficence, nonmaleficence, and justice. Respect for patient autonomy is central to medical ethics and to the doctor-patient relationship. Physicians should involve patients in health care decisions commensurate with the patient's capacity to understand and make decisions. Even when a condition affects a patient's decision-making capacity, the patient may be able to participate in some aspects of the decision-making process. Competence (or incompetence) is a legal term, which can be used to refer to a patient being granted autonomy or not. Harm and safety are not ethical principles, but are related to the principle of nonmaleficence, or "do no harm." The principle of justice addresses the question of who ought to receive the benefits of an intervention and/or bear its burdens. An injustice occurs when a person who is entitled to a benefit is denied without good reason or when some burden is imposed unduly.

You diagnose polymyalgia rheumatica in a 63-year-old female and begin treatment with oral prednisone, 20 mg daily. You anticipate several months of treatment with prednisone and plan to taper the dose as tolerated but are concerned about her bone health. Which one of the following would be most appropriate for helping to prevent osteoporotic fractures in this patient? A) A FRAX score based on DXA results now B) A FRAX score based on DXA results in 6 months C) A calcium intake of 1200 mg daily D) Alendronate, 35 mg weekly

ANSWER: A Glucocorticoid use is the most common cause of secondary osteoporosis. Glucocorticoids increase bone resorption early, so addressing the issue at the start of treatment is vital in preventing fractures. The use of glucocorticoids is associated with an increased risk of fracture in the first 6 months. A bone density test should be performed shortly after starting corticosteroid treatment (SOR C). The fracture risk can then be calculated using the FRAX assessment tool. If the risk of major osteoporotic fracture is sufficiently elevated, then treatment is recommended. One caveat is that the FRAX score should be adjusted upward if the prednisone dosage is >7.5 mg daily. Calcium alone has not been shown to reduce the risk of fracture in osteoporosis. Calcium and vitamin D together have been shown to prevent decreases in bone mineral density with low-dose prednisone use but the effect with high doses is unknown. If a patient has an increased fracture risk, oral bisphosphonates should be started. It is not recommended that they be used empirically to reduce the risk of fracture.

A 38-year-old male presents with a 2-month history of increased postprandial nausea and nonbloody, painless, loose stools. He feels well otherwise. Ten months ago he underwent bariatric surgery, which involved creating a small stomach pouch. There were no complications from the surgery and he does not take any medications. A review of his diet reveals that he has three small meals and two snacks daily. He has one serving of vegetables or bread with each meal. He has 16 oz of coffee with breakfast, 12 oz of soda with lunch, 12 oz of beer with dinner, and a cup of water or milk with snacks. His snack is usually cheese or peanut butter and a cracker. An examination is unremarkable. You are concerned with his diet habits and your recommendations include A) no fluid for 15 minutes before or after meals and snacks B) limiting carbonated beverages to 8 oz per meal or snack C) increasing daily servings of fibrous vegetables and whole grain breads D) eating one meal and three snacks daily E) eating a diet lower in fat

ANSWER: A Hundreds of thousands of Americans have undergone bariatric surgery, and family physicians are often asked to provide long-term postoperative management. Many bariatric surgery procedures create a small stomach pouch. Dietary compliance is essential to minimize feeding intolerance symptoms such as postprandial nausea, emesis, and diarrhea. Post bariatric surgery diet recommendations typically include the following: • Avoid fluid 15 minutes before and after meals. Fluids with meals will move food more quickly through the pouch and decrease the feeling of fullness. • Avoid carbonated beverages entirely. • Eat three small protein-rich meals and one or two snacks daily. Lower fat diets are not typically recommended. • Whole grains and fibrous vegetables often exacerbate symptoms so there is no need to increase these foods.

At a routine visit, a 65-year-old male former smoker reports shortness of breath and a cough that has been worsening slowly over the last 6 months. On examination you hear bibasilar inspiratory crackles. An EKG, chest radiograph, and echocardiogram are normal. CT of the chest shows multiple bilateral patchy areas of consolidation. The most appropriate next step is to A) take a detailed history of medication use and lifetime environmental exposures B) start an antibiotic C) start furosemide (Lasix) D) start an inhaled short-acting -agonist as needed E) refer for pulmonary rehabilitation

ANSWER: A Idiopathic pulmonary fibrosis occurs most often in male former smokers over the age of 60. For patients with newly diagnosed interstitial lung disease (ILD) with suspected idiopathic pulmonary fibrosis, the American Thoracic Society recommends taking a detailed history of medication use and environmental exposures over the patient's lifetime. In an observational study of 1084 patients, 47% were identified as having hypersensitivity pneumonitis on a detailed assessment of new-onset ILD with an unknown cause. Laboratory testing for connective tissue disease is also recommended. Antibiotics would be appropriate to treat a bacterial infection. Furosemide is used to treat heart failure. An inhaled short-acting -agonist and pulmonary rehabilitation would not be appropriate at this time.

A 68-year-old male with a history of diabetes mellitus, hypertension, coronary artery disease, and heart failure with reduced ejection fraction sees you for a follow-up visit 2 weeks after cardiac catheterization and placement of two drug-eluting stents. He tells you that shortness of breath slightly limits his physical activity. Before the procedure an echocardiogram revealed apical hypokinesis and an estimated left ventricular ejection fraction of 25%. His current medications include carvedilol (Coreg), atorvastatin (Lipitor), aspirin, clopidogrel (Plavix), spironolactone (Aldactone), sacubitril/valsartan (Entresto), metformin (Glucophage), and empagliflozin (Jardiance). On examination the patient is euvolemic, his pulse rate is 58 beats/min, his blood pressure is 112/60 mm Hg, and his hemoglobin A1c is 7.2%. Which one of the following would be the most appropriate management of his heart disease? A) No medication changes and repeat echocardiography in 2 months B) Replacing empagliflozin with sitagliptin (Januvia) C) Replacing sacubitril/valsartan with lisinopril (Prinivil, Zestril) D) Immediate placement of an implantable cardiac defibrillator E) Referral for left atrial appendage closure

ANSWER: A This patient presents with ischemic cardiomyopathy associated with heart failure with reduced ejection fraction and New York Heart Association (NYHA) class II symptoms. He is taking appropriate medical therapy, including dual antiplatelet agents, a high-intensity statin, a -blocker, an aldosterone antagonist, and an angiotensin receptor-neprilysin inhibitor. Sacubitril/valsartan is superior to an ACE inhibitor in patients such as this (SOR A). SGLT2 inhibitors such as empagliflozin, unlike DPP-4 inhibitors such as sitagliptin, have been associated with improved symptoms and lower rates of cardiovascular death in patients with heart failure (SOR A). An implantable cardiac defibrillator (ICD) is indicated to decrease sudden cardiac death in a patient with heart failure treated with guideline-directed medical therapy who has a left ventricular ejection fraction (LVEF) 30% with NYHA class I symptoms or an LVEF 35% with NYHA class II-III symptoms (SOR A). However, the patient must be at least 40 days out from a myocardial infarction and at least 90 days out from revascularization. The patient in this case should have repeat echocardiography to reassess his LVEF before he is referred for ICD placement.

A 24-year-old male with sickle cell disease (Hb SS) presents to the emergency department (ED) with a 5-day history of severe pain in his lower back, legs, and shoulders. The review of systems is positive for jaundice and scleral icterus, but is negative for headache, chest pain, shortness of breath, and priapism. He reports that his symptoms are typical of his previous vaso-occlusive pain, and he believes this episode was triggered by the recent cold weather. He tried to avoid coming to the ED by using oral medication at home, including naproxen, 500 mg twice daily, and immediate-release oxycodone (Roxicodone), 10 mg every 4 hours as needed. He takes no other medications and reports no known drug allergies. He is pain free on approximately 75% of days, and he can typically manage his pain at home during the remaining times. The patient's vital signs are unremarkable except for a mildly elevated pulse rate of 102 beats/min. His WBC count is 17,000/mm3 (N 5000-10,000) and his hemoglobin level is at its baseline of 8.0 g/dL. The patient is admitted to the hospital for intravenous fluids and pain medications via patient-controlled analgesia. This is his second admission of the year for a vaso-occlusive pain crisis. He responds well over the next few days and is discharged to home in good condition. Which one of the following should be discussed with the patient at follow-up? A) Daily hydroxyurea therapy B) Oral penicillin prophylaxis C) Switching from immediate-release oxycodone to long-acting oxycodone (OxyContin) twice daily D) Annual transcranial Doppler ultrasonography E) Scheduled blood transfusions to maintain a hemoglobin level >10 g/dL

ANSWER: A In a clinical trial of patients with sickle cell disease, those taking hydroxyurea had two fewer severe vaso-occlusive pain crises per year, and they also had a decreased need for blood transfusions. Hydroxyurea is also the only medication that has been shown to prevent acute chest syndrome (ACS) in sickle cell disease, with a number needed to treat of 6 to prevent an episode of ACS over a 21-month period. Hydroxyurea therapy has traditionally been recommended for patients who have three or more severe vaso-occlusive pain crises per year, or for those who have daily pain that affects their quality of life. However, given the risk-to-benefit profile of the medication, current guidelines now recommend offering hydroxyurea to reduce complication rates for all patients older than 9 months of age with sickle cell anemia (SOR B). Oral penicillin prophylaxis to prevent pneumococcal sepsis is indicated for young children (typically up until age 5), not young adults. Similarly, transcranial Doppler ultrasonography to screen for stroke is performed in children and adolescents. This adult patient, who has no headache or other worrisome symptoms, does not require screening. This patient does not have daily pain or pain on the majority of days, so he does not need daily long-acting opioid therapy. Blood transfusions in asymptomatic patients can lead to iron overload in patients with sickle cell disease. There is no consensus regarding the hemoglobin level that should automatically prompt a blood transfusion in these patients.

A 60-year-old male presents with left lower quadrant abdominal pain. His medical and surgical histories are remarkable only for a history of hypertension controlled with hydrochlorothiazide and lisinopril (Prinivil, Zestril), and no polyps seen on screening colonoscopy 5 years ago. He is afebrile, and a physical examination is notable only for mild abdominal tenderness in the left lower quadrant without peritoneal signs. A urinalysis is normal. You diagnose mild diverticulitis. Which one of the following management options would be indicated at this time? A) Rest and clear liquids B) Avoidance of seeds, nuts, and popcorn C) Abdominal CT D) Referral for colonoscopy E) Hospital admission for intravenous fluids and intravenous antibiotics

ANSWER: A In patients with mild diverticulitis, outpatient management with rest and oral fluids is preferred. Avoidance of seeds, nuts, and popcorn does not reduce recurrence rates. CT of the abdomen may be indicated if the diagnosis is uncertain or if complications are suspected. Colonoscopy is contraindicated acutely and is only necessary for follow-up when age-appropriate cancer screening is indicated, or in cases of complicated disease. Antibiotics may not be necessary in all cases, and hospital admission is unnecessary for mild cases.

A 72-year-old female comes to your office to discuss newly diagnosed peripheral artery disease. She has a history of long-standing type 2 diabetes, hypertension, and hyperlipidemia. Her current medications include atorvastatin (Lipitor), 80 mg daily; metformin (Glucophage XR), 2000 mg daily; lisinopril (Prinivil, Zestril), 40 mg daily; and aspirin, 81 mg daily. On examination she has a blood pressure of 114/62 mm Hg. Laboratory studies reveal an LDL-cholesterol level of 68 mg/dL and a hemoglobin A1c of 7.7%. If added to her current regimen, which one of the following medications would most likely improve her walking distance? A) Cilostazol B) Clopidogrel (Plavix) C) Ezetimibe (Zetia) D) Liraglutide (Victoza) E) Warfarin (Coumadin)

ANSWER: A In patients with peripheral artery disease (PAD), cilostazol, a phosphodiesterase inhibitor with antiplatelet and vasodilatory properties, increases the maximal walking distance on a treadmill by approximately 25% compared with placebo. The side effects include tachycardia, diarrhea, and increased bleeding tendency, and it is contraindicated in patients with heart failure or a low ejection fraction. A supervised exercise program is also an important component of a comprehensive approach to PAD. In one trial a supervised exercise program resulted in an increase of 2.1 minutes of mean peak walking time at 6 months. Clopidogrel is an antiplatelet agent that has been shown to be slightly more effective than aspirin in reducing the risk of a composite outcome of ischemic stroke, myocardial infarction, or death from vascular causes. Dual antiplatelet therapy can be considered to reduce the risk of cardiovascular events though it carries an increased risk of bleeding. Warfarin has no evidence of benefit in PAD and adds to bleeding risk. Statins are associated with a lower risk of cardiovascular events in patients with PAD. Ezetimibe has no evidence of additional benefit. ACE inhibitors appear to be the preferred blood pressure agent for patients with PAD given the findings of the HOPE trial, in which ramipril resulted in a lower risk of adverse cardiovascular outcomes compared to placebo over 5 years of follow-up. Blood pressure targets are not clear in PAD but this patient's blood pressure is already <120/75 mm Hg and she is unlikely to benefit from additional blood pressure lowering. Intensifying therapy for this patient's type 2 diabetes by adding liraglutide also has no evidence of benefit for PAD.

A 12-year-old female is brought to your office because of a lesion on her left lateral upper eyelid. The lesion started 3 days ago as a small, red "pimple," and since then it has increased in size and is tender. She does not have any fever, tearing, or conjunctival irritation. No treatment has been attempted. Which one of the following would be the most appropriate initial management? A) Warm, damp compresses for 10 minutes four times daily B) Topical erythromycin ophthalmic ointment (Ilotycin) C) Topical hydrocortisone 0.5% D) Systemic antibiotics with coverage for staphylococci E) Incisional drainage using a sterile 18- or 20-gauge needle

ANSWER: A This patient presents with an external hordeolum or stye. Warm, damp compresses for 10 minutes four times a day would be the best initial management. Topical erythromycin ointment and topical hydrocortisone cream are not indicated for treatment of a stye. Although staphylococci are commonly involved in this process, antibiotics are not recommended unless there is evidence of adjoining cellulitis. Warm compresses allow for spontaneous drainage and resolution. Anti-inflammatory medications are not recommended for hordeolum externum management, however they could become necessary if it becomes a chalazion. If the hordeolum has not resolved in about 1 week, incision may be necessary.

A 3-year-old male is brought to your office by his mother because he has been limping. The mother reports that she saw him fall while playing at the park with some other children yesterday, but that he seemed fine afterward. This morning, however, he did not want to put any weight on the leg when he got up. The child has also had a fever off and on for 3-5 days. His mother states that he had a temperature of 102°F today. The child does not want to bear weight on the affected extremity. Plain film imaging of both lower extremities is negative. Which one of the following would be the most appropriate next step in the workup? A) A CBC, erythrocyte sedimentation rate, and C-reactive protein level B) Joint aspiration C) Ultrasonography D) Bone scintigraphy E) MRI

ANSWER: A In this scenario trauma is a consideration due to the patient's history of falling. However, with a history of fever >38.5°C (101.3°F) it is appropriate to order a CBC, erythrocyte sedimentation rate (ESR), and C-reactive protein (CRP) level as the next step in the evaluation. An oral temperature >38.5°C, refusal to bear weight on the affected leg, an ESR >40 mm/hr, a peripheral WBC count >12,000/mm3, or a CRP level >20 mg/L should raise suspicion for septic arthritis (SOR C). Septic arthritis of the hip should be suspected in this patient. As the evaluation progresses, joint aspiration may be considered to evaluate for septic arthritis or transient synovitis. MRI may also be appropriate as the evaluation progresses after a CBC, ESR, and CRP level. Ultrasonography and bone scintigraphy would not be a consideration at this time. For hip effusions in children, ultrasonography is recommended over plain radiography, but it does not differentiate between sterile, purulent, and hemorrhagic effusions.

A 59-year-old male presents with pain over the lateral distal aspect of his right foot. He states that he accidentally struck a filing cabinet with his foot about 5 days ago. He immediately had pain and swelling over the dorsum of his foot and his second to fifth toes, and there has been no improvement. He has tried buddy taping his toes but this has not provided any relief. A physical examination is notable for trace edema over the dorsum of the right foot and tenderness to palpation over the distal fifth metatarsal and fifth phalanx. The foot is neurovascularly intact with a strong dorsalis pedis pulse and normal sensation. A radiograph of the right foot shows a minimally displaced oblique right fifth proximal phalanx shaft fracture. In addition to pain control, which one of the following would be most appropriate? A) A rigid-sole shoe and weight bearing as tolerated B) A pneumatic brace and weight bearing as tolerated C) A short leg walking cast and weight bearing as tolerated D) A posterior splint and no weight bearing E) Immediate referral to an orthopedic surgeon and no weight bearing

ANSWER: A Lesser toe fractures such as this typically can be managed conservatively with buddy taping and a rigid-sole shoe. Because this patient has already tried buddy taping, it is appropriate to have him use a rigid-sole shoe. Pneumatic braces can be used for some nondisplaced tuberosity avulsion fractures. Metatarsal shaft fractures are often initially treated with a posterior splint and then transitioned to a walking cast. Referral to an orthopedic surgeon is typically limited to patients with a high-risk fracture such as a displaced Jones fracture, or to patients who are highly competitive athletes.

A 3-year-old female is brought to your office by her mother with a 3-day history of a rash. The patient's mother reports that the child is eating less than usual but tolerating fluids. The mother also reports that other children in the patient's day care have a similar rash. On examination the patient's temperature is 38.2°C (100.8°F). The patient appears alert and active. Examination of the pharynx reveals ulcerations in the posterior oral cavity and soft palate. There is a maculopapular rash on the fingers, soles of the feet, and genitals. The remainder of the examination is normal. Which one of the following would be the most appropriate next step? A) Symptomatic treatment only, and increased oral hydration B) Amoxicillin twice daily for 7 days C) Valacyclovir (Valtrex) twice daily for 7 days D) Antinuclear antibody testing E) Rapid plasma reagin testing

ANSWER: A This patient has hand-foot-and-mouth disease frequently caused by enterovirus 71 or coxsackievirus A16. It is most common in children under 5 years of age and occurs most often in the fall and spring. It is characterized by painful maculopapular or papulovesicular lesions on the hands and feet, and in the oral cavity. Lesions can also appear on the genitals, trunk, or cheek. Management includes symptomatic treatment of pain and oral hydration. Antibiotics and antiviral treatment are not recommended. Laboratory testing is not appropriate for this condition.

Which one of the following oral iron preparations is most effective for the treatment of iron deficiency anemia in a patient with non-dialysis-dependent chronic kidney disease? A) Ferric citrate (Auryxia) B) Ferrous fumarate C) Ferrous gluconate D) Ferrous sulfate

ANSWER: A Oral iron supplements, including ferrous fumarate, ferrous gluconate, and ferrous sulfate, are generally ineffective when used by hemodialysis patients and are only modestly effective when used by patients with non-dialysis-dependent chronic kidney disease. The one exception is ferric citrate, which is highly efficacious in all patients with chronic kidney disease. Intravenous iron preparations such as iron dextran are similarly effective in both groups and are considered the gold standard for the treatment of iron deficiency in patients on chronic hemodialysis.

An obese 45-year-old female returns for follow-up to discuss recent laboratory results. She has known hypertension and takes amlodipine (Norvasc), 5 mg daily. She has bilateral knee osteoarthritis and uses ibuprofen as needed for pain. Recent laboratory studies show a fasting glucose level of 120 mg/dL, normal electrolytes, and normal kidney and liver function. She has a total cholesterol level of 203 mg/dL, an LDL-cholesterol level of 134 mg/dL, an HDL-cholesterol level of 46 mg/dL, and a triglyceride level of 260 mg/dL. She has a strong family history of diabetes mellitus and cardiovascular disease. Which one of the following has been shown to be most effective in preventing diabetes in patients such as this? A) Participation in the National Diabetes Prevention Program B) Acarbose (Precose) C) Liraglutide (Victoza) D) Metformin (Glucophage)

ANSWER: A Patients who participate in long-term lifestyle intervention programs such as the CDC's National Diabetes Prevention Program have an approximately 30% reduction in progression to type 2 diabetes. These programs promote weight loss of 7% of body weight and encourage physical activity with a weekly goal of 150 minutes of moderate-intensity exercise. Because an estimated 1 in 3 adults in the United States has prediabetes (diagnosed by a fasting glucose level of 100-125 mg/dL, a hemoglobin A1c of 5.7%-6.4%, or a 2-hour plasma glucose level of 140-199 mg/dL), interventions to reduce progression to full diabetes are vital. Weight loss of 5% of body weight alone is not as effective as a comprehensive prevention program. Metformin is also effective in reducing the progression to diabetes, but it is not as effective as lifestyle intervention programs for most patients. While alpha-glucosidase inhibitors have been shown to reduce the incidence of diabetes in patients with prediabetes, they are not as effective as metformin, which can reduce the progression by 18%. There is good evidence that liraglutide leads to weight loss and the lowering of blood glucose, but it has not been found to be as effective as lifestyle intervention programs or metformin in reducing progression to diabetes.

Long-term oxygen therapy is recommended for patients with COPD who have a resting oxygen saturation at or below a threshold of A) 88% B) 89% C) 90% D) 93% E) 95%

ANSWER: A Patients with moderate to severe COPD should be evaluated periodically for hypoxemia to determine the need for long-term oxygen therapy. In a Cochrane review of six randomized, controlled trials, oxygen therapy improved survival in select patients with COPD and severe resting hypoxemia (a resting arterial partial pressure of oxygen <55 mm Hg or an oxygen saturation <88%).

A 42-year-old male sees you for follow-up after his third episode of pneumonia. He has no other significant medical history. He has never smoked, drinks alcohol occasionally, and has no other drug use or known exposures. A physical examination is normal. Pulmonary function tests demonstrate an FEV1 of 72% of predicted and an FEV1/FVC ratio of 0.68, which does not normalize with bronchodilator administration. A chest radiograph shows hyperinflation but no other significant findings. Laboratory Findings Platelets 102,000/mm3 (N 150,000-450,000) Creatinine 0.7 mg/dL (N 0.6-1.2) AST 56 U/L (N 8-48) ALT 43 U/L (N 7-55) Albumin 3.3 g/dL (N 3.5-5.0) Which one of the following conditions best explains this patient's abnormal findings? A) alpha1-Antitrypsin deficiency B) Cystic fibrosis C) Goodpasture syndrome D) Hereditary hemochromatosis E) Sarcoidosis

ANSWER: A This patient is in his forties without clear risk factors and has both an irreversible obstructive pulmonary defect consistent with COPD (FEV1 <80% of predicted and an FEV1/FVC ratio <0.70) and liver abnormalities associated with advanced fibrosis. These combined findings are the hallmark of alpha1-antitrypsin deficiency. This patient has a high likelihood of advanced liver fibrosis based on the low albumin level and noninvasive scoring using the fibrosis-4 (FIB-4) index (age × AST/(platelets [in mm3] × ALT½ = 3.52 for this patient). Further evaluation with transient elastography should be performed to confirm cirrhosis. Cystic fibrosis is less likely to cause a typical obstructive picture on pulmonary function tests (PFTs) and does not typically cause liver fibrosis. Goodpasture syndrome is a vasculitis that classically involves the lungs and the kidneys and is more likely to cause a restrictive pattern on PFTs. Hereditary hemochromatosis can cause early liver disease, including cirrhosis, but is not a significant cause of respiratory disease. Sarcoidosis may involve both the lung and the liver, and can cause obstructive or restrictive patterns on PFTs. However, this patient's radiograph did not show the characteristic hilar adenopathy and granulomatous disease of sarcoidosis.

A 24-year-old female presents with a 2-day history of mild to moderate pelvic pain. She has had two male sex partners in the last 6 months and uses oral contraceptives and sometimes condoms. A physical examination reveals a temperature of 36.4°C (97.5°F) and moderate cervical motion and uterine tenderness. Urine hCG and a urinalysis are negative. Vaginal microscopy shows only WBCs. The initiation of antibiotics for treatment of pelvic inflammatory disease in this patient A) is appropriate at this time B) requires an elevated temperature, WBC count, or C-reactive protein level C) should be based on the results of gonorrhea and Chlamydia testing D) should be based on the results of pelvic ultrasonography

ANSWER: A Pelvic inflammatory disease (PID) is a clinical diagnosis, and treatment should be administered at the time of diagnosis and not delayed until the results of the nucleic acid amplification testing (NAAT) for gonorrhea and Chlamydia are returned. The clinical diagnosis is based on an at-risk woman presenting with lower abdominal or pelvic pain, accompanied by cervical motion, uterine, or adnexal tenderness that can range from mild to severe. There is often a mucopurulent discharge or WBCs on saline microscopy. Acute phase indicators such as fever, leukocytosis, or an elevated C-reactive protein level may be helpful but are neither sensitive nor specific. A positive NAAT is not required for diagnosis and treatment because an upper tract infection may be present, or the causative agent may not be gonorrhea or Chlamydia. PID should be considered a polymicrobial infection. Pelvic ultrasonography may be used if there is a concern about other pathology such as a tubo-ovarian abscess.

A 16-year-old Muslim male with type 1 diabetes would like to start fasting during Ramadan. He currently takes insulin glargine (Lantus) every morning and regular insulin with meals. Which one of the following is the most appropriate recommendation for this patient during Ramadan? A) Avoid fasting because it is unsafe in patients with type 1 diabetes B) Decrease insulin glargine by 50% C) Decrease insulin glargine by 50% and take it at night D) Decrease insulin glargine by 25% and use sliding scale regular insulin with meals E) Stop insulin glargine and use sliding scale regular insulin four times a day

ANSWER: A Ramadan is a holy month during which Muslims fast from dawn until sunset. It is an obligation for all healthy adult Muslims. The Quran does exempt the sick from fasting, but many ill people will still fast. Recommendations in the literature include risk stratification criteria to determine whether fasting is safe for patients with diabetes mellitus. Generally, patients with well controlled type 2 diabetes can safely fast with adjustments in their medications; however, it is thought that fasting is unsafe for patients with type 1 diabetes. Fasting can increase the risk of severe hypoglycemia as well as hyperglycemia and ketoacidosis if medications are withheld.

A 3-year-old female is brought to your office by her mother with a 3-day history of cough, and you diagnose a common cold. The mother asks for a recommendation to relieve the cough. You tell her that the safest and most effective management for cough would be A) honey B) codeine C) dextromethorphan D) diphenhydramine E) ibuprofen

ANSWER: A Safe and effective options to treat cough in young children include nasal saline irrigation, a menthol rub, and honey (in children 12 months of age or older). Codeine should no longer be used for cough in anyone under 18 years of age. Over-the-counter cough and cold medications are not recommended for children under 4 years of age due to the lack of evidence of benefit and the significant side effects. Ibuprofen has not been shown to be effective for cough.

Which one of the following developmental milestones would be expected in a typical 12-month-old child? A) Standing independently B) Identifying at least two body parts C) Using three words other than names D) Scribbling spontaneously E) Building a three-cube tower

ANSWER: A Standing independently is an expected developmental milestone for a 12-month-old child. Identifying at least two body parts, using three words other than names, scribbling spontaneously, and building a three-cube tower are expected milestones for older children.

A previously healthy 62-year-old female presents to your office with a 3-day history of fever and a cough productive of purulent sputum. On examination she has a temperature of 39.2°C (102.6°F), a blood pressure of 110/70 mm Hg, a pulse rate of 92 beats/min, a respiratory rate of 25/min, and an oxygen saturation of 94% on room air. She shows no signs of confusion. An examination is significant for crackles at the right lower lung base and a chest radiograph confirms an infiltrate in the same location. Which one of the following treatment settings would be most appropriate at this time for this patient's community-acquired pneumonia? A) Outpatient B) The emergency department C) A regular hospital inpatient floor D) The intensive-care unit

ANSWER: A The CRB-65 (confusion, respiratory rate, blood pressure, 65 years of age) rule is a validated tool for risk stratification in the primary care setting. It can be used to determine who is a good candidate for outpatient treatment of community-acquired pneumonia. Patients are given 1 point for each of the following signs or symptoms: new-onset confusion, a respiratory rate >30/min, a blood pressure <90 mm Hg systolic or <60 mm Hg diastolic, and an age of 65 years or older. Patients with 0 points, such as this patient, are at low risk and can be managed in the outpatient setting unless there are other significant comorbidities or social factors that make outpatient treatment contraindicated. Patients with a score of 1-2 are at moderate risk and should be hospitalized in most cases. Patients with a score of 3-4 are at high risk and should be considered for hospitalization in an intensive-care unit.

A 42-year-old male comes to your office for a health maintenance evaluation. He has not been to your office in the past 5 years and has no medical issues. He tells you that his father was diagnosed with hyperlipidemia and hypertension in his sixties. He recently used an online clinic where he could get free medical advice and testosterone testing. He was advised to see you for a stress test. He has recently started an exercise program and is walking 5 days a week for 30 minutes a day with no significant chest pain or shortness of breath. He wants to start running and work up to a 5K race. Based on U.S. Preventive Services Task Force and other expert guidelines on stress testing, which one of the following would you advise for this patient? A) No stress testing B) An exercise EKG C) Exercise echocardiography D) An exercise sestamibi stress test E) Dobutamine stress echocardiography

ANSWER: A The U.S. Preventive Services Task Force, the American College of Physicians, the American College of Cardiology, and the American Academy of Family Physicians recommend against cardiac screening with stress testing in low-risk asymptomatic individuals, so an exercise EKG, exercise echocardiography, an exercise sestamibi stress test, and dobutamine stress echocardiography would not be recommended for this patient. Cardiovascular screening tests in asymptomatic patients have a low yield and may produce many false positives, leading to costly and potentially harmful invasive procedures. Many patients ask for this type of screening, but explaining current guidelines may help them understand why they should not be screened.

A 55-year-old male with a 40-pack-year smoking history comes to your office with the results of spirometry he had at a health fair. He quit smoking 1 year ago. He does not have any cough, dyspnea, wheezing, or sputum production, but he is concerned that the spirometry results show an FEV1/FVC ratio of 0.65 and an FEV1 of 70% of predicted, which indicates mild to moderate airflow obstruction. Based on the best available evidence, which one of the following should you recommend in order to prevent the development of symptomatic airflow obstruction? A) No treatment B) A long-acting anticholinergic C) A long-acting beta-agonist D) An inhaled corticosteroid E) Combination therapy with a corticosteroid and long-acting -betaagonist

ANSWER: A There is no evidence from randomized, controlled trials to show that treating asymptomatic individuals, with or without risk factors for airflow obstruction, prevents future respiratory symptoms or reduces subsequent declines in lung function. Partly for this reason, the U.S. Preventive Services Task Force and joint guidelines issued by the American College of Physicians, American College of Chest Physicians, American Thoracic Society, and European Respiratory Society recommend against screening for COPD in asymptomatic adults. Regardless of the results of this patient's spirometry testing, treatment should not be initiated in the absence of symptoms. A long-acting anticholinergic, a long-acting beta-agonist, an inhaled corticosteroid, and combination therapy with a corticosteroid and long acting beta-agonist would not be recommended for this patient.

An 18-month-old female is brought to your office by her mother for evaluation of a cough. The patient has had low-grade fevers and a runny nose for 2 days. She now has a cough that is worse at night. On examination she has a temperature of 37.5°C (99.5°F), a pulse rate of 120 beats/min, a respiratory rate of 30/min, and an oxygen saturation of 92% on room air. She is noted to have hoarseness, mild inspiratory stridor, and a barking cough. She does not have drooling or a muffled voice. Which one of the following should be ordered to confirm the diagnosis? A) No further testing B) A CBC C) A viral culture D) Rapid antigen testing E) A radiograph of the neck

ANSWER: A This patient has croup, which is diagnosed clinically and no further testing is usually indicated. A CBC is nonspecific and is usually only indicated if a bacterial cause of stridor is suspected, such as bacterial tracheitis, epiglottitis, retropharyngeal abscess, or peritonsillar abscess. Viral cultures and rapid antigen testing should be reserved for instances in which the patient fails to respond as expected to initial treatment. A neck radiograph is not indicated in the absence of findings that suggest possible epiglottitis, such as drooling or a muffled voice.

A 32-year-old female who is one of your longtime patients calls you because of a 24-hour history of painful urination with urinary frequency and urgency. She is otherwise healthy and does not have any fever, chills, back pain, or vaginal discharge. She uses an oral contraceptive pill and states that her last menstrual period was normal and occurred last week. Which one of the following would be most appropriate at this time? A) Empiric antibiotic treatment B) A urinalysis C) A urine culture D) Plain abdominal radiographs E) Pelvic ultrasonography

ANSWER: A This patient has symptoms of acute simple cystitis and does not have any symptoms that would suggest a complicated urinary tract infection or vaginal infection. In these cases treatment with oral antibiotic therapy may be prescribed without further evaluation (SOR B). Simple cystitis is a clinical diagnosis and a urinalysis and urine culture are not necessary. The patient does not have any symptoms that warrant evaluation with abdominal radiographs or pelvic ultrasonography.

You see an otherwise healthy 67-year-old male who is recovering in the hospital after a knee replacement. The patient's skin is cold and clammy and he is having trouble remembering where he is. He is receiving oxygen via nasal cannula, which has resolved some mild desaturations he was having this morning. The patient's temperature is 39.6°C (103.3°F) and his heart rate is 142 beats/min. The nurse tells you that the patient's WBC count was elevated this morning. Which one of the following interventions should be performed immediately? A) An infusion of intravenous fluids B) An infusion of dopamine C) An infusion of norepinephrine D) CT angiography

ANSWER: A This patient is demonstrating early symptoms of possible sepsis. The tachycardia, elevated temperature, and elevated WBC count meet three of the four criteria for systemic inflammatory response syndrome. If an infection is documented the patient would meet the criteria for sepsis. Early recognition and intervention are key to reducing mortality from sepsis. In this patient the hypoperfusion and confusion in the setting of recent surgery should raise suspicion for possible sepsis. The first step in resuscitation is to ensure adequate oxygenation, and this patient is already receiving supplemental oxygen, which has improved his oxygenation. An infusion of vasopressors is indicated when adequate fluid resuscitation fails to improve organ perfusion and normalize mean arterial pressures. Blood cultures will be important to guide definitive management, but fluid resuscitation is a more immediate priority. While patients with recent knee replacements are at risk for deep vein thrombosis and pulmonary embolism, initial fluid resuscitation and stabilization are more important and should not be delayed in order to perform CT angiography.

A 73-year-old female sees you for the first time for a health maintenance visit. Her medical history includes hyperlipidemia, GERD, insomnia, and osteoarthritis, but she is otherwise healthy. Her estimated 10-year risk of atherosclerotic cardiovascular disease is 14%. She lives independently. She has a past history of alcohol abuse but has not used alcohol in 20 years, and is a lifetime nonsmoker. Her current medication regimen includes aspirin, 81 mg once daily; melatonin, 3 mg at bedtime; acetaminophen, 500 mg three times daily; atorvastatin (Lipitor), 20 mg once daily; and famotidine (Pepcid), 20 mg once daily. Today her blood pressure is 130/70 mm Hg, pulse rate 72 beats/min, and oxygen saturation 95% on room air. Which one of the following changes to her current medication regimen would improve the likelihood of benefit and reduce the likelihood of harm? A) Stopping aspirin B) Stopping melatonin C) Stopping acetaminophen and starting diclofenac, 50 mg twice daily D) Stopping atorvastatin and starting rosuvastatin (Crestor), 40 mg at bedtime E) Stopping famotidine and starting omeprazole (Prilosec), 20 mg once daily

ANSWER: A This patient is generally healthy and highly functional at baseline. She has multiple medical problems but they are unlikely to significantly reduce her longevity. She has no known history of cardiovascular or cerebrovascular disease, so aspirin was being used for primary prevention. The U.S. Preventive Services Task Force (USPSTF) has stated that there is insufficient evidence to recommend for or against the use of aspirin for primary prevention of cardiovascular disease in adults over the age of 70. Since the USPSTF guidelines were published, evidence from two large randomized, controlled trials provided strong support for the discontinuation of aspirin for most older adults without prior cardiovascular disease, with an indication that it increases the risk of all-cause mortality. In this patient at low to moderate risk who is already taking a statin, the risk of continuing aspirin exceeds the potential benefit. There is no indication for changing this patient from a moderate-intensity to a high-intensity statin. For uncomplicated gastric reflux in older adults, H2-blockers are preferred over proton pump inhibitors, which are associated with an increased risk of infections and fractures. This patient has no concerns about osteoarthritis, and although she has a remote history of alcohol abuse, her dosing of daily acetaminophen is well below the threshold of concern for liver injury. Switching to an NSAID such as diclofenac would place her at risk for short-term and long-term renal complications. Melatonin is not known to have long-term adverse effects in older adults.

A 66-year-old male with a history of diabetes mellitus, hypertension, and venous insufficiency presents with a 3-cm diameter ulceration on his lateral lower leg. Recent vascular studies show no significant arterial occlusion and a foot monofilament examination is normal. You assess the wound and do not find significant devitalized tissue or signs of infection. Which one of the following interventions is most likely to promote wound healing? A) A compression bandage B) Dermal matrix dressing C) Hydrogel dressing D) Wet-to-dry dressings E) Prescription diabetic footwear

ANSWER: A Treatment of chronic wounds is a common clinical challenge in primary care. This case represents a venous stasis ulceration rather than a diabetic foot wound or pressure ulcer. When treating a wound, the treating physician should optimize factors that promote healing, including nutrition, smoking cessation, and blood glucose control. Local wound care should emphasize treating any infection, debriding devitalized tissue, avoiding ongoing pressure or shear force, and maintaining a clean and appropriately moist healing environment. Compression hose and bandages have been shown to improve healing times in venous stasis ulcers (SOR B). Wet-to-dry dressings or specialized dressings such as hydrogel or skin substitutes such as dermal matrix dressings have not been consistently shown to decrease healing times. Prescription diabetic footwear may be an important part of offloading diabetic foot wounds, but it is not relevant to a wound on the lower leg.

A 62-year-old male with hypertension presents to your office with substernal chest pain radiating into his left arm for the past 20 minutes. He also has diaphoresis and nausea. He has a blood pressure of 156/96 mm Hg, a pulse rate of 84 beats/min, and an oxygen saturation of 93% on room air. An EKG shows ST-segment elevations in leads V1 and V2. Your medical assistant calls 911 for immediate transport to the local hospital's emergency department. While awaiting the ambulance's arrival you give the patient low-dose aspirin and sublingual nitroglycerin. Which one of the following would be most appropriate regarding oxygen therapy at this time? A) No oxygen therapy B) Oxygen via nasal cannula at 2 L/min C) Oxygen via nasal cannula at 6 L/min D) 100% oxygen with a regular mask E) 100% oxygen with a nonrebreathing mask

ANSWER: A While oxygen supplementation is routinely initiated for patients who are suspected of having acute coronary syndrome, evidence does not support a benefit from this unless the patient is hypoxic. Oxygen supplementation is recommended if the patient has an oxygen saturation <90%, if the patient is at risk for hypoxemia, or if the patient is in respiratory distress.

You advise a 58-year-old male with chronic kidney disease to limit his salt intake. He asks about the benefit of this change, since he thoroughly enjoys his high-salt diet. For patients with chronic kidney disease, current evidence shows that a low-salt diet A) decreases mortality B) lowers blood pressure C) increases proteinuria D) increases the time to end-stage renal disease and the need for dialysis

ANSWER: B A recent Cochrane review revealed that a low-salt diet consistently lowers blood pressure in patients with chronic kidney disease (CKD). Current evidence is lacking as to whether a low-salt diet leads to decreased mortality or an increased time to dialysis. A low-salt diet has also been shown to decrease proteinuria in patients with CKD.

A 46-year-old female with symptoms of solid and liquid dysphagia undergoes esophageal manometry for suspected achalasia. These symptoms of achalasia are caused by which one of the following pathophysiologic mechanisms? A) More than 50% of swallows being weak or failed B) Aperistalsis in the distal two-thirds of the esophagus, with incomplete lower esophageal sphincter relaxation C) High-pressure esophageal contractions, with normal relaxation of the esophagogastric junction D) Increased premature contractions of the distal esophagus, with normal relaxation of the esophagogastric junction E) Mechanical obstruction due to an upper esophageal web

ANSWER: B Achalasia is associated with the loss of ganglion cells in the esophageal wall, which leads to the loss of normal esophageal peristalsis and failure of relaxation of the lower esophageal sphincter (LES). Incomplete LES relaxation is highly specific for achalasia. Ineffective esophageal motility is demonstrated by an inability to generate an effective swallow >50% of the time during motility testing. Hypercontractile esophagus, also known as jackhammer esophagus, leads to high-pressure contractions in the esophagus but normal LES functioning. Diffuse esophageal spasm leads to increased premature contractions of the esophagus with normal functioning of the LES. Esophageal webs are mechanical obstructions unrelated to achalasia.

The U.S. Preventive Services Task Force recommends vision screening to detect amblyopia A) at 2 years of age B) at 3-5 years of age C) the summer before the child enters first grade D) once at 4-5 years of age and once at 8-9 years of age

ANSWER: B Amblyopia is one of the most common causes of vision abnormalities in children, and early detection and treatment can help prevent vision loss. The U.S. Preventive Services Task Force recommends vision screening for all children at least once between 3 and 5 years of age to detect the presence of amblyopia or its risk factors (B recommendation).

A 42-year-old male sees you because of abnormal laboratory results from a recent health fair. Laboratory Findings Albumin 4.1 g/dL (N 3.5-5.5) Alkaline phosphatase 88 U/L (N 36-92) AST 152 U/L (N 10-59) ALT 70 U/L (N 13-40) Total bilirubin 0.9 mg/dL (0.3-1.2) Based on these results, which one of the following is the most likely cause of his elevated liver transaminase levels? A) Acute viral hepatitis B) Alcoholic liver disease C) Cholestasis D) Nonalcoholic fatty liver disease

ANSWER: B An AST/ALT ratio >2 is highly suggestive of alcoholic liver disease. Acute viral hepatitis usually presents with AST and ALT levels >25 times the upper limit of normal. Cholestasis is unlikely to be the cause in a patient with normal alkaline phosphatase and bilirubin levels. Although nonalcoholic fatty liver disease is the most common cause of asymptomatic elevated liver transaminase levels in the United States, it is usually associated with an AST/ALT ratio <1.

A 30-year-old male has moderate to severe rectal pain that occurs with bowel movements and lasts for several hours. He reports minimal bright red blood on the toilet tissue. On examination he is noted to have an anal fissure in the posterior midline. Sitz baths and fiber supplementation have provided no relief, and topical nifedipine therapy did not resolve his symptoms. Which one of the following would be an effective treatment? A) Beta-Blocker injection B) Botulinum toxin injection C) Corticosteroid injection D) Lidocaine injection E) Rubber band ligation

ANSWER: B An anal fissure is a longitudinal tear in the anoderm that occurs most often midline. The initial treatment is sitz baths and increased fiber intake. If there is no improvement, botulinum toxin is indicated. Topical nitroglycerin and diltiazem may also be used. Neither Beta-blocker injections nor corticosteroid injections are indicated. Lidocaine would relieve pain but not promote healing. Rubber band ligation is indicated for the treatment of internal hemorrhoids.

Which one of the following cancers consistently declined in both incidence and mortality between 1975 and 2015? A) Breast cancer B) Cervical cancer C) Hodgkin's lymphoma D) Lung cancer E) Prostate cancer

ANSWER: B As a result of an effective screening program in a disease with a precancerous phase amenable to identification and treatment, cervical cancer declined in both incidence (rate of disease diagnosis) and mortality (rate of death from a specific cause) between 1975 and 2015. The incidence of breast cancer has risen over time as mammography screening has become widespread. Mortality from breast cancer has declined over time, although there is some debate as to whether this is related to screening or improved treatments. Hodgkin's lymphoma incidence has remained stable over time, suggesting no change to the causes of this cancer, but mortality has declined considerably as treatments have improved. Lung cancer incidence and mortality both increased in the 1970s and 1980s before they began to decline, mirroring the rise and fall of the most potent risk factor for lung cancer, which is cigarette smoking. Prostate cancer saw a steep rise in incidence with the rollout of screening with PSA testing in the early 1990s. The incidence has since declined, as has mortality from prostate cancer.

A 45-year-old female presents to the emergency department with a 1-week history of facial swelling and progressive dyspnea with exertion. She was diagnosed 1 week ago with non-Hodgkin's lymphoma but her medical history is otherwise unremarkable. After hospital admission, which one of the following would be the most appropriate next step in the management of this condition? A) Intravenous antibiotics B) Urgent chemotherapy and radiation C) Urgent chemotherapy and plasmapheresis D) Urgent echocardiography E) Urgent bronchoscopy

ANSWER: B Because of the prevalence of cancer in the United States, it is important for family physicians to recognize oncologic emergencies. This patient presents with signs and symptoms related to superior vena cava syndrome, which is caused by compression of the superior vena cava. This is most often caused by lung cancer or lymphoma, but it can also be related to indwelling catheters, lymph nodes, or metastatic tumors. After ensuring that the patient is hospitalized and stable, the initial treatment options include intravenous corticosteroids, chemotherapy, radiation, and occasionally intravascular stenting. Antibiotics are not warranted because this condition is not the result of an infection. Hyperviscosity syndrome is another oncologic emergency associated with leukemia, multiple myeloma, and Waldenström's macroglobulinemia. It is treated with chemotherapy and plasmapheresis. Echocardiography and bronchoscopy are not indicated in the initial management of superior vena cava syndrome.

A fully immunized 7-month-old male is brought urgently to your office after his parents noted a possible seizure. The mother says that the infant began to "shake all over" for about 3-4 minutes and then promptly fell asleep for about 20 minutes. When he awoke he was alert but fussy and crying. He has been ill for the last few days with a cough, congestion, decreased oral intake, and fevers up to 101°F. On examination he has an oral temperature of 38.3°C (100.9°F), a heart rate of 170 beats/min, a respiratory rate of 50/min, and an oxygen saturation of 97% on room air. The infant is fussy but consolable. His mucous membranes are moist, his tympanic membranes are clear, and he has a normal oropharynx. He has clear rhinorrhea. Examination of the heart is normal, and examination of the lungs reveals rhonchi and wheezes. He is moving all of his extremities normally. Which one of the following would be the most appropriate initial step in the evaluation of this child? A) A basic metabolic panel B) Radiography of the chest C) MRI of the brain D) Electroencephalography E) A lumbar puncture

ANSWER: B Children who have a simple febrile seizure and appear neurologically intact do not require routine testing except to determine the source of their fever (SOR C). This child has signs of possible pneumonia so a chest radiograph would be warranted to look for the source of infection that triggered the fever. Routine laboratory testing is not indicated in the workup of simple febrile seizures. There is a low risk that these children will have low sodium or glucose levels, and this would not predict seizure recurrence. Routine neuroimaging such as MRI is not recommended for febrile seizures. Electroencephalography is not useful for predicting the recurrence of simple febrile seizures and would not be indicated in the workup of these seizures. A lumbar puncture is indicated only in cases where the child has neurologic findings suggestive of meningitis, but that is not the case for this child.

A 9-year-old male with a history of moderate persistent asthma is brought to the emergency department with an acute exacerbation. His symptoms began with a runny nose and nasal congestion 2 days ago. His parents state that he has not had any fevers or chills and he was eating and drinking well until a few hours ago when his breathing started to appear more labored. After multiple treatments with inhaled albuterol (Proventil, Ventolin) and oral prednisolone he remains tachypneic and wheezy. Which one of the following intravenous medications should be added to the patient's current treatment to reduce the likelihood of hospital admission? A) Ketorolac B) Magnesium sulfate C) Methylprednisolone D) Omalizumab (Xolair) E) Theophylline

ANSWER: B Children who present to the emergency department with an asthma exacerbation and fail to improve adequately with inhaled short-acting bronchodilators and corticosteroids may benefit from treatment with intravenous (IV) magnesium sulfate. A 2016 Cochrane review of three randomized, controlled trials found that this reduced hospital admissions by 68%. Ketorolac is not known to have any benefit in the treatment of asthma. Oral administration of corticosteroids is as effective as IV administration, so there is no reason to give IV methylprednisolone. Omalizumab may be used to prevent exacerbations in patients with severe asthma who do not achieve adequate control with high-dose inhaled corticosteroids, but it has no role in the management of acute exacerbations. IV theophylline is not recommended for asthma exacerbations given its safety profile and poor efficacy compared to short- acting bronchodilators.

A 19-year-old football player presents to your office after being removed from a game when he sustained a head injury. He has vomited twice since the injury and is disoriented. You determine that imaging is indicated. Which one of the following would be most appropriate at this time? A) Plain radiography of the skull B) CT of the head C) MRI of the brain D) Functional MRI of the brain

ANSWER: B Evaluation of uncomplicated concussions does not require imaging. When a more severe injury is evident, the modality of choice is CT. Plain radiographs have minimal utility in the evaluation of head trauma. MRI is most appropriate for evaluation of prolonged symptoms. Functional MRI is still a research tool for evaluating brain injuries.

Which one of the following is most likely to cause a false-positive urine drug screen for amphetamines? A) Amlodipine (Norvasc) B) Bupropion (Wellbutrin) C) Levofloxacin (Levaquin) D) Pantoprazole (Protonix) E) Sertraline (Zoloft)

ANSWER: B False-positive results on drug testing can occur from cross-reactivity of commonly used medications with the assay. Multiple commonly used medications are known to cause a false-positive result for amphetamines, including bupropion, labetalol, ranitidine, and trazodone. Amlodipine is not implicated in abnormal drug screen results. Levofloxacin can cause a false-positive result for opioids. Proton pump inhibitors such as pantoprazole are known to cause a false-positive result for cannabinoids. Sertraline can cause a false-positive result for benzodiazepines.

In patients hospitalized with acute respiratory infections, procalcitonin levels are useful for A) nutritional status assessment B) reducing inappropriate use of antibiotics C) ruling out pulmonary embolism D) early identification of the syndrome of inappropriate secretion of antidiuretic hormone (SIADH) E) identifying acute respiratory distress syndrome

ANSWER: B Procalcitonin is a biomarker for the presence of severe bacterial infections such as pneumonia and sepsis. Its utility in clinical decision-making is reducing unnecessary antibiotic use, which reduces antibiotic resistance. Its use in guiding treatment is associated with reduced mortality (SOR A) and with fewer antibiotic treatment days and fewer antibiotic complications (SOR B). Procalcitonin levels are not useful in assessing nutritional status, ruling out pulmonary embolism, early identification of the syndrome of inappropriate secretion of antidiuretic hormone, or identifying acute respiratory distress syndrome.

A 16-year-old female presents with chronic acne on her nose, forehead, and chin consisting of a few comedones and a few mildly inflamed papules and pustules. She says it is minimally improved after 12 weeks of daily adapalene 0.1% gel. There are no scars or cysts. The patient would like to try to achieve better control. Which one of the following would you recommend at this time? A) Continue adapalene 0.1% gel for 12 more weeks B) Add clindamycin (Cleocin T) 1% gel for up to 12 weeks C) Add clindamycin 1% gel for maintenance D) Stop adapalene 0.1% gel and start clindamycin 1% gel for maintenance E) Stop adapalene 0.1% gel and start erythromycin 2% gel for maintenance

ANSWER: B Family physicians are often asked to manage mild to moderate acne vulgaris. Topical retinoids such as adapalene and benzoyl peroxide are first-line therapy and a trial of therapy is typically 8-12 weeks. Topical antibiotics may be added to topical retinoids or benzoyl peroxide to achieve better symptom control. To decrease emerging antibiotic resistance, studies support limiting antibiotic use to 12 weeks except in severe cases, not using antibiotics as monotherapy, and using clindamycin rather than erythromycin. Adding clindamycin gel rather than erythromycin gel for up to 12 weeks is recommended for this patient at this time.

A 60-year-old male with a long-standing history of type 2 diabetes is admitted to the hospital. He takes four oral medications for the treatment of diabetes at home. You decide to switch him to insulin instead of continuing oral medications while he is hospitalized. He is eating his meals well. After calculating the total daily insulin dose, which one of the following would be most appropriate? A) Administer the total daily dose as long-acting insulin in equal doses every 12 hours B) Administer half of the total daily dose of insulin as long-acting insulin and the other half as short-acting insulin in three divided doses, given with each meal C) Administer the total daily dose as short-acting insulin in three divided doses, given with each meal D) Administer the total daily dose as short-acting insulin in four divided doses, given with each meal and at bedtime E) Administer the total daily dose as a short-acting sliding scale regimen based on bedside glucose readings, in four divided doses

ANSWER: B Frequently patients taking oral medications for the treatment of diabetes mellitus need to be switched to insulin while hospitalized. There are formulas to calculate the total daily dose based on weight, renal function, insulin resistance, and other factors. The recommended regimen is half of the calculated total daily dose given as long-acting insulin such as glargine to provide basal insulin and half given as short-acting insulin such as lispro to provide prandial insulin. The short-acting insulin is divided into thirds to be given with each meal. The American Diabetes Association (ADA) recommends an insulin regimen with a basal and a prandial component for non-critically ill patients in the hospital with good nutritional intake. A correction component can be added to this regimen. The ADA strongly discourages the use of only a sliding scale insulin regimen. The reactive nature of sliding scale does not control glucose levels well and does not address the basal insulin needs of patients.

A 66-year-old female with multiple medical problems has routine laboratory work performed during a regularly scheduled clinic visit. All of the laboratory values are normal except for a serum calcium level of 11.0 mg/dL (N 8.5-10.2). Which one of the following medications in her current regimen is most likely to cause an elevated calcium level? A) Alendronate (Fosamax) B) Lithium C) Omeprazole (Prilosec) D) Sertraline (Zoloft) E) Spironolactone (Aldactone)

ANSWER: B Hypercalcemia is a commonly encountered laboratory abnormality. It is important for family physicians to be aware of common medications that can cause elevated calcium levels. Of the options listed, lithium is the only medication that can cause high calcium levels. In addition, thiazide diuretics, excluding aldosterone receptor antagonists such as spironolactone, often cause elevated calcium levels. Hypercalcemia is not a side effect of alendronate, omeprazole, sertraline, or spironolactone.

Which one of the following types of cardiomyopathy is associated with a systolic murmur on examination that increases in intensity during Valsalva maneuvers? A) Dilated B) Hypertrophic C) Peripartum D) Restrictive E) Takotsubo

ANSWER: B Hypertrophic cardiomyopathy is the most common primary cardiomyopathy and is defined by left ventricular hypertrophy (primarily septal thickening) without cardiac dilation that may cause left ventricular outflow obstruction. It is a potential cause of sudden cardiac death. Many patients are asymptomatic and may potentially only be diagnosed by auscultation of a systolic murmur on examination that increases in intensity during Valsalva maneuvers. The other listed cardiomyopathies are not associated with the classic murmur of hypertrophic cardiomyopathy, but instead are associated with signs of heart failure. Dilated cardiomyopathy is defined by enlargement of the ventricles, systolic dysfunction, and normal left ventricular wall thickness that can result in symptoms of heart failure. Peripartum myopathy is defined by left ventricular dysfunction presenting at the end of pregnancy or in the first few months after delivery. Restrictive cardiomyopathy is defined by increased myocardial stiffness that impairs ventricular filling while normal systolic function is maintained and it may present with signs of right-sided heart failure. Takotsubo cardiomyopathy is defined as the sudden onset of left ventricular dysfunction in response to extreme stress and often presents with symptoms mirroring that of acute coronary syndrome.

A 52-year-old male sees you for a routine follow-up visit for diabetes mellitus. His hemoglobin A1c is 7.6%. He also notes that he has been having increasing pain in his right shoulder over the past few months but he cannot recall any specific injury. The pain is a dull, poorly localized ache that radiates into the biceps and is aggravated when he reaches overhead. On examination you note decreased range of motion in forward flexion, internal and external rotation, and abduction. There is normal strength with resisted activation of the rotator cuff muscles and there are no impingement symptoms. Which one of the following is the most likely diagnosis? A) Osteoarthritis of the shoulder B) Adhesive capsulitis C) SLAP lesion D) Infraspinatus tendinopathy E) Supraspinatus tendinopathy

ANSWER: B Increasing shoulder pain that is hard to localize and decreased range of motion are the hallmark findings for adhesive capsulitis, also known as frozen shoulder. The underlying pathology is contraction of the glenohumeral capsule. It is an idiopathic condition but has an increased prevalence in patients with diabetes mellitus and hypothyroidism. Adhesive capsulitis is often self-limited but can persist for years in some patients. Nonsurgical treatment options include physical therapy, oral or intra-articular corticosteroids, acupuncture, and hydrodilatation.

A 65-year-old female presents to the emergency department with a 24-hour history of abdominal pain, nausea, and vomiting. She reports that her last bowel movement was 2 days ago and she has not passed any flatus. The pain is diffuse and she rates it as 4 on a scale of 10 but says that it reaches a level of 8 at times. She has hypertension, which has been controlled with amlodipine (Norvasc), 5 mg daily. She had an appendectomy at age 25 and had two normal vaginal deliveries. She has been postmenopausal since age 52. An examination reveals a blood pressure of 120/80 mm Hg, a pulse rate of 110 beats/min, a respiratory rate of 16/min and nonlabored, and an oxygen saturation of 95% on room air. Examination of the heart and lungs is normal. On examination of the abdomen you note high-pitched bowel sounds, and the abdomen is distended and tympanic, and diffusely tender with no appreciable masses. A radiograph of the abdomen shows a small bowel obstruction. Which one of the following is the most likely cause of this patient's small bowel obstruction? A) Constipation B) Intestinal adhesions C) A neoplasm D) Ventral herniation E) Volvulus

ANSWER: B Intestinal adhesions are the most common cause of small bowel obstruction, accounting for 60%-75% of cases. Less common causes include neoplasms in 13%-20% of cases, herniation in 2%-15% of cases, and volvulus in <5% of cases. Severe constipation is a rare cause of small bowel obstruction. Further evaluation with CT of the abdomen and pelvis would be indicated to rule out ischemia or perforation and to determine the etiology of this patient's small bowel obstruction. Initial management includes nasogastric tube decompression and intravenous fluid resuscitation. Surgical consultation is recommended. Immediate surgery is indicated for unstable patients, a closed loop obstruction such as volvulus that cannot be reduced, intestinal ischemia, or perforation. Most cases resolve with conservative management, but surgery is recommended if the obstruction has not resolved after 3-5 days.

A 34-year-old female at 32 weeks gestation presents with a right-sided, pounding headache that began 8 hours ago and is similar to headaches she has had in the past. She is sensitive to light and sound, and has vomited several times since the onset of pain. She has taken acetaminophen without relief. She takes prenatal vitamins but no other routine medications. On examination her blood pressure is normal. Which one of the following would be the most appropriate treatment for this patient? A) Dihydroergotamine B) Metoclopramide (Reglan) C) Naproxen D) Oxycodone (OxyContin) E) Sumatriptan (Imitrex)

ANSWER: B Metoclopramide and acetaminophen are the only two medications considered safe for abortive migraine treatment during pregnancy (SOR B). The dopamine antagonist antiemetics are considered second-line abortive treatments in the general population. Dihydroergotamine should not be used during pregnancy due to its oxytocic properties and the potential risk of intrauterine growth restriction with its use. NSAIDs are not considered safe during pregnancy, particularly in the first and third trimesters. Opioids are only moderately useful for migraine treatment and should be avoided during pregnancy due to their abuse potential. Triptans are generally considered safe during the first trimester but not in the second and third trimesters. Their use has been associated with uterine atony, increased risk of bleeding during delivery, and increased risk of preterm birth.

A healthy 30-month-old male is brought to your office because of right elbow pain that began acutely today when his father grabbed his wrist as he fell off a stool at home. There are no other symptoms. An examination is notable for the patient's resistance to move his right arm, which he holds partially flexed and pronated at the elbow. The affected arm is otherwise normal. Which one of the following would you recommend at this time? A) Splinting and close follow-up B) Attempted reduction of the subluxed radial head now C) A radiograph of the right elbow D) Referral to physical therapy E) Referral to an orthopedic surgeon

ANSWER: B Nursemaid's elbow is a common injury in young children, often occurring with sudden upward traction on the arm that subluxes the radial head. Patients typically report acute elbow pain. Many children hold the elbow partially flexed and pronated and decline to move the elbow. A clinical diagnosis is made when the history and examination are typical for the condition, as is the case here. Manual reduction in the office is the initial treatment and succeeds more than 70% of the time. Splinting is not a first-line treatment. Imaging and referral are not indicated before attempting reduction.

The initial treatment of choice for head lice is A) lindane 1% shampoo B) permethrin 1% shampoo (Nix) C) pyrethrum 0.3%/piperonyl butoxide 4% shampoo (Rid) D) ivermectin 0.5% lotion (Sklice) E) malathion 0.5% lotion (Ovide)

ANSWER: B Permethrin 1% shampoo is recommended as first-line treatment for head lice. Lindane should not be used because of neurotoxicity. Pyrethrum 0.3%/piperonyl butoxide 4% shampoo, ivermectin 0.5% lotion, and malathion 0.5% lotion are alternative treatments but should not be used unless two treatments with permethrin are unsuccessful.

A patient born in 1954 requests screening for hepatitis C. His anti-HCV antibody screen is positive. Which one of the following would be most appropriate at this point? A) A confirmatory second anti-HCV test in 4-6 weeks B) A qualitative HCV RNA test C) Quantitative HCV genotyping D) Treatment with a dual antiviral regimen pending additional testing E) Referral for a percutaneous liver biopsy

ANSWER: B The CDC recommends that all adults 18 years of age or older receive a one-time screening for hepatitis C virus (HCV). Persons with risk factors for HCV exposure should be screened periodically, based on risk level. If the anti-HCV antibody screen is positive, then a qualitative HCV RNA test is the next step (SOR C). Prior to initiating treatment, a quantitative HCV RNA and genotype testing is necessary (SOR A). In addition, assessing the degree of fibrosis will provide information regarding the urgency of treatment. Percutaneous liver biopsy is generally the preferred evaluation after obtaining quantitative and genotype results, all of which can guide treatment decisions.

According to the Ottawa ankle and foot rules, which one of the following patients with ankle or foot pain after a twisting injury requires radiographs to rule out a fracture? A) An 18-year-old male basketball player who landed on an opposing player's foot while rebounding the ball and has tenderness anterior to the lateral malleolus B) A 23-year-old female who twisted her ankle while playing soccer and has tenderness at the base of the fifth metatarsal C) A 30-year-old male who twisted his ankle a week ago after stepping on his child's toy and has persistent swelling with tenderness anterior to the medial malleolus D) A 48-year-old female who slipped while going down stairs and has tenderness and bruising inferior to the lateral malleolus

ANSWER: B The Ottawa ankle and foot rules were designed to determine the need for radiographic films in ankle and foot injuries. The instrument has a sensitivity of nearly 100% and a specificity of 30%-40%. According to these rules, ankle radiographs are indicated if there is any pain in the malleolar zone, plus one or more of the following: bony tenderness over the distal 6 cm of the posterior lateral or medial malleolus (the areas of potential fracture) or the inability to bear weight for four steps immediately after the injury and at the time of the examination. Foot radiographs are indicated if there is any pain in the mid-foot zone, plus one or more of the following: bony tenderness at the base of the fifth metatarsal or the navicular bone or the inability to bear weight for four steps immediately after the injury and at the time of the examination.

A 32-year-old white female presents to your office for a health maintenance visit required by her employer. The patient lives with her husband and two children. Her hobbies include water skiing and traveling. She does not have any significant past medical history. Her last Papanicolaou (Pap) smear was normal 1 year ago, and all of her previous Pap smears have been normal. A review of systems and a physical examination are unremarkable. Based on U.S. Preventive Services Task Force recommendations, there is good evidence to screen this patient for A) illicit drug use B) intimate partner violence C) skin cancer D) vitamin D deficiency

ANSWER: B The U.S. Preventive Services Task Force recommends screening for intimate partner violence for women of reproductive age, with referral to support services for those with a positive screen (B recommendation). In asymptomatic adults the current evidence is insufficient to recommend screening for illicit drug use, skin cancer, or vitamin D deficiency.

One of your patients will turn 65 in 2 weeks and your practice manager routinely encourages scheduling a Welcome to Medicare preventive visit soon after patients' 65th birthdays. This patient continues to work full time and is currently insured through his employer-sponsored health insurance. In order to bill for a Welcome to Medicare visit after this patient turns 65, which one of the following is true? A) The patient must be enrolled in Medicare Part A B) The patient must be enrolled in Medicare Part B C) The patient must be enrolled in Medicare Part D D) The patient must be over age 65 and the specific type of Medicare enrollment is not relevant

ANSWER: B The Welcome to Medicare preventive visit, also known as an Initial Preventive Physical Examination (IPPE), is a one-time service that can be provided within the first year of a patient's enrollment in Medicare Part B. Medicare Part B covers provider visits and outpatient services such as laboratory testing. Beneficiaries are automatically enrolled in Part A when they apply to Medicare, which provides coverage for hospital-based and hospice care. Because Part A does not typically carry a monthly premium, some working older adults who continue to have insurance through their employer may opt to obtain Part A only, and wait on Part B coverage, which does have a monthly premium. Medicare Part D is prescription drug coverage.

A 57-year-old male with a history of heart failure sees you for follow-up. He describes symptoms of mild dyspnea on exertion with ordinary activities such as shopping or yard work. An echocardiogram shows an ejection fraction of 37%. According to the New York Heart Association criteria, this patient's heart failure would be classified as which one of the following? A) Class I B) Class II C) Class III D) Class IV

ANSWER: B The appropriate classification of heart failure is important for monitoring the disease. The most common currently used system is the New York Heart Association (NYHA) functional classification. In this system, class I is defined as heart disease in a patient with no symptoms and no limitations of physical activity. Patients with class II heart failure have mild symptoms with normal physical activity. Class III heart failure refers to significant limitations of activity, including symptoms with less than normal activities. Patients with class IV heart failure have symptoms at rest and are unable to carry on activity without discomfort.

Which one of the following patients should undergo screening for vitamin D deficiency? A) A 27-year-old male with recurrent major depressive disorder B) A 58-year-old male with a glomerular filtration rate of 28 mL/min/1.73 m2 C) A 69-year-old female with chronic osteoarthritis pain D) A 75-year-old female with recurrent falls E) An 88-year-old male with severe fatigue

ANSWER: B The measurement of vitamin D levels is recommended only for patients with decreased kidney function, various skeletal diseases, or hypercalcemia (SOR C). Vitamin D deficiency is common in patients with chronic kidney disease, and it is associated with cardiovascular morbidity and mortality in those patients. However, good-quality evidence is lacking regarding the effect of vitamin D supplementation in these patients. There is no recommendation for screening the general population for vitamin D deficiency (SOR B), nor should routine vitamin D supplementation be recommended for community-dwelling adults (SOR A). Vitamin D supplementation has not been found to improve depression, osteoarthritis, chronic pain, or fatigue (SOR A). Meta-analyses have found no relationship between vitamin D deficiency and falls, fractures, or mortality.

A 65-year-old female comes to your office for her Welcome to Medicare visit. Her current medication regimen includes losartan (Cozaar), 50 mg daily, for hypertension; oxybutynin, 5 mg daily, for overactive bladder; doxepin, 5 mg at night, for depression and insomnia; and atorvastatin (Lipitor), 10 mg daily, for hyperlipidemia. She is doing well on her current medications and is requesting refills today. You advise her that the benefits of her current regimen for her overactive bladder and depression need to be weighed against her increased risk of A) cirrhosis B) dementia C) Parkinson's disease D) peripheral vascular disease E) pulmonary fibrosis

ANSWER: B The use of anticholinergic medications and other sedatives has been prospectively linked to an increased risk of dementia. Treatment of overactive bladder and depression could include anticholinergics, which can be inadvertently combined with over-the-counter medications such as chlorpheniramine. Alternative medications should be considered for older patients when possible. There is no association between the use of anticholinergic agents and cirrhosis, Parkinson's disease, peripheral vascular disease, or pulmonary fibrosis.

A 76-year-old male sees you for a routine health maintenance examination. He says that he has developed a mild tremor over the past few years, and it has now become bothersome to him because it affects his handwriting and his ability to eat in public. His mother had a similar tremor. He has no tremor at rest. When you ask him to hold out his hands, you note a tremor in both hands and wrists. His voice is also somewhat tremulous. He asks if there are any treatments to reduce the tremor. Which one of the following would be the most appropriate next step? A) Observation only B) A trial of a -blocker C) A trial of a dopaminergic agent D) Serum ceruloplasmin and 24-hour urinary copper excretion levels E) MRI of the brain

ANSWER: B This patient presents with symptoms of a benign essential tremor. This is a postural symmetric tremor that is most often noted in the hands and wrists. A positive family history is often present. The diagnosis can be made clinically and a trial of -blockers is warranted, especially since this patient is having significant interference in daily and social activities. A resting tremor would suggest parkinsonism, which warrants treatment with a dopaminergic agent. Young patients presenting with a tremor should be evaluated for Wilson's disease, with ceruloplasmin and urinary copper excretion testing. MRI of the brain would be warranted for a cerebellar tremor, which would manifest as a postural intention tremor.

A 36-year-old male sees you for follow-up of progressive fatigue and lightheadedness that has worsened over the past 3 months. He has lost 5 kg (11 lb) during this time. On examination he has a BMI of 21 kg/m2, a blood pressure of 88/48 mm Hg, and a pulse rate of 66 beats/min. A skin examination is notable for patches of nonpigmented skin on the hands. Initial laboratory testing is significant for a sodium level of 132 mEq/L (N 135-145) and a potassium level of 5.3 mEq/L (N 3.5-5.0). Which one of the following tests would confirm the most likely diagnosis? A) 17-hydroxyprogesterone B) ACTH stimulation C) Dexamethasone suppression D) Late night salivary cortisol E) Plasma renin and aldosterone

ANSWER: B This clinical case is consistent with Addison's disease, or adrenal insufficiency. This case is most likely the result of autoimmune disease, given the concurrent vitiligo, but it may also be idiopathic or secondary to cancers such as lymphoma or infections such as tuberculosis. Adrenal insufficiency is suggested by a low morning cortisol level, but the test of choice to confirm this diagnosis is the ACTH stimulation test. 17-Hydroxyprogesterone deficiency causes congenital adrenal hyperplasia, which typically presents in childhood. This test is part of newborn screening in the United States. Acquired 17-hydroxyprogesterone deficiency can present in adulthood as adrenal insufficiency but a low 17-hydroxyprogesterone level does not confirm adrenal insufficiency. Late night salivary cortisol is an initial test for corticosteroid excess (Cushing syndrome) and the dexamethasone suppression test is used to confirm that disorder. Renin and aldosterone levels can be helpful to characterize mineralocorticoid deficiency but they are not diagnostic.

A 32-year-old female comes to your office because of palpitations. She reports a sensation of her heart racing that lasts for several seconds, occurs at rest, and has been occurring daily for the past couple of weeks. She has not had any loss of consciousness or other associated symptoms, has no history of recent stressors or anxiety, and does not drink caffeine or take any illicit drugs. She is otherwise healthy and takes no medications. An examination is unremarkable and an EKG and basic laboratory studies are all normal. Which one of the following would be the most appropriate next step in your evaluation? A) Reassurance only B) A 24-hour Holter monitor C) A 30-day cardiac event monitor D) An exercise stress test E) Transthoracic echocardiography

ANSWER: B This patient does not have any obvious metabolic or psychiatric explanations for her palpitations, raising the clinical suspicion for an underlying cardiac cause. Reassurance without further investigation of a possible cardiac cause would be inappropriate. Given that her symptoms are occurring daily, a 24-hour Holter monitor would be the most appropriate next step in the evaluation. If her symptoms occurred less frequently, a 30-day cardiac event monitor, which stores data after being activated, could be used. An exercise stress test would be indicated as the next step if her symptoms were exertional. Transthoracic echocardiography would be indicated as the next step if there were a clinical suspicion for structural heart disease based on a previous history of cardiac disease or worrisome signs and symptoms such as dyspnea, crackles, lower extremity edema, or elevated jugular venous pressure.

You are working in an urgent care clinic when a 68-year-old male with chronic hypertension sees you for refills of his medications. He has been out of his medication for the past month and could not get an appointment with his primary care physician for refills. His blood pressure is persistently 190/115 mm Hg, even after he has rested in a quiet room for 30 minutes. His blood pressure previously had been well controlled. He has a moderate headache but otherwise feels well. An examination, including a funduscopic examination, is normal. Which one of the following management options would be most appropriate at this time? A) Refill his usual medications and arrange for follow-up in 1 week B) Administer oral labetalol (Trandate) every 30 minutes until his blood pressure is <180/110 mm Hg C) Administer oral nifedipine (Procardia) every 30 minutes until his blood pressure is <180/110 mm Hg D) Administer sublingual nifedipine every 30 minutes until his blood pressure is <180/110 mm Hg E) Refer for immediate hospitalization for intravenous antihypertensive treatment

ANSWER: B This patient has a hypertensive urgency, defined as symptomatic acute severe hypertension without evidence of acute end-organ injury. Hypertensive urgencies may be managed in the ambulatory setting. Emergent intravenous treatment at the hospital is not indicated. This patient should be treated with an oral agent with a fairly rapid onset of action, such as clonidine, labetalol, captopril, or prazosin. Topical nitroglycerin is also an option. Nifedipine may cause unpredictable blood pressure reduction and should be avoided. The patient may be discharged to resume his usual medications after his symptoms have improved and his blood pressure is below 160-180/110 mm Hg, with follow-up within a week.

A 28-year-old gravida 2 para 1 at 28 weeks gestation sees you because her 2-year-old son was diagnosed with influenza A earlier that day. She has a history of asthma. Her son had received the influenza vaccine this influenza season but she had declined due to concerns about adverse effects on her pregnancy. In addition to influenza vaccine now, which one of the following is recommended for postexposure chemoprophylaxis of influenza in this patient? A) Baloxavir marboxil (Xofluza) B) Oseltamivir (Tamiflu) C) Peramivir (Rapivab) D) Rimantadine (Flumadine) E) Zanamivir (Relenza)

ANSWER: B This patient has a young child who has been diagnosed with influenza, and she has two factors that place her in the high-risk category for complications from influenza: pregnancy and asthma. Pregnancy is an indication for influenza vaccine, not a reason to avoid it. In addition to influenza vaccine now, she should begin a 2-week course of chemoprophylaxis to allow time for the vaccine to become effective. The safety record for oseltamivir in pregnancy is good, and it is the preferred choice in this situation. Baloxavir marboxil has not been approved for chemoprophylaxis and should be avoided in pregnancy. Peramivir is an intravenous formulation that is used in hospitalized patients with severe influenza. Amantadine and rimantadine are not active against influenza B, and most influenza A strains are resistant. Zanamivir should be avoided in patients with asthma.

previously healthy 18-year-old female presents with finger pain. About 5 days ago she started to have mild burning of her left distal index finger. Two days later she developed worsening pain and redness of her fingertip. She does not remember injuring her finger or having a similar problem previously. On examination you note erythema of the medial palmar tip of her affected finger, with several vesicles that have opaque fluid in them. The distal digital pulp is soft but tender. Which one of the following is the most appropriate treatment for this condition? A) Warm water soaks B) Pain control and dressings C) Antibiotics D) Antifungals E) Incision and drainage

ANSWER: B This patient has herpetic whitlow, which is a viral infection of the distal finger caused by herpes simplex. Primary herpetic whitlow is generally a self-limited infection. The recommended treatment is pain management and keeping it covered with a dressing to prevent transmission. Warm water soaks are useful to manage superficial hand infections but are not indicated to treat herpetic whitlow. Herpetic whitlow is a viral infection, so antibiotics and antifungals would not be beneficial, although antibiotics would be appropriate if a secondary bacterial infection is suspected or if an abscess is confirmed by ultrasonography. Off-label use of antiviral medications should be considered only for patients with recurrent lesions, those with symptoms for less than 48 hours, and those who are immunocompromised. Incision and drainage should not be performed because it increases the risk of bacterial superinfection.

A 72-year-old female presents with bothersome palpitations. She is otherwise healthy and is not taking any medications. A physical examination is normal, including thyroid and eye examinations. Laboratory studies reveal a serum TSH level of 0.2 U/mL (N 0.4-4.0) and normal T3 and free T4 levels. An EKG reveals frequent premature atrial contractions but is otherwise normal. Ultrasonography of the thyroid does not reveal any nodules, thyroid scintigraphy shows diffuse uptake, and an anti-thyrotropin-receptor (thyroid-stimulating immunoglobulin) antibody level is significantly elevated. Which one of the following is the most likely diagnosis? A) Central hypothyroidism B) Graves disease C) Iodine deficiency D) Solitary toxic thyroid nodule E) Toxic multinodular goiter

ANSWER: B This patient has subclinical hyperthyroidism caused by Graves disease. A positive anti-thyrotropin-receptor (thyroid-stimulating immunoglobulin) antibody result is virtually diagnostic of Graves disease. Central hypothyroidism is associated with a low TSH level and low T3 and T4 levels. Iodine deficiency is associated with goiter and hypothyroidism. Nodular thyroid disease is unlikely given the imaging results. Treatment of this patient's mild Graves disease is probably indicated, given her age and cardiac symptoms.

A 35-year-old female presents with fatigue. She follows a strict vegan diet and does not take any medications. There is no history of any abnormal bleeding. Although she has had fatigue for 2 years, she now notices that she has a craving for ice. Results of a laboratory evaluation are shown below. WBCs 5100/mm3 (N 3400-10,800) Basophils 0.0 × 103/ L (N 0.0-0.2) Immature granulocytes 0% Immature granulocyte count 0.0 × 103/ L (N 0.0-0.1) Platelets 271,000/mm3 (N 150,000-379,000) RBCs 4.75 million/mm3 (N 3.77-5.28) Reticulocytes 1.2% (N 0.6-2.6) Hemoglobin 9.2 g/dL (N 11.1-15.9) Hematocrit 32.3% (N 34.0-46.6) Mean corpuscular volume 68 m3 (N 79-97) Mean corpuscular hemoglobin 19.4 pg/cell (N 26.6-33.0) Mean corpuscular hemoglobin concentration 28.5 g/dL (N 31.5-35.7) RDW 18.1% (N 12.3-15.4) Total iron binding capacity 519 g/dL (N 250-450) Unsaturated iron binding capacity 482 g/dL (N 131-425) Iron 37 g/dL (N 27-159) Iron saturation 7% (N 15-55) Transferrin 405 mg/dL (N 200-370) Ferritin 4 ng/mL (N 15-150) Vitamin B12 353 pg/mL (N 160-950) Given these results, which one of the following would be the next step in the management of this patient? A) Recheck her hemoglobin level in 3 months B) Start ferrous sulfate, 325 mg orally C) Order an intravenous iron infusion D) Order a blood transfusion E) Refer for colonoscopy

ANSWER: B This patient has symptoms and laboratory findings consistent with iron deficiency. Because she is premenopausal, excessive menstrual bleeding is a possible etiology, as is insufficient iron intake, given her strict vegan diet. The best first step is to replace her iron stores with an oral replacement. Although a hemoglobin level should be rechecked in 3 months, this should happen after the initiation of iron therapy. Intravenous iron infusion is reserved for patients who cannot tolerate oral therapy. This patient does not require a blood transfusion at this time. If she does not respond to iron therapy, and common causes such as dietary restriction and menstrual bleeding are excluded, then she should be referred for colonoscopy to search for a gastrointestinal cause of blood loss.

A 58-year-old male with uncontrolled type 2 diabetes sees you for follow-up after a recent hospitalization for urosepsis treatment with intravenous antibiotic therapy. His hospital course was complicated by Clostridioides (Clostridium) difficile colitis, and he completed oral vancomycin (Vancocin) therapy 2 weeks ago. He was nearly back to his baseline but has had recurrent watery diarrhea for the past 3 days. You confirm a recurrent infection. Which one of the following would be the most appropriate treatment? A) Probiotics B) Fidaxomicin (Dificid), 200 mg twice daily for 10 days C) Metronidazole (Flagyl), 500 mg three times daily for 10 days D) Vancomycin, 125 mg four times daily for 10 days E) Fecal microbiota transplantation

ANSWER: B This patient presents with his first recurrence of Clostridioides (Clostridium) difficile infection, which was previously treated with vancomycin. Initial episodes can be treated with vancomycin (strong recommendation, high quality of evidence), fidaxomicin (strong recommendation, high quality of evidence), or metronidazole if the other two treatments are unavailable (weak recommendation, high quality of evidence). However, fidaxomicin is recommended for recurrent infection if vancomycin was prescribed for the initial episode (weak recommendation, moderate quality of evidence). If available, a prolonged tapered course of vancomycin could be used if a 10-day course was prescribed initially (weak recommendation, low quality of evidence). Vancomycin is only recommended for a first recurrent episode if metronidazole was used initially (weak recommendation, low quality of evidence). Metronidazole is not recommended for recurrent episodes. Probiotic administration is not recommended due to insufficient data. Fecal microbiota transplantation is only recommended for a second or subsequent recurrent infection (strong recommendation, moderate quality of evidence).

A 63-year-old female presents to your office with a sudden onset of lightheadedness and mild nausea that she first noted when getting out of bed this morning. She has had repeated episodes of a spinning sensation when tilting her head up or down. Her symptoms have been so severe that she could not go to work today. She has a history of essential hypertension that is well controlled on hydrochlorothiazide. She has not had any headache, hearing loss, tinnitus, or recent illness or trauma. She has a temperature of 36.8°C (98.2°F), a blood pressure of 136/80 mm Hg, a heart rate of 80 beats/min, a respiratory rate of 12/min, and an oxygen saturation of 96% on room air. You perform the Dix-Hallpike maneuver with her right ear down and in the dependent position and note a latent torsional, upbeating nystagmus. The most appropriate intervention at this time would be A) prolonged upright positioning B) canalith repositioning procedures C) vestibular function testing D) vestibular suppressant medication E) MRI of the brain

ANSWER: B This patient presents with symptoms consistent with right benign paroxysmal positional vertigo (BPPV) and torsional, upbeating nystagmus provoked by the Dix-Hallpike maneuver. Diagnostic criteria include both patient history and physical examination findings. Symptoms suggesting BPPV include an acute onset of brief episodic vertigo triggered by positional changes relative to gravity. In the 2017 clinical practice guidelines on BPPV, the American Academy of Otolaryngology-Head and Neck Surgery Foundation strongly recommended accurate diagnosis of posterior semicircular canal BPPV by performing the Dix-Hallpike maneuver (B recommendation). The guidelines also strongly recommend treatment with a canalith repositioning procedure (A recommendation). Positional restrictions, particularly postprocedural restrictions, are not recommended due to insufficient evidence (A recommendation). Vestibular testing should also be avoided in patients who meet diagnostic criteria for BPPV and who do not have other vestibular symptoms such as hearing loss or tinnitus (C recommendation). Vestibular suppressant medication such as antihistamines and benzodiazepines should be avoided due to potential risks and lack of evidence to suggest that these medications are as effective as repositioning procedures (B recommendation). Radiographic imaging such as MRI of the brain or CT of the head should not be obtained in patients who meet diagnostic criteria for BPPV and who do not have other signs or symptoms of neurologic pathology (C recommendation).

A 56-year-old male with a long-standing history of severe COPD presents with a 1-week history of progressive dyspnea. He says that yesterday it worsened to the point where he could not speak in full sentences. On examination he appears pale and diaphoretic. He has a blood pressure of 100/58 mm Hg, a pulse rate of 122 beats/min, a respiratory rate of 28/min, and an oxygen saturation of 82% on room air. He has diminished air entry bilaterally. A chest radiograph is shows a large right pneumothorax. Which one of the following is LEAST likely to relieve this patient's dyspnea? A) Supplemental 100% oxygen B) Nebulized albuterol and ipratropium (Atrovent) C) Needle thoracentesis D) Chest tube placement

ANSWER: B This patient's radiograph shows a large right pneumothorax. Initial management includes administration of 100% humidified oxygen. Definitive therapy includes reducing the pleural dead space and thereby improving oxygenation, using either needle thoracentesis or chest tube placement. When the pneumothorax is small (<15% of the hemithorax) or when the patient is asymptomatic, supplemental oxygen and observation are usually adequate. Supplemental oxygen not only helps with the hypoxia, it also produces up to a fourfold increase in pleural reabsorption of the air in the pneumothorax. Nebulized albuterol and ipratropium play a role in managing dyspnea in acute exacerbations of COPD and are not likely to relieve the dyspnea from a pneumothorax (SOR B).

A 41-year-old Hispanic male presents to your office with a cough. He does not speak English and his 16-year-old daughter offers to translate for him. Which one of the following is most appropriate regarding medical interpreters for patients who do not speak English? A) If a trained medical interpreter is not on staff, using a family member is an acceptable alternative B) A trained medical interpreter should be offered to decrease errors, improve clinical outcomes, and improve satisfaction in care C) When using an interpreter, the physician should address the interpreter directly to avoid confusion and promote better communication D) Using a family member for interpretation meets the legal requirement of Title VI of the Civil Rights Act for patients with limited English

ANSWER: B Title VI of the Civil Rights Act requires a trained interpreter to be offered for patients with limited English proficiency (SOR C). Failure to offer this service is considered discriminatory and illegal. Relying on family, friends, or bilingual staff increases medication errors, unnecessary testing, and the risk of malpractice, and decreases compliance with recommendations. When using a medical interpreter, the physician should address the patient directly to avoid confusion (SOR C).

A 55-year-old female presents to your office because she has intermittent locking of her right ring finger when it is flexed. It is painful and she often has to use her other hand to extend the finger. Her work involves repetitive movement of her hands and she requests a treatment option that will involve as little missed time from work as possible. Which one of the following would be the most cost-effective option? A) NSAIDs B) A corticosteroid injection C) Splinting of the distal interphalangeal joint D) Physical therapy E) Surgical correction

ANSWER: B Trigger finger is a common reason for referral to a hand surgeon. Risks factors for this condition include trauma, overuse, diabetes mellitus, and carpal tunnel syndrome. It is much more common in women than in men and the average age of onset is 58. Trigger finger develops when there is scarring and inflammation of the A1 pulley, the first of a five-pulley system in the hand. Stenosis of the A1 canal or nodules on the tendon can produce locking, cracking, and pain when the digit is flexed. The most cost-effective treatment strategy is the use of corticosteroid injections. The success rate is 57% after the initial injection and 86% following the second injection within a 6-month time frame. When the problem is mild, NSAIDs and splinting may be effective. Physical therapy and surgical correction are not indicated for this patient.

The inability to use and make sense of numbers is a common problem encountered in physician-patient communication that can make it difficult to achieve shared decision-making. Which one of the following methods has been shown to be a helpful strategy when discussing numbers? A) Using relative risk instead of absolute risk B) Using icon arrays (pictographs) to show ratios C) Using percentages instead of frequencies D) Framing outcomes in either positive or negative terms, but not both

ANSWER: B Use of simple graphical representations and other visual aids can greatly enhance a patient's comprehension of numbers. One useful tool is expressing ratios as an icon array in which a shape is repeated a specific number of times to represent the denominator and some of the shapes are shaded in to represent the numerator. Other techniques include using absolute risk instead of relative risk, using frequencies instead of percentages, and framing outcomes in both positive and negative terms.

A healthy 26-year-old female would like to start using regular contraception and asks about her options. Her menses are normal and her last menstrual period was 10 days ago. If initiated today, which one of the following contraceptive methods does NOT require the use of backup contraception? A) Oral contraceptives B) An etonogestrel subdermal implant (Nexplanon) C) A copper-containing IUD (ParaGard) D) A levonorgestrel IUD (Mirena)

ANSWER: C A copper-containing IUD may be placed at any time during the menstrual cycle and does not require the use of backup contraception. If oral contraceptives are started the day of the appointment, then backup contraception is needed for the first week. If oral contraceptives will be started with the next menstrual cycle, then backup contraception is needed until the pills are started. Backup contraception is needed if a hormonal implant is placed more than 5 days from the start of the last menstrual period, and if a levonorgestrel IUD is placed more than 7 days from the start of the last menstrual period.

For patients with atrial fibrillation, which one of the following comorbid conditions represents the strongest indication for thromboprophylaxis with warfarin (Coumadin), rather than a direct oral anticoagulant? A) A CHA2DS2-VASc score 3 B) End-stage chronic kidney disease C) A mechanical heart valve D) Mild mitral stenosis E) Severe mitral regurgitation

ANSWER: C According to 2019 guidelines from the American College of Cardiology, American Heart Association, and Heart Rhythm Society, patients with nonvalvular atrial fibrillation and an elevated CHA2DS2-VASc score (>2 in men and >3 in women) should receive anticoagulation, preferably with a direct-acting oral anticoagulant (DOAC), rather than warfarin (level of evidence A). Recent evidence has shown that DOAC options are not inferior to, and in some studies are superior to, warfarin for preventing strokes and systemic embolic events, with a lower risk of serious bleeding. However, warfarin is still recommended over a DOAC for valvular atrial fibrillation that occurs in the presence of moderate to severe mitral stenosis or a mechanical heart valve. For atrial fibrillation in patients with other forms of valvular heart disease, including mitral regurgitation and mild mitral stenosis, DOAC therapy is preferred over warfarin. For patients with atrial fibrillation and end-stage chronic kidney disease, both apixaban (a direct factor Xa inhibitor) and warfarin are comparable options.

A 45-year-old female sees you for the first time for a health maintenance examination. She has a 20-pack-year history of smoking. She has been sexually active with women only, has been in a monogamous relationship for 15 years, and has never been pregnant. A Papanicolaou smear 2 years ago was normal. She was tested 15 years ago for syphilis and hepatitis B with negative results. She has not had any other screening tests for sexually transmitted infections. Which one of the following screenings would be appropriate for this patient? A) Bacterial vaginosis B) Chlamydia trachomatis and gonorrhea C) HIV D) HPV E) Syphilis

ANSWER: C According to the CDC, all individuals between 13 and 64 years of age should be tested at least once for HIV. Although bacterial vaginosis is more common in women who have sex with women (WSW), screening is not recommended by the CDC. Screening for Chlamydia trachomatis and gonorrhea is appropriate for women younger than 25 years of age and for women at risk, including those with new partners, multiple partners, or a new partner with a sexually transmitted infection. This patient is >25 years of age and does not have any additional risk factors. The U.S. Preventive Services Task Force recommends screening for cervical cancer with cytology every 3 years or cytology and HPV co-testing every 5 years for women 30-65 years of age. This patient's Papanicolaou test was normal 2 years ago. Annual syphilis screening is appropriate for men who have sex with men but is not indicated for WSW.

A 67-year-old female patient began taking a bisphosphonate for treatment of osteoporosis 2 years ago after a DXA scan revealed a T-score of -2.7. Her FRAX score showed a 10-year probability of hip fracture of 5%. You order a repeat DXA scan and her T-score is now -2.3 and her FRAX score is 3.5%. Which one of the following should you recommend to this patient regarding the duration of treatment with a bisphosphonate? A) Stop taking it now B) Continue taking it for 1 more year C) Continue taking it for 3 more years D) Continue taking it for 7 more years E) Continue taking it indefinitely

ANSWER: C According to the National Osteoporosis Foundation, treatment is indicated for patients at high risk of fracture, including those with osteoporosis, defined as a T-score of -2.5 or less, or osteopenia, defined as a T-score of -1 to -2.5 and a 10-year probability of hip fracture of at least 3% using the FRAX tool. Bisphosphonates are considered first-line pharmacologic therapy. Treatment beyond 5 years in women who do not have a persistent T-score of -2.5 or less has not been shown to result in further decreases in rates of clinical vertebral fractures, nonvertebral fractures, or mortality (SOR C). In addition, there is increasing evidence that the risk of atypical fracture of the femur increases with the use of bisphosphonates beyond 5 years. Inappropriate prescribing of drugs that are not discontinued after their usual effective or recommended period is known as "legacy prescribing" and can contribute to inappropriate polypharmacy.

A 46-year-old male who injects heroin daily presents with a 6-month history of progressive dyspnea on exertion, a productive cough, and fatigue. He does not have any fever, chills, malaise, or hemoptysis. He has not had any sick contacts and has never smoked. On physical examination he shows no signs of distress, has a normal oxygen saturation on room air, and has normal breath sounds. A chest radiograph reveals bilateral perihilar shadowing. A subsequent lung biopsy will most likely show A) adenocarcinoma B) branching hyphae C) foreign body granulomas D) caseating granulomas E) noncaseating granulomas

ANSWER: C Although persons who inject drugs are at high risk for a variety of pulmonary infectious diseases, this patient's presentation, including the relatively slow development of symptoms, is most consistent with pulmonary foreign body granulomas. These result from the injection of crushed pills, talc, or other foreign substances, which are then deposited in the vasculature of the lungs. Adenocarcinoma is not as likely given the patient's age and nonsmoking history. Branching hyphae would be seen in aspergillosis but this patient does not have fevers or malaise. Caseating granulomas are seen in tuberculosis, which is less likely given the absence of fever and hemoptysis. Noncaseating granulomas, seen in sarcoidosis, would not be more likely in this patient than in the general population.

A 36-year-old female presents to your office with a 24-hour history of redness in her right eye. It is associated with mild pain but no drainage. On examination her visual acuity is 20/20 bilaterally, her pupillary reflex is normal, extraocular movements are intact, and there is no discharge noted. There is a focal area of hyperemia of the episcleral blood vessels noted along the medial aspect of the eye. Fluorescein staining is normal. This patient's presentation is most consistent with which one of the following? A) Bacterial conjunctivitis B) Viral conjunctivitis C) Episcleritis D) Iritis E) Keratitis

ANSWER: C An acute red eye is a common presentation in primary care and it is critical to differentiate serious causes from benign causes. Episcleritis is a self-limited condition that can be idiopathic and presents with mild discomfort and focal hyperemia. Conjunctivitis is typically associated with a discharge that is clear in viral cases and mucopurulent in bacterial cases. Iritis is associated with significant pain, a poorly reactive pupil, diminished vision, and photophobia. This patient does not have changes in visual acuity, photophobia, or severe pain as seen in keratitis, which would also cause an abnormal fluorescein stain showing corneal ulceration.

You are examining a 65-year-old male from Central America with a history of rheumatic valvular disease. Which one of the following is the principal auscultatory finding of aortic regurgitation? A) An S3 gallop heard best at the cardiac apex with the patient supine B) A triphasic pericardial friction rub with the patient seated and leaning forward C) A low-pitched decrescendo diastolic murmur that is loudest at the lower left sternal border with the patient seated and leaning forward D) A high-pitched crescendo/decrescendo midsystolic murmur that is loudest at the right upper sternal border and radiates to the carotid arteries with the patient sitting upright E) A harsh holosystolic murmur that is loudest at the lower left sternal border and radiates to the left lateral chest wall with the patient in the left lateral decubitus position

ANSWER: C Aortic regurgitation in an older adult may be due to a congenital bicuspid aortic valve, which often is accompanied by aortic stenosis. Rheumatic aortic valvular disease may also cause aortic regurgitation, which is the most common cause in the developing world but less common in the United States. The hallmark murmur of aortic regurgitation in either case is a "blowing" decrescendo diastolic murmur along the tract from the aortic valve (upper right sternal border) down to the lower left sternal border, where it is loudest. It is best heard with the patient sitting, leaning forward, and holding his or her breath in expiration. Both bicuspid aortic valve and rheumatic valve disease may also be associated with aortic stenosis. The typical murmur of aortic stenosis is a mid- to long crescendo/decrescendo systolic murmur, loudest at the right upper sternal border, and often radiating to the carotid arteries. An S3 gallop may also be present in decompensating aortic regurgitation, due to the associated left ventricular dilatation, but this is a secondary finding in later stages. A harsh holosystolic murmur at the lower left sternal border radiating to the axilla is characteristic of mitral valve regurgitation, another potential condition found in rheumatic valvular disease. A pericardial friction rub is the principal auscultatory finding in acute pericarditis.

A 25-year-old female is concerned about skin lesions that have repeatedly appeared and resolved, sometimes lasting for months. She has tried treatment with topical over-the-counter antibiotic ointments with no success. She states that her mother has similar lesions. On examination her vital signs are unremarkable except for a BMI of 32 kg/m2. The examination also reveals multiple deep-seated inflammatory nodules in both axillae that are up to 1.5 cm in size and painful to touch. You also note some malodorous drainage and scars in her axillae that she says are from previous lesions. No other areas are affected. She is afebrile and has no other symptoms. Which one of the following is the most likely diagnosis? A) Acne conglobata B) Cutaneous Crohn's disease C) Hidradenitis suppurativa D) Pilonidal cyst E) Recurrent primary bacterial abscess

ANSWER: C Hidradenitis suppurativa, or acne inversa, is a chronic folliculitis affecting intertriginous areas, causing deep scarring and affecting quality of life. Patients should also be screened for depression. Hidradenitis suppurativa usually occurs between 18 and 39 years of age and affects females and African-Americans more often. Risk factors include family history, smoking, and obesity. Hidradenitis suppurativa is associated with several other comorbidities, including Crohn's disease and diabetes mellitus. Clinical features range from mild inflammatory nodules to widespread abscesses, sinus tracts, and scarring, affecting different parts of the body. Lesions occur most commonly in the axillae, but gluteal folds, the groin, the perianal area, and the perineum can also be affected. The nodules are painful and may vary in size (0.5-2 cm) and last days to months. The abscesses are typically sterile with purulent, malodorous drainage. The Hurley classification system defines the different stages (stage I-III) and treatment is based on the current stage, which ranges from topical treatment with clindamycin to combinations of topical and oral antibiotics and use of biologics. This patient presents with the typical features of hidradenitis suppurativa, including its depth and a chronic and recurrent course. Acne conglobata affects mainly the back, face, chest, and neck in men. Cutaneous Crohn's disease involves perianal lesions with fistulous tracts past the dentate line, which this patient does not have. A pilonidal cyst usually contains hair and skin debris and is typically located near the tailbone. A primary bacterial abscess does not feature accompanying nodules and is superficial rather than deep, and is nonchronic in nature.

An obese 10-year-old female has been diagnosed with hypertension and is starting treatment. Which one of the following should be the target for lowering the systolic and diastolic blood pressure? A) <130/80 mm Hg B) <140/90 mm Hg C) <90th percentile for age, sex, and height D) <95th percentile for age, sex, and height

ANSWER: C Hypertension in children is defined as a blood pressure >95th percentile for age, sex, and height on three separate office visits. The goal for treatment should be to lower the systolic and diastolic blood pressures below the 90th percentile for age, sex, and height. Once children are over 13 years of age, the target should be a blood pressure <130/80 mm Hg.

A 25-year-old male presents with a 4-month history of crampy abdominal pain, diarrhea, and fatigue. His symptoms began gradually but have become more severe and he is now experiencing rectal bleeding. He says that his abdominal pain seems to temporarily improve after eating. He has smoked five cigarettes per day for the past 8 years. He is surprised to learn that he has lost 7 kg (15 lb) when he is weighed today. His vital signs include a blood pressure of 116/70 mm Hg, a heart rate of 76 beats/min, a respiratory rate of 12/min, and a temperature of 37.7°C (99.9°F). A physical examination reveals abdominal tenderness and mild distention. An anorectal examination is significant for a perianal fistula. A laboratory evaluation is notable for mild anemia. His kidney and liver function are normal. Which one of the following is the most likely diagnosis? A) Celiac disease B) Chronic pancreatitis C) Crohn's disease D) Irritable bowel syndrome E) Ulcerative colitis

ANSWER: C Crohn's disease may present insidiously with diarrhea, abdominal pain, rectal bleeding, fever, weight loss, and fatigue. Red-flag symptoms include perianal lesions, a first degree relative with inflammatory bowel disease, weight loss of 5% of the patient's usual weight, abdominal pain for more than 3 months, nocturnal diarrhea, fever, the absence of abdominal pain for 30-45 minutes after eating, and the absence of rectal urgency. This patient exhibits symptoms consistent with Crohn's disease. While anemia is also common in celiac disease, rectal bleeding is not. Chronic pancreatitis does not generally present with improved pain after eating. Irritable bowel syndrome is not associated with fever, rectal bleeding, anemia, or perianal fistulas. Ulcerative colitis is not associated with perianal lesions.

A 13-year-old female is brought to your office by her adoptive mother. They do not know the patient's biological family history. They are concerned because, unlike all of her friends, she has not yet started to menstruate. Breast development began 2 years ago. On examination her breasts show a secondary mound from the areola and nipple above the contour of her breast. She has dark, coarse hair covering the mons pubis consistent with a stage 4 sexual maturity rating. If her sexual development continues to be normal, at what age should you recommend evaluation for primary amenorrhea? A) 13 B) 14 C) 15 D) 16 E) 17

ANSWER: C Delayed puberty in girls is defined as the absence of breast development by age 13. Typically, menarche starts 2.5 years after the onset of breast development, with an average age of 12.5 years (normal range 9-15 years). In girls with otherwise normal sexual development, the absence of menarche by 15 years of age should prompt an evaluation for primary amenorrhea.

A 6-month-old male is brought to your office by his parents for a well child check. He is their first child. On examination you note that he has two erupted teeth. The family lives in a town with fluoridated water and the parents would like to know how to care for their child's teeth. Which one of the following is the most effective preventive measure for this child's newly erupted teeth? A) No care is needed at this point B) A parent wiping or brushing the teeth with water twice daily C) A parent brushing the teeth with a smear of low-fluoride toothpaste twice daily D) A fluoride supplement daily E) Topical sealants

ANSWER: C Early and consistent dental care in infants has been shown to reduce the rate of early childhood caries. The American Academy of Pediatric Dentistry recommends the use of low-fluoride toothpaste for tooth cleaning, starting with newly erupted teeth. According to the CDC, dental caries is one of the most prevalent chronic conditions among children in the United States. Dental caries can start soon after eruption of the first teeth. Brushing an infant's teeth twice daily with a "smear" of fluoridated toothpaste is recommended. Fluoridated toothpaste has been found to be safe and effective for infants' teeth. The use of fluoride in toothpaste is more effective for the prevention of caries than wiping or brushing the teeth with water. A daily fluoride supplement is not appropriate for this patient because the family's local water source already contains fluoride. Topical sealants are intended for molar teeth, not incisors.

A 55-year-old male with a BMI of 32 kg/m2 presents to your office to discuss weight management. He has moderately well controlled type 2 diabetes and hypertension. He prefers not to modify his diet and would like to know if he can expect significant weight loss from exercising. He plans to walk briskly for 45 minutes daily. Which one of the following would be the best advice for this patient? A) Moderate exercise alone is ineffective for weight loss B) Moderate exercise alone is superior to dietary changes for weight loss C) Moderate exercise alone is moderately beneficial for weight loss D) Adding moderate exercise to dietary changes substantially increases weight loss

ANSWER: C Exercise alone does have some substantial benefits, including improved insulin and glycemic control in diabetes, a beneficial effect on blood pressure, a reduction of cardiovascular risks, and a maintenance of weight loss. However, it is only moderately beneficial for promoting weight loss, including when exercise is added to diet changes.

A 14-year-old female is brought to your office as a new patient for a routine well child examination. She has had very little medical care since the pre-kindergarten evaluation. She feels well and does not take any medications. Her past medical, surgical, family, and social histories are unremarkable. A review of systems is notable for no history of menstruation. An examination is notable for a height at the first percentile and a lack of any breast development. Laboratory studies reveal an elevated FSH level. Which one of the following would be the most appropriate next step? A) Follow-up every 3-6 months for assessment of pubertal development B) A corticotropin stimulation test C) Karyotyping D) Radiography of the hand for bone age E) MRI of the brain and pituitary

ANSWER: C Family physicians are often asked to evaluate delays in puberty. Underlying etiologies should be excluded in females >13 years of age who lack any breast development, which may signify delayed puberty. A past medical history and a physical examination, as well as a gonadotropin measurement, should be performed. The incidence of Turner syndrome (TS) is 1/3000 births. Females with TS lack normal X chromosome gene expression and typically have delayed puberty; amenorrhea; elevated FSH, reflecting hypogonadism; and short stature. Delayed diagnosis of TS is common, and short stature and delayed puberty are sometimes the only symptoms. This patient has unexplained short stature, delayed puberty, and an elevated FSH level, so karyotyping to rule out TS is the next step in evaluation. Ongoing surveillance after 13 years is not indicated and may delay therapy. A corticotropin stimulation test would usually be used to rule out Cushing syndrome in a setting of precocious puberty and would not be used in this situation. Radiography of the hand for bone age may support a finding of delayed growth (and thus support treatment with growth hormone for a TS patient), but would not provide valuable diagnostic information in this scenario. An elevated FSH level is consistent with a functioning hypothalamus and pituitary and does not support obtaining MRI of the brain.

While serving as team physician at a football game on a hot afternoon you are asked to evaluate an obese 17-year-old player who has begun limping. The player reports that he is very hot. An examination is normal except for a gastrocnemius spasm. You diagnose heat cramps. As you prepare to start treatment with isotonic fluid, stretching, and massage, the coach asks you how soon the athlete can return to play. You inform the coach that the player can resume participation A) now B) tomorrow C) upon resolution of his symptoms D) when his temperature is <38.3°C (100.9°F) E) after acclimating to the heat for 1 week

ANSWER: C Family physicians serving as team physicians for school-aged athletes are often asked to evaluate heat-related illnesses. It is important to differentiate mild and serious heat-related conditions to help protect athletes. Heat-related muscle cramps are common and self-limited, and players may resume activity upon resolution of symptoms. Measurement of the patient's temperature is not indicated in this scenario. Malaise, significant fatigue, and presyncope are symptoms of heat exhaustion, which does require prolonged removal from play, along with monitoring of the patient's temperature and mental status.

During rounds at the nursing home, you are informed that there are two residents on the unit with laboratory-confirmed influenza. According to CDC guidelines, who should receive chemoprophylaxis for influenza? A) Only symptomatic residents on the same unit B) Only symptomatic residents in the entire facility C) All asymptomatic residents on the same unit D) All residents of the facility regardless of symptoms E) All staff regardless of symptoms

ANSWER: C In long-term care facilities, an influenza outbreak is defined as two laboratory-confirmed cases of influenza within 72 hours in patients on the same unit. The CDC recommends chemoprophylaxis for all asymptomatic residents of the affected unit. Any resident exhibiting symptoms of influenza should be treated for influenza and not given chemoprophylaxis dosing. Chemoprophylaxis is not recommended for residents of other units unless there are two laboratory-confirmed cases in those units. Facility staff of the affected unit can be considered for chemoprophylaxis if they have not been vaccinated or if they had a recent vaccination, but chemoprophylaxis is not recommended for all staff in the entire facility.

A 35-year-old male first presented to your office 4 months ago with a persistent chronic cough. He is a nonsmoker with no significant past medical history. Over the past few months he has been evaluated for GERD, asthma, eosinophilic bronchitis, and upper airway cough syndrome without symptomatic relief or diagnosis. Which one of the following is recommended for chronic refractory cough in this otherwise healthy male? A) Cyclobenzaprine B) Duloxetine (Cymbalta) C) Gabapentin (Neurontin) D) Lorazepam (Ativan) E) Propranolol

ANSWER: C In randomized, controlled trials, gabapentin has demonstrated benefit for treating a refractory chronic cough after 4 weeks of treatment (SOR C). Chronic cough may be due to a hypersensitivity of the cough reflex, either centrally or peripherally. Cyclobenzaprine, duloxetine, lorazepam, and propranolol have not proven to be beneficial in reducing or eliminating chronic cough.

A 20-year-old female with no significant past medical history is brought to your office with a sudden onset of hives, shortness of breath, dizziness, and vomiting. The patient is in anaphylaxis. Intramuscular administration of epinephrine should be given immediately at which one of the following sites? A) Mid-outer aspect of the shoulder B) Mid-outer aspect of the buttocks C) Mid-outer aspect of the thigh D) Mid-outer aspect of the calf E) Lower medial abdominal wall

ANSWER: C In the event of anaphylaxis, epinephrine should be given in the mid-outer aspect of the thigh (anterolateral aspect of the vastus lateralis, mid-muscle belly). Epinephrine is more quickly absorbed and produces higher tissue and plasma levels when injected in the vastus lateralis than in other muscles. It should not be injected subcutaneously.

A 70-year-old female is admitted to the hospital with right upper quadrant pain determined to be secondary to acute cholecystitis. After initial stabilizing treatment the surgeon recommends cholecystectomy but the patient declines. Her family wants her to have the surgery and they tell you they think she has dementia and cannot make this decision. Determining the patient's capacity to make a specific medical decision should include assessment of which one of the following? A) Beliefs, values, reasoning, and communication B) Integrity, competence, fears, and communication C) Understanding, appreciation, reasoning, and communication D) Understanding, personal wishes, intelligence, and communication E) Values, intelligence, expression, and dialogue

ANSWER: C Informed consent for medical care requires that the physician provide patients with accurate information about the risks and benefits of a given treatment course. It also requires that the patient has medical decision-making capacity. In general, medical decision-making capacity is individual to each treatment decision. It is not usually necessary for a physician to determine if a patient is competent to make all decisions, only whether they have the capacity to make the current medical decision. Determining capacity involves assessing a patient's understanding of the benefits, risks, and alternatives to the proposed treatment and their appreciation of those benefits and risks. The physician should also recognize whether the patient showed reasoning in making their decision and that they are able to communicate their decision. Patients' beliefs, values, and personal wishes are certainly important in medical decision-making but simply listing those does not demonstrate capacity. It is not necessary to determine the baseline intelligence level of a patient when determining their capacity for medical decision-making, as the decision itself is assessed as described above, not solely on their underlying intelligence. Assessing patients' fears about a medical decision is valuable but not a direct component of determining capacity.

A healthy 29-year-old female sees you for a health maintenance visit. You recommend influenza vaccine but she declines because she has angioedema with ingestion of any eggs or egg products. She has an epinephrine auto-injector (EpiPen) and last used it approximately 9 months ago when she accidentally ingested a small amount of egg. She has not seen her allergist in more than 3 years but is planning to call soon for an appointment. Based on CDC guidelines, which one of the following would you recommend for this patient regarding influenza vaccination? A) No influenza vaccination regardless of vaccine form B) Vaccination only with inactivated influenza vaccine C) Vaccination with any licensed form of influenza vaccine D) Referral to an allergist for specific allergen testing prior to vaccination

ANSWER: C It is important to provide influenza vaccine to as many individuals as possible. Patients with an egg allergy can receive any form of the influenza vaccine. Patients who have an angioedema reaction to egg products should receive the vaccine in an office setting. Testing for specific responses to egg allergens is not recommended prior to giving the vaccine.

A 48-year-old male presents with a 1-year history of feeling nervous. He feels well otherwise except for mild discomfort from arthritis in both knees. A physical examination is normal, and laboratory studies, including thyroid function, are also normal. You make a diagnosis of generalized anxiety disorder. The patient declines psychotherapy and prefers pharmacologic treatment. Which one of the following is the first-line long-term treatment for this patient? A) Alprazolam (Xanax) B) Buspirone C) Duloxetine (Cymbalta) D) Imipramine (Tofranil) E) Quetiapine (Seroquel)

ANSWER: C Most adults with generalized anxiety disorder (GAD) should be offered drug therapy if nondrug therapies are ineffective or if the patient is not interested in them. SSRIs and SNRIs are recommended as first-line drug therapies because of their tolerability and efficacy compared with other drug therapies. Of the options listed, the SNRI duloxetine would be most appropriate for the treatment of this patient's GAD. Alprazolam may be helpful for short-term treatment of anxiety but not as a long-term treatment. Buspirone, imipramine, and quetiapine are not indicated as initial therapy.

Which one of the following patients should receive antibiotic prophylaxis to prevent infective endocarditis prior to having dental work that will include periodontal manipulation? A) A 6-year-old male with an unrepaired atrial septal defect B) A 32-year-old female with mitral valve prolapse with significant regurgitation C) A 52-year-old male with a past history of a transcatheter aortic valve replacement D) A 60-year-old male with atrial fibrillation due to rheumatic mitral valve disease E) A 70-year-old female with hemodynamically significant aortic stenosis/aortic insufficiency

ANSWER: C Of the patients listed, only the patient with a transcatheter-implanted aortic valve is at increased risk of infective endocarditis (IE) associated with dental procedures. Any patient with a history of valve repair or replacement that involves prosthetic material is at increased risk for IE, but even those with significant valvular disease do not benefit from prophylaxis. Certain patients with congenital heart disease should also receive prophylaxis, but an isolated atrial septal defect is not associated with an increased risk of IE after dental procedures.

A 45-year-old female sees you for follow-up after an emergency department visit in which CT of the abdomen and pelvis was performed to detect kidney stones. Kidney stones were not seen and her flank pain was determined to be musculoskeletal in origin. She is feeling better now. However, the CT showed a simple-appearing 5.2-cm ovarian cyst with assessment limited by artifact. She does not have any symptoms, pelvic pain, bloating, fevers, night sweats, or unintentional weight loss. There is no family history of ovarian or breast cancer. She reports regular menstrual cycles. Which one of the following would be most appropriate at this point? A) Reassurance only B) A CA-125 level C) Ultrasonography of the pelvis D) MRI of the pelvis E) Referral to a gynecologic oncologist

ANSWER: C Ovarian incidentalomas are very common, and appropriate management depends upon the size and appearance of the incidentaloma as well as the menopausal status of the patient. The Society of Radiologists in Ultrasound states that simple cysts <5 cm in premenopausal women and simple cysts <3 cm in postmenopausal women are considered normal and do not require follow-up. The American College of Radiology recommends that immediate ultrasonography be performed in the evaluation of simple-appearing cysts that are incompletely characterized by CT and are >5 cm in premenopausal women or >3 cm in postmenopausal women. This patient's cyst appears benign, but further evaluation is recommended due to the large size. CA-125 levels have low sensitivity and specificity in premenopausal women and would not be indicated in this case. The initial imaging of choice is pelvic ultrasonography, so MRI of the pelvis is not necessary. Referral to a gynecologic oncologist is not indicated because there is no current evidence to suggest malignancy.

According to the Institute of Medicine, which one of the following domains of health care quality is most impacted by social determinants of health? A) Effectiveness B) Efficiency C) Equity D) Safety E) Timeliness

ANSWER: C Social determinants of health are key drivers of health inequities because of their impact on the health of patients. Some examples of social determinants are socioeconomic status, education, employment, social support networks, and neighborhood characteristics. These social determinants of health have a greater impact on the health of a population than health care, behavior, or biologic factors. Thus, a focus on health equity is an essential component of population health. The 2001 Institute of Medicine (IOM) Crossing the Quality Chasm report defined six domains of health care quality, including care that does not vary regardless of factors such as gender, ethnicity, geographic location, and socioeconomic status. The other IOM-defined domains of quality care include providing health care services likely to benefit the patient while avoiding those not likely to benefit (effectiveness), avoiding waste in health care (efficiency), avoiding harm to patients (safety), avoiding unnecessary waiting for care and harmful delays in care (timeliness), and care that respects individual patient values (patient-centered care).

A 57-year-old female with a history of diabetes mellitus, hypertension, and depression sees you for a routine follow-up visit. Her vital signs include a heart rate of 88 beats/min, a blood pressure of 162/84 mm Hg, and a BMI of 32 kg/m2. The recommended antihypertensive regimen for reducing cardiovascular events in this patient is an ACE inhibitor plus A) an beta-blocker B) an angiotensin receptor blocker C) a calcium channel blocker D) a loop diuretic

ANSWER: C The ACCOMPLISH trial demonstrated that an ACE inhibitor (ACEI) in combination with a calcium channel blocker (CCB) reduced both fatal and nonfatal cardiovascular events in patients with diabetes mellitus and hypertension. The benefit of an ACEI and a CCB for reducing cardiovascular events was greater than that of an ACEI and a thiazide diuretic. Evidence has shown that combination therapy for most patients should include a CCB, an ACEI or angiotensin receptor blocker (ARB), or a thiazide diuretic (SOR A). The American College of Cardiology/American Heart Association guidelines recommend against centrally acting medications such as beta-blockers for first-line therapy. Combining ACEIs and ARBs is not recommended, as the risk of side effects such as hyperkalemia outweighs the benefits. Loop diuretics are not considered first-line antihypertensive agents.

An 81-year-old female with a history of hypothyroidism, type 2 diabetes, and stage 4 chronic kidney disease presents to your clinic with moderate to severe low back pain that began 2 days ago. A history is negative for trauma, fever, and other systemic symptoms. Her vital signs are within normal limits. A physical examination reveals moderate point tenderness to palpation across the lower thoracic and upper lumbar spine, but is otherwise unremarkable, including a negative straight leg raising test. A plain radiograph of the thoracic and lumbar spine reveals degenerative changes and suggests an osteoporotic compression fracture. A CBC, erythrocyte sedimentation rate, and C-reactive protein level are normal. Which one of the following would be most appropriate at this point? A) Denosumab (Prolia) B) Scheduled ketorolac C) Scheduled acetaminophen and lidocaine patches D) A fentanyl patch (Duragesic), 25 g/hr E) Referral to a neurosurgeon for percutaneous vertebral augmentation

ANSWER: C The differential diagnosis for acute nonradicular low back pain is broad and should include osteoarthritis, discitis, myofascial pain, and vertebral compression fracture, among other possibilities. This patient's age and comorbidities, as well as the examination and normal laboratory findings, make vertebral compression fracture a likely possibility. The most appropriate next step is to proceed with conservative pain management with scheduled acetaminophen and topical lidocaine patches. While denosumab may be appropriate management in the future if this patient's workup reveals osteoporosis, it is not indicated for initial pain management. NSAIDs such as ketorolac should be avoided in patients with significant chronic kidney disease. Long-acting opioids are not recommended for initial treatment of acute pain. Significant controversy persists regarding the use of percutaneous vertebral augmentation (vertebroplasty or kyphoplasty) for vertebral fractures, in large part because two randomized, controlled trials suggested no benefit to the procedures over placebo. The American Academy of Orthopaedic Surgeons recommends that procedural intervention only be considered if initial conservative management fails (SOR C).

43-year-old male presents to the emergency department with the acute onset of sharp, stabbing chest pain when inhaling and exhaling. The pain worsens with coughing and deep breathing. He has no significant previous medical history but recently returned from a work trip to Japan and has noted right leg swelling for the past week. He has no other symptoms. He is a smoker and has a family history of coronary artery disease in his paternal grandfather. On examination he appears uncomfortable, and his lungs are clear. A cardiac examination is notable for tachycardia. He has a blood pressure of 110/70 mm Hg, a heart rate of 112 beats/min, a respiratory rate of 18/min, a temperature of 37.7°C (99.9°F), and an oxygen saturation of 89% on room air. Which one of the following test results is most likely to confirm the diagnosis? A) Elevated troponin levels B) Acid-fast bacilli on a Gram stain C) A filling defect on CT angiography D) Air in the pleural space on a chest radiograph E) Diffuse ST elevation on an EKG

ANSWER: C The differential diagnosis of pleuritic chest pain includes several serious causes that should be considered in the evaluation of a patient with this type of pain. Pulmonary embolism is the most common cause of pleuritic chest pain. This patient presents with the acute onset of pleuritic chest pain associated with travel, a swollen leg, and smoking, which are common risk factors for pulmonary embolism. A filling defect on CT angiography can confirm this diagnosis. Elevated troponin levels would confirm a diagnosis of acute myocardial infarction, which would be more likely if the patient were older, experienced pain with exertion, and had other red-flag symptoms such as diaphoresis, nausea and vomiting, or radiating pain. Acid-fast bacilli on a Gram stain would confirm a diagnosis of tuberculosis (TB), which is associated with travel to or exposure to contacts from high-risk areas. TB would also present with other red-flag symptoms such as hemoptysis, fever, night sweats, and weight loss. A chest radiograph showing air in the pleural space would confirm a diagnosis of pneumothorax, which is usually present with decreased breath sounds on physical examination. An EKG with diffuse ST-segment elevation would confirm a diagnosis of pericarditis, which is usually associated with a recent or current viral infection or prior history of pericarditis.

The mother of a 1-year-old child wants to know how useful influenza vaccine is for preventing influenza in young children. The influenza season is typically significant in your area and the child attends day care. You answer her question by explaining how many children need to be vaccinated in order to prevent one case of influenza. This statistic is referred to as the A) absolute risk reduction B) relative risk reduction C) number needed to treat D) number needed to harm E) prevalence

ANSWER: C The number needed to treat (NNT) is the number of patients that must be treated with a particular therapy to prevent one bad outcome. It is calculated as 1/absolute risk reduction (ARR) where the ARR is written as a decimal (if the ARR is 5%, the NNT = 1/0.05 = 20). ARR is the arithmetic difference in risk or outcome rates between the treatment group and the control group. The relative risk reduction indicates how much the risk or outcome was reduced in the treatment group compared to the control group. The number needed to harm is the number of patients necessary to receive an intervention instead of the alternative in order for one additional patient to experience an adverse event. Prevalence is the proportion of a particular population found to be affected by a medical condition (SOR A). According to a recent Cochrane review, five children would need to receive influenza vaccination to prevent one case of influenza, and 12 children would need to be vaccinated to prevent influenza-like illness.

A 72-year-old female has a known history of aortic stenosis. Which one of the following symptoms or physical findings would indicate a need for urgent repeat echocardiography and cardiology referral? A) Headache B) Palpitations C) Presyncope D) Edema of both lower extremities E) A grade 2/6 systolic ejection murmur radiating to the aorta

ANSWER: C The three cardinal symptoms of aortic stenosis are angina, dyspnea, and presyncope or syncope. Once these events occur, the natural history of the disease changes dramatically. The risk for death increases from <1% per year to 2% per month, such that 75% of symptomatic patients die within 3 years unless they receive a valve replacement. Headache, palpitations, edema of the lower extremities, and a grade 2/6 ejection systolic murmur radiating to the aorta are not indicative of severe aortic stenosis.

Your practice is starting a screening program for depression. Which one of the following is a cardinal symptom of depression included in the Patient Health Questionnaire-2 (PHQ-2) screening instrument? A) Fatigue B) Lack of appetite C) Little interest or pleasure in doing things D) Restlessness E) Sleep disturbance

ANSWER: C The two cardinal symptoms of depression are depressed mood and anhedonia. These are the two criteria in the Patient Health Questionnaire-2 (PHQ-2) screening instrument for depression. A positive result on the PHQ-2 should prompt further evaluation, including questions about other symptoms such as energy level, appetite changes, sleep disturbance, psychomotor changes, and suicidality.

A 38-year-old male comes to your office for follow-up of his diabetes mellitus. He takes metformin (Glucophage) and dulaglutide (Trulicity) and his hemoglobin A1c is 6.5%. He has an LDL-cholesterol level of 120 mg/dL, an HDL-cholesterol level of 55 mg/dL, and a triglyceride level of 190 mg/dL. He asks your advice about statin therapy for cholesterol management. You advise him to start A) a low-intensity statin now B) a moderate-intensity statin now C) statin therapy at 40 years of age D) statin therapy when his 10-year atherosclerotic cardiovascular disease (ASCVD) risk is >5% E) statin therapy when his 10-year ASCVD risk is >7.5%

ANSWER: C There is high-quality evidence from randomized, controlled trials that a moderate-intensity statin should be initiated for all patients age 40-75 years with diabetes mellitus regardless of their calculated 10-year atherosclerotic cardiovascular disease (ASCVD) risk. There is not strong evidence supporting the use of statins before age 40 in patients with diabetes unless their LDL-cholesterol level is very high. The ASCVD risk score is valid for patients over 40 years of age and cannot be calculated before then.

An 18-year-old male is brought to the urgent care clinic by his friends several hours after he cut his hand when he punched someone in the mouth during a fight involving several people. On examination he is slightly intoxicated but alert, and shows no signs of apparent distress. He has multiple superficial scratches on his arms and legs, swelling over his left eye, and a 5-mm laceration over the third metacarpophalangeal joint of his right hand with intact sensation and motor function and no active bleeding. There is no appreciable erythema. Which one of the following would be the most appropriate treatment of his hand injury? A) Irrigation of the wound alone B) Irrigation of the wound followed by suturing of the laceration C) Irrigation of the wound and oral amoxicillin/clavulanate (Augmentin), with follow-up in 24 hours D) Intramuscular ceftriaxone, with follow-up in 24 hours E) Referral to the hospital for admission and intravenous antibiotics

ANSWER: C This patient has a clenched-fist bite wound. Without antibiotics these wounds have a high rate of infection, and irrigation alone is not sufficient. Because this patient has a superficial wound with no evidence of tendon involvement or functional compromise, he does not require parenteral antibiotics and can be managed as an outpatient. These wounds should be allowed to heal by secondary intention, so suturing is not appropriate.

A 65-year-old female sees you to establish primary care. Her past medical history is significant only for hypertension, which is well managed on hydrochlorothiazide, and menopausal symptoms managed with oral estrogen/progestogen for the past 10 years. She has no current health concerns and her vital signs are normal. She requests a refill of her hormone medication. Which one of the following would be most appropriate at this time? A) Refill her hormone medication, but with a tapering schedule to end the therapy B) Recommend switching from oral hormone therapy to vaginal estrogen and oral progestogen C) Discuss the possible risks of continued hormone therapy before deciding whether she should continue D) Order a bone density test and use the results to determine whether to continue hormone therapy E) Recommend discontinuation of hormone therapy as soon as possible, as she has already been on the treatment for more than 5 years

ANSWER: C This patient has been on menopausal hormone therapy for an extended period of time. A discussion about cessation is warranted, as it is common practice to use hormones for the shortest duration possible. The American College of Obstetricians and Gynecologists does not recommend discontinuation based on age or duration of treatment alone, however, recommending instead that the patient history and symptoms be taken into account. Tapering hormones versus abrupt discontinuation of hormones is not well studied, and patients can do well with either plan. Vaginal estrogen should be used with similar cautions as oral estrogen and is primarily indicated for genitourinary symptoms of menopause, so switching from oral estrogen to vaginal estrogen would not be indicated in this case. Menopausal hormone therapy does decrease the risk for hip fractures, but the American Academy of Family Physicians, like the U.S. Preventive Services Task Force, does not recommend using hormone therapy for prevention of chronic conditions, so the patient's bone density should not be a consideration.

A 28-year-old female presents with a several-month history of gradually worsening pain on the radial side of the right wrist that increases while she is at work. She does not have any numbness or tingling. Her symptoms began after she started a new job as a machinist operating a press, which involves repetitive grabbing and pulling. An examination reveals no gross abnormalities and only minimal soft-tissue tenderness around the base of the thumb. Range of motion of the thumb is limited by pain that is provoked when she makes a fist over her thumb and moves the hand into ulnar deviation. There is no pain with axial compression or rotation of the first metacarpophalangeal joint. Which one of the following is the most likely diagnosis? A) Arthritis of the first carpometacarpal joint B) Carpal tunnel syndrome C) de Quervain's tenosynovitis D) Stress fracture of the scaphoid E) Ulnar nerve entrapment

ANSWER: C This patient has de Quervain's tenosynovitis, which is caused by inflammatory changes in the extensor pollicis brevis and the abductor pollicis longus or their tendon sheaths. It should be suspected when a patient presents with insidious pain in the radial wrist at the base of the thumb. Some patients may report a history of new repetitive use of the hand. It is more common in women, especially new mothers who are repeatedly picking up their children. The Finkelstein test, which involves making a fist over the thumb and moving the hand into ulnar deviation, has good sensitivity and specificity and is usually sufficient to make the diagnosis. Arthritis of the first carpometacarpal joint typically involves pain with axial compression of the carpometacarpal joint. Carpal tunnel syndrome is produced by compression of the median nerve at the wrist. Affected patients report numbness, tingling, and pain in the hand, which often worsens at night or after use of the hand. While patients with de Quervain's tenosynovitis may have tenderness around the soft tissues of the anatomic snuffbox, deep tenderness in the snuffbox would be more concerning for an underlying scaphoid fracture. This patient does not have a history of falling onto an outstretched hand, which is usually the mechanism of injury. Ulnar nerve entrapment causes ulnar arm pain and numbness in the fourth and fifth fingers.

A 42-year-old male presents with a 10-day history of hoarseness. He also has a 2-month history of reflux symptoms and has been taking antacids as needed. He does not take any other medications. There is no history of fever, weight loss, night sweats, or appetite changes. You note that the patient is hoarse, and a physical examination is normal, including HEENT, cardiovascular, and pulmonary examinations. He is a member of a community choir that rehearses twice a week. In addition to voice rest, which one of the following would be most appropriate at this time? A) Supportive care only B) Azithromycin (Zithromax) C) Omeprazole (Prilosec) D) A course of prednisone

ANSWER: C This patient has hoarseness that has been present for less than 2 weeks. In addition to voice rest, treatment in patients with a history of GERD should include a 3- to 4-month trial of a high-dose proton pump inhibitor (SOR C). In patients with hoarseness lasting longer than 2 weeks without an apparent benign etiology, the larynx should be examined by direct or indirect laryngoscopy (SOR C). Antibiotics and oral corticosteroids should not be used for the empiric treatment of hoarseness in the absence of signs and symptoms that suggest an underlying cause.

A 42-year-old Asian male presents for follow-up of elevated blood pressure. He has no additional chronic medical problems and is otherwise asymptomatic. An examination is significant for a blood pressure of 162/95 mm Hg but is otherwise unremarkable. Laboratory Findings unremarkable Urine microalbumin negative According to the American College of Cardiology/American Heart Association 2017 guidelines, which one of the following would be the most appropriate medication to initiate at this time? A) Clonidine (Catapres), 0.1 mg twice daily B) Hydralazine, 25 mg three times daily C) Lisinopril/hydrochlorothiazide (Zestoretic), 10/12.5 mg daily D) Metoprolol tartrate (Lopressor), 25 mg twice daily E) Triamterene (Dyrenium), 50 mg daily

ANSWER: C This patient has hypertension and according to both JNC 8 and American College of Cardiology/American Heart Association 2017 guidelines, antihypertensive treatment should be initiated. For the general non-African-American population, monotherapy with an ACE inhibitor, an angiotensin receptor blocker, a calcium channel blocker, or a thiazide diuretic would be appropriate for initial management. It is also appropriate to initiate combination antihypertensive therapy as an initial management strategy, although patients should not take an ACE inhibitor and an angiotensin receptor blocker simultaneously. Studies have shown that blood pressure control is achieved faster with the initiation of combination therapy compared to monotherapy, without an increase in morbidity. Lisinopril/hydrochlorothiazide would be an appropriate choice in this patient. -Blockers, vasodilators, -blockers, and potassium-sparing diuretics are not recommended as initial choices for the treatment of hypertension.

A 67-year-old female with a history of type 2 diabetes, hypertension, osteoarthritis, and hyperlipidemia sees you for follow-up of laboratory results after hepatic steatosis and mild splenomegaly were noted on abdominal ultrasonography performed during a recent emergency department visit for right upper quadrant pain. She is currently asymptomatic. Her vital signs include a blood pressure of 128/72 mm Hg and a BMI of 34 kg/m2. An examination is significant only for trace bilateral lower extremity edema. A cardiovascular examination is otherwise normal and neurologic and abdominal examinations are normal. Laboratory Findings Platelets 112,000/mm3 (N 150,000-450,000) Creatinine 0.8 mg/dL (N 0.6-1.2) Hemoglobin A1c 6.6% AST 68 U/L (N 8-48) ALT 55 U/L (N 7-55) Albumin 3.2 g/dL (N 3.5-5.0) Bilirubin 0.5 mg/dL (N 0.1-1.2) INR 1.0 (N <1.2) Which one of the following medications should be AVOIDED in this patient to reduce the likelihood of complications of her medical condition? A) Acetaminophen B) Atorvastatin (Lipitor) C) Ibuprofen D) Lisinopril (Prinivil, Zestril) E) Metformin (Glucophage)

ANSWER: C This patient has previously unrecognized compensated hepatic cirrhosis. While the diagnosis of cirrhosis should be confirmed and assessed by methods such as transient elastography, the family physician should recognize this as presumed cirrhosis based on the splenomegaly and laboratory findings. NSAIDs such as ibuprofen should be avoided in patients with cirrhosis due to the risk of renal insufficiency (SOR B). While toxic to the liver at high doses, acetaminophen can be safely used for analgesia in cirrhotic patients, though many hepatologists recommend limiting dosing to 2 g daily (SOR C). Statins can be safely used in compensated cirrhosis and may improve steatosis (SOR A). Lisinopril and metformin can also be used safely, hypertension should not be overtreated as cirrhosis progresses to a decompensated state.

A 65-year-old female with hypertension and hyperlipidemia sees you for follow-up after a visit 1 week ago because of uncontrolled hypertension and new-onset type 2 diabetes. She has smoked one pack of cigarettes per day for the past 40 years. Her medications include amlodipine (Norvasc), 10 mg daily; hydrochlorothiazide, 12.5 mg daily; and atorvastatin (Lipitor), 20 mg daily. At last week's visit you added metformin (Glucophage), 500 mg twice daily, and lisinopril (Prinivil, Zestril), 5 mg daily. The patient is physically active and asymptomatic. She is 165 cm (65 in) tall and weighs 59 kg (130 lb) with a BMI of 22 kg/m2. A physical examination is normal. A summary of her recent vital signs and laboratory data is listed below. One week ago Today Blood pressure 165/92 mm Hg 162/88 mm Hg Hemoglobin A1c 7.2% not measured Glucose 183 mg/dL 156 mg/dL Potassium 4.2 mEq/L (N 3.5-5.0) 5.1 mEq/L BUN 12 mg/dL (N 8-25) 20 mg/dL Creatinine 0.8 mg/dL (N 0.6-1.2) 1.4 mg/dL You diagnose resistant hypertension and consider evaluating for an underlying cause of secondary hypertension. Which one of the following tests would most likely yield a diagnosis in this case? A) Plasma free metanephrines B) A low-dose dexamethasone suppression test C) MR angiography of the renal arteries D) Polysomnography

ANSWER: C This patient has resistant hypertension as defined by persistent uncontrolled hypertension despite the use of three adequate antihypertensives, including a diuretic. She has multiple risk factors for atherosclerotic disease, and a rise in her creatinine level after the addition of an ACE inhibitor suggests renovascular hypertension. An imaging procedure to evaluate for the presence of renovascular hypertension, such as MR angiography of the renal arteries, is indicated. Measurement of plasma free metanephrines is part of the workup for suspected pheochromocytoma. Low-dose dexamethasone suppression testing is useful for the diagnosis of Cushing syndrome. Polysomnography is the standard diagnostic test for obstructive sleep apnea, a leading cause of secondary hypertension.

A 35-year-old female comes to your office for follow-up of an emergency department (ED) visit for palpitations. She tells you that she was driving on the highway with her three small children when she suddenly felt her heart racing, along with chest tightness, lightheadedness, and severe anxiety. She pulled over and called 911. While she waited for EMS she took diltiazem (Cardizem), 30 mg orally, which had been prescribed following a similar episode several years ago. Her symptoms lasted about 10 minutes and had improved by the time EMS arrived. An examination, EKG, and chest radiograph in the ED were all normal. Which one of the following findings in her previous medical record would confirm your diagnosis? A) A Generalized Anxiety Disorder-7 (GAD-7) score of 6 B) An elevated TSH level C) P waves hidden within a narrow QRS complex on an event recorder D) Mitral valve prolapse on an echocardiogram E) Atherosclerotic plaque seen on carotid ultrasonography

ANSWER: C This patient presents with a history consistent with typical atrioventricular nodal reentrant tachycardia, which is the most common type of supraventricular tachycardia (SVT). She is also using "pill-in-the-pocket" treatment, which is effective for infrequent SVT. Because the symptoms are episodic and the tachycardia is paroxysmal, patients generally present with normal examination and EKG findings. Further evaluation with event monitoring may identify a narrow-complex tachycardia with P waves hidden within the QRS complex or identified early after it. Most patients with SVT have structurally normal hearts. An elevated Generalized Anxiety Disorder-7 (GAD-7) score is consistent with a diagnosis of generalized anxiety disorder (GAD). However, GAD is a common misdiagnosis in patients with SVT, particularly females. While hyperthyroidism is associated with tachycardia, hypothyroidism usually is not. Mitral valve prolapse is not specifically associated with SVT. Carotid atherosclerosis is not associated with SVT either, but knowledge of its presence may help determine treatment.

A 50-year-old female sees you for a routine health maintenance visit. On examination you note a thyroid nodule and ultrasonography confirms a solid 1.5-cm nodule. Her TSH level is normal. Which one of the following would be the most appropriate next step in the management of this patient? A) Antithyroid antibody titers B) A thyroid scan C) A fine-needle aspiration biopsy D) Surgical excision of the nodule E) Repeat ultrasonography in 6 months

ANSWER: C Thyroid nodules 1 cm that are solid or have suspicious features require a fine-needle aspiration biopsy to rule out malignancy. Fine-needle aspiration should not be performed on nodules <1 cm. The evaluation of a single thyroid nodule does not call for testing of antithyroid antibody titers. If the patient had a low TSH level then a radionuclide thyroid uptake scan (nuclear medicine thyroid scan) is indicated to look for a toxic nodule. These hyperfunctioning nodules are seldom malignant. Surgical nodule excision is not indicated since needle aspiration is diagnostic. Repeat ultrasonography in 6 months is not indicated because this would not change management.

Which one of the following is considered one of the most common food allergens? A) Beef B) Black beans C) Fish D) Onions E) Watermelon

ANSWER: C When considering food allergens, patients should specifically be asked about foods that most commonly precipitate an IgE response. While any food might cause an allergic reaction, skin testing has limited positive predictive value, but can be helpful in the evaluation of a patient with an immediate reaction to food. A negative allergy test with a negative oral challenge has a good negative predictive value. Testing should be performed in an allergist's office in case there is an anaphylactic response. Testing should focus on foods that make up the majority of food allergens, including cow's milk, eggs, soy, fish, shellfish, tree nuts, wheat, and peanuts (level of evidence C). Beef, black beans, onions, and watermelon are not among the most common food allergens.

When performing a geriatric assessment, which one of the following is an instrumental activity of daily living? A) Bathing B) Dressing C) Transferring between the bed and a chair D) Using the telephone E) Using the toilet

ANSWER: D The foundation of geriatric assessment is assessing the individual's ability to perform tasks required for living. Activities of daily living are self-care activities that are performed daily, such as eating, bathing, dressing, transferring between the bed and a chair, and toileting, including bladder and bowel function. Instrumental activities of daily living include activities necessary to live independently, such as using a telephone, doing housework, preparing meals, taking medications properly, and managing finances.

You are advising a group of medical students who are planning a tobacco cessation program for expectant mothers. The medical students want to build an advertising program that touts the pregnancy benefits of tobacco cessation. You tell the medical students that evidence suggests that tobacco cessation in pregnant women A) decreases the risk for cesarean delivery B) decreases the risk for preeclampsia C) decreases the need for epidural anesthesia D) increases infant birth weight E) increases the risk for preterm delivery

ANSWER: D Smoking during pregnancy increases the risk for fetal growth restriction. The U.S. Preventive Services Task Force identified evidence that tobacco cessation increases infant birth weight and decreases the risk for preterm delivery. There is no evidence that tobacco cessation in pregnant women decreases the risk for cesarean delivery, the risk for preeclampsia, or the need for epidural anesthesia.

Based on American Cancer Society guidelines for cervical cancer screening, when should HPV DNA co-testing first be performed along with Papanicolaou testing? A) At the onset of sexual activity B) At age 21 C) At age 25 D) At age 30 E) At age 35

ANSWER: D According to American Cancer Society guidelines for cervical cancer screening, Papanicolaou (Pap) testing should begin at age 21 irrespective of sexual activity and should be continued every 3 years until age 29. The preferred screening strategy beginning at age 30 is Pap testing with HPV co-testing, which should be continued every 5 years until age 65. Cervical screening may be discontinued at that time if the patient's last two tests have been negative and the patient was tested within the previous 5 years.

A 22-year-old Russian female has recently been hired to work at your clinic. She is going through the orientation and screening process and you are asked to screen her for tuberculosis. She tells you that she received the bacille Calmette-Guérin (BCG) vaccine as a child. Which one of the following would be the recommended screening for this patient? A) No screening B) A tuberculin skin test (TST) C) A two-step TST D) An interferon-gamma release assay (IGRA, QuantiFERON-TB Gold) E) A chest radiograph

ANSWER: D According to the CDC, Infectious Diseases Society of America, and American Thoracic Society, an interferon-gamma release assay (IGRA) is recommended for individuals who are 5 years of age or older who are likely infected with Mycobacterium tuberculosis (TB), have a low to intermediate risk of disease progression, and either have a history of bacille Calmette-Guérin (BCG) vaccination or are unlikely to return to have their tuberculin skin test (TST) read (SOR B). A TST is an acceptable alternative if an IGRA is not available, is too costly, or is too burdensome. While testing is not recommended in persons at low risk for tuberculosis, it may be required by law or credentialing bodies. A chest radiograph would be indicated in suspected cases of currently or previously active pulmonary TB.

A 16-year-old male who was recently diagnosed with moderate persistent asthma sees you for follow-up in early September. He uses fluticasone propionate/salmeterol (Advair), 250/50 g twice daily. He has an Asthma Control Test score of 22, which indicates well controlled asthma, and his peak flow is 90% of the predicted value. Spirometry performed 6 months ago showed an FEV1 of 90% of the predicted value. He can demonstrate good technique for using his inhalers. Which one of the following would be most appropriate at this time? A) Allergy testing B) The Asthma Therapy Assessment Questionnaire C) Increasing fluticasone propionate/salmeterol to 500/50 micrograms twice daily D) Influenza vaccine E) Spirometry

ANSWER: D According to the National Asthma Education and Prevention Program and the CDC, influenza vaccination is an important part of asthma care. Influenza and pneumonia can be more serious for patients with asthma. Asthma is the most common medical condition in patients hospitalized with influenza. Both adults and children with asthma are more likely to develop pneumonia with influenza. Influenza vaccination has been associated with reduced oral corticosteroid use in asthma exacerbations and reduced severity-adjusted asthma exacerbations. Pneumococcal vaccine is also recommended but there is little data to show improved patient-oriented outcomes for asthma patients vaccinated against pneumococcal disease. Unless the patient has new symptoms, there is no indication for allergy testing. This patient's Asthma Control Test score indicates good control, so there is no reason to add an additional control test such as the Asthma Therapy Assessment Questionnaire. An increase in fluticasone propionate/salmeterol to 500/50 micrograms twice daily is not indicated when asthma is well controlled. This patient had spirometry performed 6 months ago, so spirometry is not indicated at this time.

In a patient with documented Helicobacter pylori infection, which one of the following studies should be performed to document clearance of the infection? A) An H. pylori IgG level B) An H. pylori IgM level C) A stool antigen test 1 week after the completion of antibiotic therapy D) A urea breath test 1 month after the completion of antibiotic therapy E) A gastric biopsy

ANSWER: D After treatment for a Helicobacter pylori infection it is essential to document clearance of the infection. This is typically done with a stool antigen test or a urea breath test performed 1 month after the completion of antibiotic therapy. If the patient is taking a proton pump inhibitor it should be discontinued prior to the test. H. pylori IgG or IgM levels and gastric biopsies are not appropriate for documenting clearance of H. pylori infection.

Which one of the following medications should be considered for patients taking more than 50 morphine milligram equivalents (MMEs) daily for chronic pain? A) Clonazepam (Klonopin) B) Clonidine (Catapres) C) Methadone D) Naloxone E) Naltrexone (Vivitrol)

ANSWER: D All patients taking more than 50 morphine milligram equivalents (MMEs) daily of chronically prescribed opioids should also be prescribed naloxone in case of overdose. Naloxone is an opioid antagonist that displaces opioids from their receptors and reverses the effects. Benzodiazepines such as clonazepam increase the risk of overdose. Clonidine has been used to treat symptoms of opiate withdrawal. Methadone is a full opioid agonist used in opioid use disorder or to treat chronic pain. Naltrexone is an opioid antagonist that must be refrigerated and administered by trained medical staff.

A 39-year-old female presents to your office for evaluation of a left-sided headache. She notes pain in the temporal region and inferior to the zygoma. The pain is constant and dull but worsens with chewing. There is no history of recent trauma. Her past medical history includes fibromyalgia and her only medication is amitriptyline, 25 mg at bedtime. Her vital signs are within normal limits and she is afebrile. On examination you note tenderness to palpation over the temporalis region but no mass or cord. There is no swelling or edema of the head, face, or neck. Which one of the following is the most likely diagnosis? A) Giant cell arteritis B) A salivary stone C) Sinusitis D) Temporomandibular disorder E) Trigeminal neuralgia

ANSWER: D Approximately 10%-15% of adults will experience a temporomandibular disorder (TMD). TMD represents a spectrum of illnesses frequently seen and readily treatable by family physicians. This multifactorial disorder is consistently associated with other pain conditions such as fibromyalgia, as in this case. Patients typically present with facial pain, jaw pain, headache, or ear pain. The symptoms are generally associated with jaw movement such as chewing. Physical examination findings can be broad but often include pain with palpation of the temporomandibular joint, or pain and/or spasm of the muscles of mastication. Giant cell arteritis should be included in the differential diagnosis but is typically seen in patients over age 50 and is often associated with other findings such as jaw claudication, visual symptoms, and palpable abnormalities over the temporal artery. Salivary stones can involve pain in a similar region, such as the parotid gland, but this is often an intermittent pain triggered only by eating. Tenderness or swelling over the gland itself may be seen. Sinusitis would usually present with nasal congestion, maxillary sinus tenderness, mucus, and fever. While trigeminal neuralgia may have a similar distribution of pain, this is typically reported as a brief attack of intense, sharp pain often produced by specific stimuli.

You see a 22-year-old female at 12 weeks estimated gestation. Antibody testing indicates that she is rubella equivocal. Which one of the following is recommended by the CDC's Advisory Committee on Immunization Practices? A) Recheck her rubella antibody titer after delivery B) Administer MMR vaccine immediately C) Administer MMR vaccine after the first trimester D) Administer MMR vaccine after delivery E) Recommend vaccination only if the patient has no prior record of receiving MMR

ANSWER: D Because of successful vaccination programs rubella has nearly been eliminated from the United States, but it is still common in other countries. Susceptible individuals can still become ill while traveling abroad. According to the CDC, women known to be pregnant or attempting to become pregnant should not receive a live-virus vaccine such as MMR. In a pregnant patient with an equivocal rubella titer it should not be assumed that the patient has protective immunity, and the patient should be offered booster vaccination after delivery. It is also recommended that women should not become pregnant for 4 weeks after receiving MMR vaccine.

healthy 2-month-old female born at term is brought to your office for a well child check. The parents report that the child is exclusively breastfed and ask whether they should be providing any additional nutrition. Which one of the following would you advise? A) No supplementation until the child is 6 months old B) Water C) Fluoride D) Vitamin D E) Rice cereal

ANSWER: D It is recommended that all infants should be given 400 IU of vitamin D because of generally decreased sun exposure in today's living situations. Additional water consumption can decrease milk intake and cause electrolyte disturbances. Fluoride supplementation and the introduction of cereal are not recommended until 6 months of age.

An older patient presents to your office with findings consistent with polymyalgia rheumatica. This patient is at greatest risk for which one of the following associated conditions? A) Antineutrophil cytoplasmic antibody-associated vasculitis B) Polyarteritis nodosa C) Takayasu arteritis D) Temporal arteritis E) Wegener's granulomatosis

ANSWER: D Both polymyalgia rheumatica (PMR) and giant cell arteritis are chronic inflammatory diseases. PMR is the most common chronic inflammatory condition in older adults. Giant cell arteritis is common in patients with PMR. Giant cell arteritis can affect any medium or large artery, particularly the extracranial carotid branches. The temporal artery is commonly involved, and the ophthalmic artery may also be affected. This can result in neuro-ophthalmic complications, including permanent blindness. For this reason giant cell arteritis is considered a medical emergency and it is essential for family physicians to evaluate any patient with PMR for giant cell arteritis. Antineutrophil cytoplasmic antibody-associated vasculitis, polyarteritis nodosa, Takayasu arteritis, and granulomatosis with polyangiitis (formerly called Wegener's granulomatosis) are not uniquely associated with PMR.

A 17-year-old male presents to your office to be screened for celiac disease. His older sister was recently diagnosed with biopsy-confirmed celiac disease. He has had intermittent abdominal bloating but no other symptoms. Which one of the following is the current best practice to screen for celiac disease in this situation? A) No screening and treatment based on family history B) Antigliadin antibodies C) Endomysial antibodies D) Serum total IgA levels and IgA tTG antibodies E) Esophagogastroduodenoscopy with small bowel biopsies

ANSWER: D Celiac disease affects about 10% of first degree relatives of a person with celiac disease and a heightened suspicion should be maintained in this higher-than-average risk group. Many patients are asymptomatic or only minimally symptomatic. Several serologic tests are available for initial screening, and should be followed by a small bowel biopsy for confirmation. IgA tTG antibody testing is currently the test of choice and should be paired with serum total IgA levels, as IgA deficiency is 10-15 times more common in patients with celiac disease than in the general population. Because only up to 10% of first degree relatives are affected, committing this patient to a lifelong dietary restriction without confirmatory testing may be unnecessary. Antigliadin antibody testing has low sensitivity and specificity. Endomysial antibody testing has higher specificity and sensitivity, but it is very costly. An initial esophagogastroduodenoscopy is likely too invasive and expensive for screening purposes in this patient.

The U.S. Preventive Services Task Force recommends which one of the following for breast cancer screening? A) Annual screening mammography for women ages 40-75 B) Annual screening mammography for women ages 50-75 C) Biennial screening mammography for women ages 35-75 D) Biennial screening mammography for women ages 50-75 E) Biennial screening mammography for women ages 45-50 and annual screening mammography for women ages 51-75

ANSWER: D Current U.S. Preventive Services Task Force guidelines for breast cancer screening recommend biennial screening mammography for women ages 50-75 (B recommendation). Biennial screening mammography can be considered for women age 40-49 after discussing the risks and benefits with the patient (C recommendation).

A 56-year-old female with a BMI of 37 kg/m2 seeks your advice regarding weight loss. Which one of the following medications in her current regimen is most likely to contribute to weight gain? A) Bupropion (Wellbutrin) B) Lisinopril (Prinivil, Zestril) C) Metformin (Glucophage) D) Mirtazapine (Remeron) E) Naproxen

ANSWER: D Mirtazapine, an antidepressant, is associated with weight gain. Lisinopril and naproxen are weight neutral. Bupropion and metformin may promote weight loss.

A 36-year-old male presents with a 6-week history of a mildly pruritic rash in his groin. An examination reveals small red-brown macules and larger patches with a sharp border. A Wood's lamp examination reveals coral-red fluorescence. Which one of the following would be the most appropriate treatment for this condition? A) 0.1% triamcinolone cream B) 2.5% hydrocortisone cream C) Nystatin cream D) Erythromycin gel E) Fluconazole (Diflucan) orally

ANSWER: D Erythrasma is a superficial infection caused by Corynebacterium minutissimum. It presents as small, red-brown macules that may coalesce into larger patches with sharp borders. It fluoresces coral red on Wood's lamp examination. Cutaneous erythrasma is treated with erythromycin (topical, twice daily until the rash resolves, or oral, 250 mg four times daily for 2 weeks). Topical clindamycin, Whitfield ointment, and antibiotic soaps may also be beneficial. Triamcinolone cream, hydrocortisone cream, nystatin cream, and oral antifungals are not effective treatments for this bacterial infection.

A 43-year-old male with uncomplicated appendicitis prefers antibiotic therapy alone over surgical treatment. You advise him that this will increase his risk for which one of the following? A) A longer absence from work B) The need for pain medications C) Disability D) Subsequent surgery

ANSWER: D Evidence shows that antibiotic therapy alone may be a viable treatment option for patients with uncomplicated appendicitis. A meta-analysis of five randomized, controlled trials found that antibiotic treatment resulted in decreased complications, less need for pain medication, and less sick leave or disability compared with initial appendectomy. However, recurrence rates requiring surgery within a year may be as high as 40% compared to a less than 10% risk of repeat surgery in those who have an appendectomy.

An elderly homeless male is brought to the emergency department. He is clearly hypothermic due to cold exposure and has superficial frostbite of his extremities. He is still conscious and shivering. In addition to rewarming him, which one of the following should you administer? A) Acetazolamide (Diamox Sequels) B) Amitriptyline C) Ceftriaxone D) Ibuprofen E) tPA

ANSWER: D Frostbite is a freezing injury that occurs when initial cooling causes vasoconstriction and localized ischemia. Continued cold exposure leads to ice crystal formation, which causes cellular lysis, electrolyte abnormalities, and microvascular occlusion. Rewarming creates an inflammatory response. Ibuprofen is the most appropriate agent for the treatment of frostbite until the wounds heal or surgery is performed (SOR C). Acetazolamide can cause frostbite at high altitudes. Amitriptyline is used to treat the pain of immersion foot (also called trench foot), which is a nonfreezing injury that happens when the foot is exposed to prolonged wet conditions above 0°C (32°F). Antibiotics are indicated if open or dirty wounds are present (SOR B). tPA has a role in treating patients with frostbite, but it is used only to decrease the risk of amputation when rewarming patients with grade 3, grade 4, or deep frostbite (SOR B).

You are asked for your advice as part of a committee formed by your local health system to focus on fall prevention. Based on U.S. Preventive Services Task Force recommendations, which one of the following interventions has the strongest evidence for preventing falls in community-dwelling older adults at increased risk for falls? A) Calcium supplementation B) Vitamin D supplementation C) Supportive footwear D) Exercise classes E) Cognitive-behavioral therapy

ANSWER: D In the United States falls are the leading cause of injury-related morbidity and mortality among older adults. The U.S. Preventive Services Task Force (USPSTF) concluded with moderate certainty that exercise interventions provide a moderate net benefit in fall prevention in community-dwelling adults 65 years of age or older who are at increased risk for falls (B recommendation). The USPSTF also concluded with moderate certainty that supplementation with calcium and vitamin D has no clear benefit in preventing falls in older adults. Environmental modifications and psychological interventions lack sufficient evidence for fall prevention.

A 60-year-old retired dock worker presents to your office with chronic low back pain due to multiple herniated lumbar discs, with radicular pain down both legs. He rates his pain as a 3 out of 10. He currently takes oxycodone (OxyContin), 10 mg every 12 hours; pregabalin (Lyrica), 150 mg every 12 hours; acetaminophen, 1000 mg every 8 hours; meloxicam (Mobic), 15 mg daily; and cyclobenzaprine, 10 mg every 8 hours. These medications have been prescribed by another physician for the past 5 years. He tells you that his pain is tolerable but his sex drive and energy level have steadily decreased since starting these medications. Which one of the following medications in this patient's regimen would be most likely to decrease his libido? A) Acetaminophen B) Cyclobenzaprine C) Meloxicam D) Oxycodone E) Pregabalin

ANSWER: D Opioid use may cause numerous adverse reactions, including drowsiness, pruritus, nausea, and constipation. In addition to these well known side effects, chronic opioid use can lead to hypogonadism through inhibition of gonadotropin-releasing hormone and an increase in prolactin. A recently published study found that long-term opioid users were nearly twice as likely to be diagnosed with hypogonadism as short-term opioid users. Given the large number of opioid users in the United States, prescribers should be aware of this adverse effect and screen for hypogonadism when appropriate. None of the other medications in this patient's regimen, including acetaminophen, cyclobenzaprine, meloxicam, and pregabalin, are the likely cause of his decreased libido.

A 27-year-old male has noticed some small bumps around the tip of his penis and is concerned that he might have a sexually transmitted infection. An examination reveals a row of approximately 2-mm, dome-shaped, skin-colored papules in a ring-like distribution around the corona of the glans penis. Which one of the following is the most likely diagnosis? A) Angiokeratomas B) Genital warts C) Lichen nitidus D) Pearly papules E) Squamous cell carcinoma

ANSWER: D Pearly papules are a benign, normal anatomic variant and are not sexually transmitted. They are dome-shaped, skin-colored papules 1-4 mm in size with a ring-like distribution around the corona of the glans penis. Angiokeratomas are red or blue papules that are well circumscribed and 1-6 mm in size. Genital warts are soft, raised masses that can be pearly and smooth or have a rough, cauliflower-like appearance, and are not confined to the penile corona. Lichen nitidus consists of discrete, hypopigmented, 1-mm papules that are not confined to the corona and can also occur on the upper extremities and abdomen. Squamous cell carcinoma may be endophytic (ulcerated) or exophytic (thickened skin or wart-like growths that can progress to a large, irregularly shaped, fungating mass).

An 85-year-old nursing home patient with dementia who has bilateral hearing aids has been slightly more confused over the past 2 weeks according to the staff. He is also speaking at a louder volume than normal. He does not have any pain, but an examination shows impacted cerumen in both ears. Which one of the following would be most appropriate in the management of this patient? A) No therapy B) Irrigation of the ears with cold water C) Use of olive oil drops D) Removal of the cerumen using peroxide E) Restraining the patient and attempting manual removal of the cerumen

ANSWER: D Removal of cerumen should be attempted when the patient has symptoms such as pain, tinnitus, hearing loss, or itching. Removal of the impaction is also indicated in patients who are not able to communicate about their symptoms, such as patients with developmental delay or dementia, a nonverbal patient who has had recent behavioral changes, or children with fever or speech delay. Cerumen impaction resulting in hearing loss can cause reversible cognitive impairment in older persons with dementia. Treatment options include irrigation with warm water, cerumenolytic agents such as carbamide peroxide otic, or manual removal if the patient is cooperative and if the procedure can be completed without the use of restraints. The use of cold water, olive oil drops, ear candling, or cotton-tipped swabs should be avoided.

A healthy 18-year-old African-American female comes to your office with a 5-day history of sharp chest pain that worsens with both inspiration and expiration. She has also noticed that the pain worsens with laughing or coughing. She had symptoms of an upper respiratory infection a week ago with no fever or shortness of breath. She has no history of trauma and no significant past medical history, and takes no regular medications. Her vital signs are normal, including oxygen saturation, and a physical examination is unremarkable. A chest radiograph is also normal. Which one of the following would be the most appropriate next step? A) A D-dimer level B) An EKG C) Spiral chest CT D) Diclofenac, 75 mg twice daily E) Prednisone, 50 mg daily for 5 days

ANSWER: D Ruling out life-threatening causes of pleuritic chest pain is the most important consideration in a pleurisy evaluation. A full history and complete physical examination with vital signs should be performed (SOR C). Tachycardia, tachypnea, hypotension, fever, or respiratory distress should raise concerns. Chest radiography should be performed if the cause of the pain is unclear (SOR C). If no red flags are raised on examination and a chest radiograph is clear, a trial of NSAIDs should be started (SOR B). NSAIDs are preferred to narcotic medications as they do not suppress respiratory drive and do not have the risk of addiction and abuse. If a life-threatening cause is suspected from the history and physical examination, then further diagnostic testing is indicated.

A 55-year-old male comes to your outpatient clinic for an annual health maintenance visit. He is a lifetime nonsmoker, consumes one alcoholic beverage a few days per week, and exercises at a local gym three times weekly. He reports a strong family history of cardiovascular disease. His father had a myocardial infarction (MI) at age 58, his mother had an MI at age 62, and his 58-year-old brother has been diagnosed with cardiovascular disease. The patient asks for your clinical opinion regarding the best dietary approaches for the primary prevention of cardiovascular disease. Which one of the following is your evidence-based recommendation? A) Skip breakfast regularly to reduce total caloric intake B) Start -carotene supplementation C) Start high-dose omega-3 fatty acid supplementation D) Start the Dietary Approaches to Stop Hypertension (DASH) diet E) Start the ketogenic diet

ANSWER: D The Dietary Approaches to Stop Hypertension (DASH) and Mediterranean diets have the best overall evidence for the primary prevention of cardiovascular disease (SOR A), and they should be regularly recommended to patients seeking to improve their risk. Other increasingly common dietary trends, such as the ketogenic diet, do not have a significant evidence base to support clear recommendations. The ketogenic diet has a strong evidence base in treatment-resistant pediatric epilepsy but not for the prevention of cardiovascular disease. Several studies have suggested that skipping breakfast may increase the risk of cardiovascular disease. The U.S. Preventive Services Task Force recommends against -carotene supplementation, due to increased risk of harm (D recommendation). There is now high-quality evidence (SOR A) that omega-3 fatty acid supplementation has no effect on cardiovascular disease.

An obese 60-year-old male presents with knee pain after a fall. He reports that 2 days ago he slipped on some ice and fell forcefully onto his left knee. He is able to walk but does have a noticeable limp. On examination he is able to flex his knee past 90° and has no point tenderness. Which one of the following would be most appropriate at this time? A) Anti-inflammatory medication B) A corticosteroid injection C) A knee brace D) A radiograph E) MRI

ANSWER: D The Ottawa knee rule is a validated tool that decreases unnecessary radiography in patients with a knee injury. According to the Ottawa knee rule, a radiograph should be obtained if any of the following are present: • age >55 years • isolated tenderness of the patella • tenderness of the head of the fibula • inability to flex the knee to 90° • or the inability to bear weight for four steps both immediately after the injury and at the time of the examination. A radiograph would be indicated for this patient based on his age. Anti-inflammatory medication may be an appropriate strategy for acute pain treatment, but this patient meets the criteria for a radiograph, which should be done first to rule out a fracture. A corticosteroid injection may be a consideration for treatment of chronic knee pain, generally in the setting of osteoarthritis, but would not be recommended in the setting of acute, undifferentiated pain. Knee braces can be useful with certain causes of knee pain, such as medial unloading knee braces for medial knee osteoarthritis, but the use of a knee brace in this situation without a diagnosis would not be warranted. MRI could be appropriate later in the workup, but the first step should be a radiograph.

A 68-year-old female comes to your office to discuss results from recent laboratory testing. During routine monitoring of electrolytes while taking lisinopril (Prinivil, Zestril) for hypertension, she was found to have a serum calcium level of 10.6 mg/dL (N 8.6-10.0). Follow-up laboratory studies revealed a parathyroid hormone level of 106 pg/mL (N 10-65) and normal levels of vitamin D, phosphorus, albumin, and creatinine. The patient feels well today and has no symptoms of concern. The patient's past medical history includes hypertension, hypothyroidism, and osteoporosis. Her current medications include lisinopril, levothyroxine (Synthroid), alendronate (Fosamax), and vitamin D. An examination today is normal, including a blood pressure of 122/74 mm Hg. Which one of the following characteristics would be an indication for surgical management of this patient's condition? A) Age >50 B) Hypertension C) Hypothyroidism D) Osteoporosis E) A serum calcium level <11 mg/dL

ANSWER: D The cause of this patient's hypercalcemia is primary hyperparathyroidism. Though the calcium levels are not severely elevated and she is asymptomatic, the overproduction of parathyroid hormone leads to calcium loss from bones and acceleration of osteoporosis. She would be a candidate for definitive management of her hyperparathyroidism via surgical removal of the offending parathyroid gland. Indications for surgery include a serum calcium level >1 mg/dL above the normal range, skeletal indications, renal indications, or age <50 years. Skeletal indications include either a previous vertebral fracture or a bone density more than 2.5 standard deviations below peak mean bone mass at the hip, lumbar spine, or distal radius. Renal indications can include an estimated glomerular filtration rate <60 mL/min/1.73 m2; a 24-hour urinary calcium level >400 mg/day; or nephrolithiasis or nephrocalcinosis seen on a radiograph, ultrasound examination, or CT.

A 26-year-old female with a history of bipolar disorder sees you for follow-up of chronic joint pain. The review of systems is positive for intermittent rashes on the dorsal hands and face and left-sided pleuritic chest pain. An examination reveals tender swelling of the hand and wrist joints bilaterally. Examination of the lungs reveals diminished breath sounds at the left base with point-of-care ultrasonography findings consistent with a small pleural effusion. Initial laboratory tests are significant for a platelet count of 96,000/mm3 (N 130,000-450,000), a positive antinuclear antibody test with a 1:80 titer (N <1:40), and negative Lyme disease and HIV tests. Which one of the following would help confirm your suspected diagnosis? A) Elevated anticyclic citrullinated peptide antibodies B) Elevated anti-smooth muscle antibodies C) Elevated anti-centromere antibodies D) Low complement levels E) Positive HLA-B27

ANSWER: D The clinical findings on examination, including symmetric polyarthritis, thrombocytopenia, positive antinuclear antibodies (ANAs), and pleural effusion, meet the American College of Rheumatology criteria for a diagnosis of systemic lupus erythematosus (SLE). The rash and the patient's neuropsychiatric history may also factor into the diagnosis, but they are not described specifically in this case. A positive ANA is sensitive but not specific for SLE. Although additional laboratory testing may not be needed to confirm SLE in this case due to classic clinical findings, low complement levels help confirm SLE and may be helpful because the ANA is only mildly elevated. Elevated anticyclic citrullinated peptide antibodies help confirm a diagnosis of rheumatoid arthritis. Anti-smooth muscle antibodies are used to confirm autoimmune hepatitis, which can also cause an elevated ANA. Anti-centromere antibodies, a subset of ANAs, are more closely associated with systemic sclerosis. A positive HLA-B27 test is associated with the seronegative spondyloarthropathies, such as psoriatic arthritis.

A 34-year-old female with type 2 diabetes that had previously been managed with metformin (Glucophage) and liraglutide (Victoza) was switched to insulin for glycemic management during her pregnancy. In the week before delivery she was achieving fasting and postprandial glucose targets with an insulin regimen of insulin detemir (Levemir), 16 U daily, and insulin lispro (Humalog), 4 U with meals. Her labor was induced at 37 weeks gestation due to elevated blood pressures and she had an uncomplicated vaginal delivery. She is planning to breastfeed. Which one of the following glucose management regimens would you recommend for her in the immediate postpartum period? A) Continue the current dosages of insulin B) Continue insulin detemir and stop insulin lispro C) Continue insulin lispro and stop insulin detemir D) Discontinue insulin and reinitiate metformin E) Discontinue insulin and reinitiate liraglutide

ANSWER: D The incidence of type 2 diabetes continues to rise in the United States, including in women of childbearing age. Type 2 diabetes complicates 1%-2% of pregnancies in the United States. Tight glycemic control (hemoglobin A1c <6.5%) is recommended during pregnancy, especially in the preconception period and the first trimester if possible. Risks of uncontrolled diabetes in pregnancy include preeclampsia, congenital defects, preterm delivery, macrosomia, and stillbirth. Insulin is the safest medication for glycemic control in pregnant patients, with requirements to achieve glycemic targets typically varying over the course of a pregnancy. After an initial increase in insulin requirements through week 9 of pregnancy, they often decline in weeks 9-16 before rising again in weeks 16-37. Beyond week 37 insulin requirements may again start to decline. Immediately after delivery of the placenta women become exquisitely sensitive to insulin, and insulin requirements may drop as low as 50% of pre-pregnancy needs. Patients with type 2 diabetes who were not taking insulin prior to pregnancy typically no longer require insulin. Metformin is safe in breastfeeding women and should be restarted in this patient who did well on it prior to pregnancy. No information is available regarding the safety of liraglutide while breastfeeding. Until more data becomes available it should be used with caution and is not the preferred agent in this patient.

A 24-year-old female presents with progressively worsening vulvar pain for 3 days. On examination a 3×3-cm tender, fluctuant mass is noted on the right labia minora. She had a similar episode of this problem last year. Which one of the following would be the most appropriate management? A) Expectant management B) Fine-needle aspiration C) Incision and drainage D) Marsupialization E) Excision under general anesthesia

ANSWER: D The most appropriate management of a recurrent Bartholin gland abscess would be marsupialization, which has a 0% recurrence rate at 6 months. Local anesthesia can be used in the office to effectively treat Bartholin gland abscesses and sedation is not required (SOR A). If the Bartholin gland abscess is >5 cm, referral to a gynecologist is recommended. Expectant management, fine-needle aspiration, or incision and drainage would likely lead to recurrence.

A family in your practice includes an 11-year-old female and a 17-year-old male who are both healthy. They are seen separately with and without their parents and have no evidence of genital warts or other lesions. The 11-year-old is not sexually active but the 17-year-old has given and received oral sex from a previous girlfriend. He says he is not sexually active at this time. Neither of these patients has received HPV vaccine in the past. Which one of the following should you recommend regarding HPV vaccination? A) No HPV vaccine for either sibling B) A two-dose HPV vaccine series for both siblings C) A three-dose HPV vaccine series for both siblings D) A two-dose HPV vaccine series for the 11-year-old and a three-dose HPV vaccine series for the 17-year-old E) A three-dose HPV vaccine series for the 11-year-old and a two-dose HPV vaccine series for the 17-year-old

ANSWER: D The recommended HPV vaccine series includes two or three doses depending on the age of initiation. Two doses of HPV vaccine are recommended for children and adolescents who start the series before 15 years of age. The second dose should be administered 6-12 months after the initial dose. If the patient is 15 years or older at vaccine initiation, then a three-dose series would be indicated at 0, 1-2, and 6 months. In this case, neither child has received HPV vaccine previously, so the 11-year-old needs a two-dose HPV vaccine series and the 17-year-old needs three doses. Although the optimal timing for HPV vaccination is before the initiation of sexual activity, it can still provide protection if administered after a patient has become sexually active. It is unlikely that this 17-year-old patient was exposed to all HPV serotypes and therefore could still receive significant benefit. There is no need to limit the administration of HPV vaccine based on testing for sexually transmitted infections.

You see a 60-year-old male for an initial office visit. He has a past history of a myocardial infarction. He has smoked two packs of cigarettes daily since the age of 18. He says that he enjoys smoking and has no plans to quit. Which one of the following would be the most appropriate response? A) I must advise you that smoking is very bad for your health B) Did you know that smoking increases your risk of another heart attack? C) What do you know about smoking's effect on your health in general? D) Would it be okay if we discuss smoking? E) Can I schedule a follow-up appointment soon to focus on quitting smoking?

ANSWER: D The stages of change model includes precontemplation, contemplation, preparation, action, and maintenance. Identifying which stage a patient is in can help guide physicians in counseling strategies. The goal is to move the patient toward taking action. This patient is in the precontemplation stage. He has no interest in changing and is opposed to quitting smoking. At this stage it is best to use the motivational interviewing technique of asking permission before presenting information and if given permission to share it in a neutral manner. Motivational interviewing has been shown to be useful in primary care for helping patients to reduce weight, blood pressure, and alcohol use (SOR A). Emphasizing the consequences of smoking or asking to schedule a follow-up visit to focus on smoking cessation may increase resistance and may also reduce patient openness to physician input at this stage.

A 45-year-old male comes to your office for a routine health maintenance examination. His medical history is significant for sarcoidosis, which was diagnosed 5 years ago during a workup for hilar adenopathy seen on a routine chest radiograph. He has been entirely asymptomatic for the past 2 years and has never required any treatment. Which one of the following annual screenings would be most appropriate? A) Bone density screening B) Echocardiography C) High-resolution CT of the chest D) An ophthalmologic examination E) A urinalysis

ANSWER: D There are numerous extrapulmonary manifestations of sarcoidosis that require periodic monitoring, even in asymptomatic patients. Because ocular involvement occurs in 20%-50% of patients and asymptomatic inflammation of the eye caused by sarcoidosis can cause permanent damage, an annual eye examination is very important. Anterior uveitis and keratoconjunctivitis are the most common symptomatic presentations of eye disease. Bone density screening is indicated for monitoring bone health during corticosteroid treatment. Cardiopulmonary manifestations of sarcoidosis are well known, but general testing, such as echocardiography or high-resolution CT of the chest, is reserved for the evaluation of symptoms, or possibly as a response to ongoing therapy. The genitourinary tract is generally not impacted by sarcoidosis and routine urine studies are not required.

A 50-year-old male with no significant past medical history presents with a 5-day history of fever to 101°F, chills, and mild diffuse joint and muscle pains. He also reports a mild headache but has not had any sore throat, rhinorrhea, cough, shortness of breath, nausea, vomiting, or change in bowel habits. He noticed a round red rash (shown below) a few days ago on his leg that has grown in size since then. It is minimally pruritic but not painful. He has no other rashes. He recently traveled to Vermont for his family's annual fall hiking trip but does not recall any insect bites. He does not take any medications and has no drug allergies. He has a blood pressure of 120/74 mm Hg, a pulse rate of 84 beats/min, and a temperature of 37.8°C (100.0°F). Cardiac, pulmonary, musculoskeletal, and abdominal examinations are normal. Which one of the following is the most likely diagnosis? A) Adenovirus B) Ehrlichiosis C) Influenza D) Lyme disease E) Rocky Mountain spotted fever

ANSWER: D This patient has a classic presentation for Lyme disease. Lyme disease is transmitted by a tick bite, but not all patients remember being bitten by a tick. The classic erythema migrans lesion appears a couple of weeks after the tick bite. The first-line treatment for Lyme disease is either amoxicillin or doxycycline. Macrolides can be used if patients have true allergies to -lactams and doxycycline, but they are less effective. This patient is not exhibiting the respiratory symptoms typically associated with adenovirus. Ehrlichiosis and Rocky Mountain spotted fever typically present with headaches and fevers but not with an erythema migrans rash. Although influenza should be considered in the differential diagnosis, it would not present with an erythema migrans rash.

A 72-year-old male comes to your office for an annual health maintenance visit. He mentions that some of his friends recently underwent health screenings and he asks if there are any cardiovascular screening tests recommended for him. His blood pressure is well controlled and he does not have any shortness of breath or chest pain. He exercises regularly. He started smoking cigarettes while he attended college but quit at age 25. He does not have a significant family history of cardiovascular disease. Which one of the following tests is recommended by the U.S. Preventive Services Task Force for patients such as this? A) An ankle-brachial index B) A high-sensitivity C-reactive protein level C) A coronary artery calcium score D) Abdominal aortic aneurysm screening with ultrasonography E) Echocardiography

ANSWER: D This patient is male, over the age of 65, and smoked in the past, so he meets the criteria for one-time screening for an abdominal aortic aneurysm (AAA) recommended by the U.S. Preventive Services Task Force (USPSTF) (B recommendation). This screening is associated with decreased AAA-related mortality. The USPSTF found insufficient evidence for screening asymptomatic adults for cardiovascular disease with an ankle-brachial index, high-sensitivity C-reactive protein level, or coronary artery calcium score. Screening asymptomatic individuals with echocardiography is not recommended at this time.

A 21-year-old female comes to your office because of concerns about excess hair growth. She has dark hairs on her upper lip and chin and around her nipples that have been consistently present since puberty, and she is dissatisfied with the cosmetic results of various hair removal methods. She has regular menses and does not wish to become pregnant at this time. An examination is consistent with some terminal hairs in the distribution she describes and is otherwise unremarkable. Which one of the following would be most appropriate at this point? A) An early morning total testosterone level B) A full hormonal workup including thyroid function testing, prolactin, 17-hydroxyprogesterone, and dehydroepiandrosterone sulfate (DHEAS) levels C) Transvaginal ultrasonography D) A 6-month trial of oral contraceptive pills

ANSWER: D This patient most likely has benign idiopathic hirsutism. It is estimated that approximately 50% of women with mild hirsutism have idiopathic hirsutism. In the absence of other worrisome findings on the history or examination, such as a rapid onset, virilization, or a high degree of hirsutism, the most appropriate next step is a trial of pharmacologic therapy, using oral contraceptive pills as the first-line agent if the patient does not desire pregnancy. A minimum 6-month trial is needed because of the length of the hair growth cycle. An early morning total testosterone level, a full hormonal workup, and transvaginal ultrasonography would be appropriate if there were other signs and symptoms of hyperandrogenism on the history or examination.

A 40-year-old female presents for follow-up of a recent diagnosis of severe persistent asthma. Skin testing for common inhalant allergens is negative, and her serum IgE levels are normal. She has a history of rhinorrhea and wheezing with exposure to aspirin. Which one of the following additional conditions is most likely to be present? A) Allergic rhinitis B) Aspergillosis C) Eczema D) Nasal polyps E) Obesity

ANSWER: D This patient presents with adult-onset, severe and persistent, non-atopic asthma and aspirin sensitivity. This type of asthma is defined as intrinsic asthma and affects approximately 10% of patients with asthma. Patients with intrinsic asthma commonly also have nasal polyps. Allergic rhinitis and eczema are both associated with atopy and patients with these conditions would also have positive skin tests for inhalant allergens and elevated IgE levels. While aspergillosis is associated with chronic sinusitis and nasal polyps, skin tests and IgE levels would also be positive. Obesity is an independent risk factor for asthma but is not specifically related to intrinsic asthma.

24-year old female presents to your office with a 3-month history of difficulty sleeping. She says that she struggles to fall asleep and wakes up multiple times at night at least three times a week. She tries to go to bed at 10:00 p.m. and wakes up at 6:30 a.m. to start her day. She lies awake for an hour in bed before falling asleep and spends up to 2 hours awake in the middle of the night trying to fall back asleep. Lately she has been feeling fatigued and having difficulty concentrating at work. You conduct a full history and physical examination and tell her to return in 2 weeks with a sleep diary. At this follow-up visit you see from her diary that she is sleeping an average of 5½ hours per night. Which one of the following would be the most appropriate recommendation? A) Set her alarm for 5:30 a.m. B) Add a mid-afternoon nap C) Move her bedtime to 9:00 p.m. D) Move her bedtime to 12:30 a.m. E) Stay up for an hour if she wakes up at 3:00 a.m.

ANSWER: D This patient presents with symptoms of chronic insomnia. Cognitive-behavioral therapy for insomnia (CBT-I) and brief behavioral therapy for insomnia (BBT-I) are effective nonpharmacologic treatments for chronic insomnia. Modified CBT-I and BBT-I can be administered by a primary care physician. The basic principles include stimulus control (sleep hygiene) and sleep restriction. Reducing time in bed increases sleep efficiency. In this case, 6 hours of time in bed would improve the patient's sleep efficiency and a bedtime of 12:30 a.m. would accomplish this goal. Generally, reduced time in bed is accomplished by postponing bedtime rather than getting up earlier. Naps generally do not improve sleep efficiency. While getting out of bed is recommended after being in bed for 30 minutes without falling asleep, or being awake for 30 minutes after being asleep, staying up for a prescribed period of time is not recommended.

A 34-year-old female has a "bump" on her middle finger (pink, raised, soft lesion at the border of the middle finger and the nail. Non erythematous, associated with indention of nail). She thinks it has been there approximately a month but says that it only recently began to bother her. She has not tried to treat it. This lesion is most likely a A) basal cell carcinoma B) dermatofibroma C) keratoacanthoma D) mucous cyst E) wart

ANSWER: D This patient's lesion is a digital mucous cyst, also known as a cutaneous myxoid cyst. Mucous cysts most commonly occur on the dorsal surface of the distal phalanx. Toe lesions are less common. The etiology is controversial. Treatment options include intralesional corticosteroid injections, repeated puncture and drainage, or surgical excision (SOR A). The other options listed are unlikely to be found on the finger with the exception of a wart, which would have a verrucous texture and appearance. While basal cell carcinomas and dermatofibromas are also smooth and nodular, neither are common on the fingers. Keratoacanthomas are smooth, dome-shaped, red papules that often expand rapidly over a few weeks' time on sun-damaged skin and may have a central keratin plug. They are more common in older patients.

A healthy 52-year-old male who runs for 40 minutes 4-5 times per week comes to your office because he has recently noted pain in the inner aspect of his right knee along the joint line. He also reports mild swelling and stiffness in his knee along with a "clicking" sensation when he flexes and extends it. In addition, he says that at unpredictable times he feels like his knee "might give way." During the examination he reports pain along the medial joint line with palpation, and the McMurray test is positive. A subsequent MRI demonstrates a degenerative tear in the lateral portion of the medial meniscus with no significant degenerative arthritis. You begin to discuss a management plan with him, considering various surgical and nonsurgical options. Given this patient's age and clinical findings, which one of the following has been shown to produce better long-term outcomes than conservative management? A) Intra-articular corticosteroids B) Hylan GF 20 (Synvisc) C) Arthroscopic meniscectomy D) Arthroscopic meniscus repair E) No additional treatment strategies

ANSWER: E A systematic review found that arthroscopic procedures for degenerative meniscal tears in middle-aged adults with little or no arthritis do not significantly improve long-term pain or function compared to conservative management consisting of physical therapy and a standardized exercise program. No studies have compared conservative management to intra-articular injections of corticosteroids or hylan GF 20 for managing degenerative meniscal tears, either alone or when combined with conservative management.

Which one of the following antihypertensive medications is associated with reduced progression of diabetic kidney disease? A) Amlodipine (Norvasc) B) Chlorthalidone C) Labetalol (Trandate) D) Spironolactone (Aldactone) E) Valsartan (Diovan)

ANSWER: E ACE inhibitors and angiotensin receptor blockers are associated with decreased progression of diabetic kidney disease. Unless otherwise contraindicated, patients with diabetes should be taking one of these agents. Amlodipine, chlorthalidone, labetalol, and spironolactone are not directly associated with decreased progression of diabetic kidney disease, but they all may contribute to a decrease in progression due to long-term control of blood pressure.

A 36-year-old female with no significant past medical history presents to your office following a syncopal episode. A thorough history and physical examination are normal. Based on American College of Cardiology/American Heart Association/Heart Rhythm Society guidelines, which one of the following would you recommend at this time? A) Reassurance, and follow-up only if she has another syncopal episode B) A troponin level C) Electrolyte levels D) A chest radiograph E) An EKG

ANSWER: E According to guidelines from the American College of Cardiology, American Heart Association, and Heart Rhythm Society, the evaluation of patients with syncope should include a thorough history and physical examination, as well as an EKG. Further studies would be indicated if the cause of syncope is unclear.

A 26-year-old female presents to your office after surviving a tornado that destroyed her home 3 months ago. She reports continued intense fear and anxiety when there is a storm. You diagnose her with posttraumatic stress disorder. Which one of the following has the best evidence of effectiveness for her condition? A) Amitriptyline B) Atypical antipsychotics C) Benzodiazepines D) Topiramate (Topamax) E) Cognitive-behavioral therapy

ANSWER: E According to the Agency for Healthcare Research and Quality's 2018 guidelines, cognitive-behavioral therapy (CBT), either exposure or mixed treatment, has the best evidence of effectiveness in the treatment of posttraumatic stress disorder (PTSD) (SOR A). CBT focused on either artificial exposures or real-life exposures reduces PTSD and depression symptoms. Artificial exposures can be imagined, written, or virtual reality. Mixed modalities such as cognitive restructuring, exposure-guided imagery, and mindfulness training also reduce PTSD and depression symptoms. Other types of cognitive therapy also have evidence of effectiveness, but the study results are less precise. The SSRIs fluoxetine and paroxetine and the SNRI venlafaxine have moderate evidence to support their use in the treatment of PTSD (SOR B). The guidelines recommend against the use of amitriptyline, atypical antipsychotics, and topiramate. Benzodiazepines are not recommended in the treatment of PTSD.

A 48-year-old female presents with concerns about hair loss. She has noticed gradual thinning of the hair on the top of her head for the last year. Her scalp is now visible through the hair. She is not taking any oral medications. A levonorgestrel IUD (Mirena) was placed 4 years ago. She has not had any recent illnesses or stressors and a review of systems is negative. Her mother had similar hair loss starting in her fifties. On examination you note thin hair on the top of the scalp, an intact frontal hair line, and growth of thin, wispy hairs on her crown. Which one of the following would be most appropriate for this patient? A) Removal of the levonorgestrel IUD B) Spironolactone (Aldactone), 50 mg daily for 6 months C) Finasteride (Proscar), 5 mg daily indefinitely D) Minoxidil (Rogaine) 2% solution for 6 months E) Minoxidil 5% foam indefinitely

ANSWER: E Hair thinning on the crown of the head with the presence of small, wispy hairs among the regular hair is characteristic of female pattern hair loss (FPHL). A family history of similar issues is often present but not necessary for the diagnosis. Topical minoxidil is the mainstay of treatment for FPHL (SOR A). It is available in a 2% solution or 5% foam for women (the 5% solution is indicated only for men). Treatment for FPHL as well as the male equivalent, androgenic alopecia, must be continued long term. With treatment there is often an initial period of increased hair loss. Regrowth is noticeable around 6 months. Discontinuation of treatment results in loss of regrown hair. There is no clear association between hormone status and FPHL. Removal of this patient's levonorgestrel IUD is unlikely to affect hair loss. Spironolactone has also been used for FPHL but evidence is lacking regarding its effectiveness. Finasteride is approved by the FDA only for males with hair loss. There is a high risk of teratogenicity with its use. It has been used in women but evidence of efficacy is minimal.

A 37-year-old male sees you to discuss some mental health concerns. He states that he has recently been unable to meet deadlines at work and has been reprimanded by his boss. He feels he cannot complete the tasks during his work hours and is worried he is going to lose his job. Your evaluation indicates that he might have adult attention-deficit/hyperactivity disorder (ADHD). Which one of the following is true regarding adult ADHD? A) Symptoms of ADHD must be present before age 6 B) Inattention does not persist into adulthood C) Hyperactivity symptoms worsen into adulthood D) Stimulants should not be prescribed in adults over the age of 30 E) Adults with ADHD should be screened for coexisting psychiatric disorders

ANSWER: E Attention deficit/hyperactivity disorder (ADHD) is a common neurodevelopmental disorder in children and adolescents. Approximately 30% of children carry the diagnosis into adulthood. Individuals with ADHD should be screened for coexisting psychiatric disorders because they are at a higher risk for problems such as anxiety and depression and are more likely than the general population to have substance use disorders. The DSM-5 criteria for the diagnosis of adult ADHD require symptoms to have been present before age 12. Inattention often persists into adulthood, while hyperactivity and impulsivity usually improve with time. Compared to children with ADHD, hyperactivity in adults often presents as talkativeness, irritability, and restlessness. Along with behavioral strategies, stimulants have been found to be effective for treatment of adult ADHD. Although there are no age restrictions for the use of stimulants, adults should have their blood pressure and heart rate monitored during use to monitor for potential cardiac complications.

A 50-year-old female sees you for the first time. She is active and has not seen a physician for 2 years. The last vaccine she received was Tdap following the birth of her granddaughter 2 years ago. She has a past history of essential hypertension and hyperlipidemia but her medical history is otherwise unremarkable. Based on CDC guidelines, which one of the following vaccines should you recommend for this patient today? A) HPV B) Pneumococcal conjugate (PCV13, Prevnar 13) C) Pneumococcal polysaccharide (PPSV23, Pneumovax 23) D) Live zoster (Zostavax) E) Recombinant zoster (Shingrix)

ANSWER: E Based on current CDC guidelines, this patient should receive the recombinant herpes zoster vaccine. It is the preferred vaccine for the prevention of herpes zoster due to much higher efficacy than the live zoster vaccine. The live vaccine is considered a second-line option and should not be administered before the age of 60. HPV vaccine is not indicated in this age group (>45 years). Pneumococcal polysaccharide vaccine (PPSV23) is recommended for adults >65 years of age. Patients who are 65 and older can discuss with their physician whether they should receive pneumococcal conjugate vaccine (PCV13). Immunocompromised patients may receive both pneumococcal vaccines prior to age 50.

An EKG performed on a patient with a history of heart failure and an ejection fraction of 30% reveals sinus rhythm with a heart rate of 55 beats/min and new left bundle branch block with a QRS interval of 160 msec. The patient is taking a beta-blocker but has a history of acetylcholinesterase inhibitor-induced angioedema. Which one of the following would be most appropriate at this point? A) Amiodarone B) Ivabradine (Corlanor) C) Sacubitril/valsartan (Entresto) D) Sinoatrial node ablation E) Cardiac resynchronization therapy

ANSWER: E Cardiac resynchronization therapy is strongly recommended for patients with symptomatic heart failure, an ejection fraction <35%, and a left bundle branch block with a QRS interval >150 msec. Amiodarone is an antiarrhythmic and would not be indicated for this patient. Ivabradine, a sinoatrial node modulator, is used in patients with symptomatic heart failure as an add-on therapy to decrease the heart rate. It is indicated in patients with a heart rate >70 beats/min despite -blockade. Sacubitril/valsartan is contraindicated in patients with a history of angioedema. Sinoatrial node ablation is indicated for some patients with sinus node dysfunction.

A 40-year-old female presents with a 3-month history of symptoms suggestive of carpal tunnel syndrome. Which one of the following typically results from impairment of the median nerve in this condition? A) Weakness of thumb abduction B) Weakness of flexion in the thumb, index, and middle fingers C) Numbness on the dorsum of the hand D) Numbness in the ulnar side of the ring finger and in the little finger E) Paresthesias or pain in the thumb, index finger, middle finger, and radial side of the ring finger

ANSWER: E Carpal tunnel syndrome (CTS) results in pressure on the median nerve as it passes through the volar wrist to the palm. Sensory symptoms include paresthesias (tingling, burning), numbness, and/or pain, and these symptoms occur before motor weakness. The sensory distribution of the median nerve is to the thumb, index finger, and long finger, and the radial half of the ring (fourth) finger. The median nerve's motor function is thumb adduction, and CTS can result in thenar eminence atrophy and weakness of thumb adduction if untreated. Morning stiffness in the fingers is associated with inflammatory arthritis rather than CTS. Numbness on the dorsum of the hand is often associated with de Quervain's tenosynovitis. Cubital tunnel syndrome usually presents with numbness and tingling along the ulnar border of the forearm and hand, as well as the ring and little fingers.

You see an 89-year-old female with advanced dementia who has stopped eating. The patient's daughter asks you about the role of tube feeding in this situation. You discuss the risks and benefits. In patients with advanced dementia who have a feeding tube placed, there is evidence of increased A) nutritional status B) healing of pressure ulcers C) quality of life D) survival E) emergency department visits

ANSWER: E Eating problems in patients with advanced dementia are common and include oral dysphagia, pharyngeal dysphagia including aspiration, the inability to feed oneself, and refusal to eat. Patients with these symptoms should be examined for reversible causes such as dental problems. In the absence of a reversible cause, conservative measures such as altering food texture or offering small portions and high-calorie supplements may promote weight gain, although none of these interventions improve function or survival. Tube feeding patients who have advanced dementia has not been compared to hand feeding in randomized, controlled trials. Observational studies have shown no difference in survival, quality of life, nutritional status, functional status, the prevention of aspiration, or the prevention and healing of pressure ulcers between the two groups. Risks associated with feeding tubes include the risks associated with placement of the tube, including the chemical and physical restraint of patients who attempt to remove the tube. Once the tube is in place, tube blockages and dislodgments are common reasons for transfer to an emergency department (ED) for care, and in one study accounted for 47% of all ED visits by nursing home residents with advanced dementia.

A 24-year-old patient wants to start the process of transitioning from female to male. He has been working with a psychiatrist who has confirmed the diagnosis of gender dysphoria. Which one of the following would be the best initial treatment for this patient? A) Clomiphene B) Letrozole (Femara) C) Leuprolide (Eligard) D) Spironolactone (Aldactone) E) Testosterone

ANSWER: E For patients with gender dysphoria or gender incongruence who desire hormone treatment, the treatment goal is to suppress endogenous sex hormone production and maintain sex hormone levels in the normal range for their affirmed gender. For a female-to-male transgender patient this is most easily accomplished with testosterone. When testosterone levels are maintained in the normal genetic male range, gonadotropins and ovarian hormone production is suppressed, which accomplishes both goals for hormonal treatment without the need for additional gonadotropin suppression from medications such as leuprolide. Clomiphene can increase serum testosterone levels, but only in the presence of a functioning testicle. Letrozole is an estrogen receptor antagonist, but it would not increase serum testosterone levels. Spironolactone has androgen receptor blocking effects and would not accomplish either of the hormone treatment goals.

A 70-year-old female develops thrombocytopenia during a prolonged hospitalization for endocarditis. Her current medications include scheduled unfractionated heparin injections for venous thromboembolism prophylaxis. You suspect heparin-induced thrombocytopenia (HIT). Assuming that her thrombocytopenia is caused by HIT, which one of the following is the most likely complication? A) Anaphylaxis B) Disseminated intravascular coagulation C) Hemorrhage D) Sepsis E) Thrombosis

ANSWER: E Heparin-induced thrombocytopenia (HIT) is an immune-mediated process that occurs in approximately 1 in 5000 hospitalized patients. Patients are at highest risk 7-10 days after exposure to unfractionated heparin, and the risk is particularly high after cardiac surgery, which is associated with an estimated rate of 1%-3%. In contrast to other causes of thrombocytopenia, HIT places patients at a paradoxically increased risk of thrombotic complications, with clotting events occurring in roughly 50% of confirmed cases of HIT. Lower-extremity deep vein thrombosis and pulmonary embolism are the most common thrombotic complications, followed by arterial thromboses, stroke, and myocardial infarction, in descending order of frequency. Thromboses often occur concurrently with the development of thrombocytopenia or shortly thereafter. The risk of HIT can be determined with the 4T scoring system, which evaluates the acuity of thrombocytopenia, timing of onset, presence of thrombosis, and alternative causes of thrombocytopenia. Patients with an intermediate or high pretest probability should be managed with prompt discontinuation of heparin and initiation of full-dose anticoagulation with a non-heparin anticoagulant, such as argatroban, danaparoid, fondaparinux, or bivalirudin, pending results of further HIT diagnostic evaluation. Anaphylaxis, disseminated intravascular coagulation, hemorrhage, and sepsis are all less common complications of HIT compared to thrombotic events.

A 42-year-old male presents to your office with colicky flank pain of several hours' duration. A urinalysis shows hematuria and CT confirms a 6-mm kidney stone in the right mid-ureter. In addition to pain control, he asks if there is anything that can be done to hasten stone passage. Based on current evidence, you recommend A) furosemide (Lasix) B) hydrochlorothiazide C) oxybutynin D) sildenafil (Viagra) E) tamsulosin (Flomax)

ANSWER: E High-quality placebo-controlled studies have demonstrated a decreased time to clearance of kidney stones with the use of -blockers, including tamsulosin. For stones 6-10 mm in size, the mean time to clearance was reduced by almost 6 days. Furosemide is a loop diuretic and can increase calcium excretion, which is associated with an increased risk for kidney stones. Hydrochlorothiazide is a calcium-sparing diuretic, leading to increases in serum calcium and decreased excretion of calcium into the urine. Hydrochlorothiazide would decrease the risk of recurrence of calcium kidney stones but would not hasten stone passage. Oxybutynin and sildenafil have not been shown to hasten stone passage.

A 24-year-old male presents for evaluation of clavicular pain after a fall while mountain biking. On examination he has a painful palpable lump and bruising over the middle third of his left clavicle. He has normal peripheral pulses, and strength and sensation of his left arm are intact. Radiographs reveal a midshaft clavicular fracture that is displaced approximately 1.5 times the width of the clavicle and is shortened by 2 cm. The proximal portion is upwardly displaced. Which one of the following would be the most appropriate management of his injury? A) A figure-of-eight splint B) A sling C) Closed reduction and a sling D) Casting E) Evaluation for surgical fixation

ANSWER: E Historically, clavicular fractures tended to be managed conservatively because it was thought that most of these fractures heal well. However, studies have shown that completely displaced fractures (displacement greater than one bone width) have a 30% rate of unsatisfactory outcomes when managed conservatively. Referral for evaluation for surgical fixation should be considered in all patients with complete displacement, especially when there is comminution or significant shortening (>18 mm in men and >14 mm in women). Splinting, a sling, closed reduction, and casting would not be appropriate in this case.

A 34-year-old female comes to your office for follow-up after an emergency department visit because of anxiety. She notes persistent anxiety, poor focus, and palpitations. She also reports that she is not hungry and has lost several pounds. She reports "odd things happening" such as sudden weakness in her legs, falling, and getting lost. When taking her history you note that the patient is hyperverbal and displays tangential speech. She has a temperature of 37.4°C (99.3°F), a heart rate of 134 beats/min, a respiratory rate of 20/min, and a blood pressure of 117/69 mm Hg. A physical examination reveals an anxious-appearing female who is tremulous at rest. A HEENT examination shows exophthalmos but no thyromegaly. A cardiac examination is unremarkable aside from tachycardia. A pulmonary examination reveals faint bibasilar crackles. An EKG shows sinus tachycardia. Laboratory results are as follows: CBC within normal limits Basic metabolic panel within normal limits TSH <0.08 U/mL (N 0.35-3.00) Free T4 4.51 ng/dL (N 0.89-1.80) Free T3 >19.0 pg/dL (N 2.3-4.2) Which one of the following would be most appropriate at this point? A) Start methimazole (Tapazole) B) Check for thyroid receptor antibody C) Obtain a radioactive iodine uptake scan D) Refer her to endocrinology E) Admit her to the hospital

ANSWER: E Hyperthyroidism is a common condition with a generally favorable prognosis. However, it is important to remember that life-threatening complications such as thyrotoxicosis, also known as thyroid storm, can occur. Symptoms of thyroid storm include fever, central nervous system dysfunction, gastrointestinal or liver dysfunction, and cardiovascular complications such as tachycardia and heart failure. The diagnosis is made using the Burch-Wartofsky Point Scale, which produces a total score based on the presence or absence of various diagnostic criteria. In this case, the patient has a score of 45, which is highly suggestive of thyroid storm. This acute, life-threatening condition typically requires care in an intensive-care unit. It would therefore be inappropriate to start treatment with an agent such as methimazole prior to hospitalization. While a thyroid receptor antibody test may be useful in identifying the cause of the condition it should not delay hospitalization. A radioactive iodine uptake test is also useful for identifying the underlying cause of hyperthyroidism but should be avoided until the thyroid storm has resolved. This patient requires hospitalization, so a referral to endocrinology would not be most appropriate at this time.

Which one of the following should NOT be consumed during pregnancy due to a potentially high mercury content? A) Catfish B) Crawfish, shrimp, and lobster C) Flounder and haddock D) Salmon and trout E) Shark and swordfish

ANSWER: E Larger ocean fish that consume other fish may accumulate mercury levels that can cause neurologic problems when consumed, so these fish should be avoided by children and pregnant or nursing women. Shark and swordfish are among the fish with the highest mercury content. Catfish, crawfish, shrimp, lobster, flounder, haddock, salmon, and trout have the least amount of mercury.

Which one of the following symptoms is most consistent with plantar fasciitis? A) Burning over the inferomedial aspect of the calcaneus B) Pain with resisted flexion of the great toe C) Paresthesia in the plantar aspect of the foot D) Swelling and pain in the retrocalcaneal region E) Stabbing pain over the anteromedial plantar aspect of the heel

ANSWER: E Plantar fasciitis is the most common cause of heel pain in adults. It is characterized by stabbing pain over the anteromedial plantar aspect of the heel. It is usually worse with ambulation after a period of inactivity, such as the first steps of the day. Paresthesias are uncommon. Burning over the inferomedial aspect of the calcaneus is more characteristic of medial calcaneal nerve entrapment. Pain with resisted flexion of the great toe is more characteristic of flexor hallucis longus tenosynovitis. Paresthesia in the plantar aspect of the foot is more characteristic of tarsal tunnel syndrome. Swelling and pain in the retrocalcaneal region is more characteristic of retrocalcaneal bursitis.

A 58-year-old athletic trainer presents with acute low back pain. He thinks the pain began after he started a new weightlifting regimen 3 weeks ago. The pain does not radiate, and prolonged standing exacerbates the pain. He does not have any bowel or bladder incontinence, fever, or saddle anesthesia. He asks which type of therapy is most likely to shorten the course of the pain. Which one of the following would be the best recommendation? A) Ice B) Targeted exercises C) Oral analgesics D) Spinal manipulation E) Evidence does not support the superiority of any modality

ANSWER: E Recommendations for the treatment of back pain often include ice, targeted exercises, oral analgesics, and spinal manipulation. Although there is some evidence that certain modalities are better than placebo, there is very little evidence to show that one modality is superior to another in relieving back pain or shortening the course of the pain. There is some evidence that spinal manipulation is superior to sham manipulation.

A 23-year-old male presents for evaluation of wrist pain after he fell while skateboarding. On examination he has left wrist pain with snuffbox tenderness. A radiograph of his wrist shows a fracture through the midportion of the scaphoid with 2 mm of displacement. Which one of the following would be the most appropriate treatment of this injury? A) A thumb spica cast B) A short arm cast C) A long arm cast D) Closed reduction followed by a thumb spica cast E) Surgical fixation

ANSWER: E Scaphoid fractures have a high risk for nonunion because the blood supply arises distally from branches of the radial artery. The proximal pole of the scaphoid is entirely dependent on this distal blood supply. To improve healing and decrease the risk of nonunion and avascular necrosis, displaced fractures should be treated with surgical fixation. Nondisplaced fractures of the distal third of the scaphoid may be treated with a short arm thumb spica cast for 4-6 weeks. Middle and proximal fractures should be treated with a long arm thumb spica cast for 6 weeks, followed by a short arm thumb spica cast.

A 47-year-old male with obesity, hypertension, and type 2 diabetes sees you because of growth of his breasts. An examination confirms gynecomastia. Stopping which one of the following medications in this patient's current regimen would most likely result in regression of his gynecomastia? A) Amlodipine (Norvasc) B) Hydrochlorothiazide C) Liraglutide (Victoza) D) Pioglitazone (Actos) E) Spironolactone (Aldactone)

ANSWER: E Spironolactone has antiandrogenic properties and may cause gynecomastia in approximately 9% of male users. Stopping the medication typically leads to regression of the gynecomastia within 3 months. Amlodipine, hydrochlorothiazide, liraglutide, and pioglitazone do not have antiandrogenic properties and are not associated with gynecomastia.

A nonverbal 22-year-old male with intellectual disability is brought to your office by the staff of the group home where he lives. They report that the patient has been functioning at his baseline until this morning when he was found to have loud breathing. No other history is available at the time of this visit. On examination he has a temperature of 37.3°C (99.1°F), a blood pressure of 124/82 mm Hg, a pulse rate of 100 beats/min, and a respiratory rate of 16/min. The patient appears to be in mild distress and a high-pitched whistling, crowing sound on inspiration is heard as you walk in the room. Which one of the following would be the most appropriate next step for this patient? A) Oral antibiotics B) Oral corticosteroids C) Nebulized albuterol D) Nebulized epinephrine E) Urgent evaluation in the emergency department

ANSWER: E Stridor is a high-pitched whistling, crowing sound on inspiration. It can be caused by obstruction of the larynx or trachea by a foreign body, vocal cord edema, a neoplasm, or a pharyngeal abscess. Acute stridor requires urgent evaluation for obstruction. This patient may have a foreign body or other obstruction in his airway and requires urgent assessment. Oral antibiotics, oral corticosteroids, nebulized albuterol, or nebulized epinephrine would not be appropriate at this time.

A 60-year-old male presents with dyspnea on exertion, occasional wheezing, and a chronic cough that is productive. He has never been hospitalized. He has smoked one pack of cigarettes per day since the age of 20. An examination reveals diminished breath sounds but no crackles, jugular venous distention, gallop, or edema. Spirometry shows a postbronchodilator FEV1 that is 45% of the predicted value, and the severity of his disease is rated as Global Initiative for Chronic Obstructive Lung Disease (GOLD) group C. In addition to albuterol as needed for symptomatic relief and smoking cessation, the initial treatment should include A) beclomethasone B) budesonide/formoterol (Symbicort) C) roflumilast (Daliresp) D) theophylline E) tiotropium (Spiriva)

ANSWER: E The goals for treatment of this patient's COPD should include prevention of or a reduction in hospitalizations, a decrease in dyspnea, slowing progression of the disease, and a decrease in mortality. Disease severity is categorized by spirometry results, the severity of symptoms such as cough and dyspnea, and the number of exacerbations, including those requiring hospitalization. Classifying patients into Global Initiative for Chronic Obstructive Lung Disease (GOLD) groups A through D helps guide treatment initiation and modification over time. The initial treatment for patients in GOLD group A is a short- or long-acting bronchodilator. Patients in GOLD group B should begin treatment with a single long-acting muscarinic antagonist (LAMA) or a long-acting -agonist (LABA). A LAMA is the initial recommendation for patients in GOLD group C, although a combination inhaled corticosteroid plus a LABA can be considered for treating persistent exacerbations. Individuals classified in GOLD group D can begin treatment with a LAMA or a combination of an inhaled corticosteroid plus a LABA.

A 65-year-old male presents to your office with bilateral weakness that has been present for several weeks. He notes increasing difficulty in climbing stairs but is able to walk on his toes and heels normally. He can open a jar and turn a doorknob without difficulty but has trouble reaching up to an overhead shelf. The history is confirmed by a physical examination. Which one of the following is most often associated with this type of muscle weakness? A) Fluoroquinolone use B) Frailty syndrome C) Guillain-Barré syndrome D) Multiple sclerosis E) Polymyositis

ANSWER: E This patient exhibits classic symptoms and signs of proximal muscle (shoulder and hip girdle musculature) weakness but not distal muscle weakness. Of the conditions listed, various types of myositis, including polymyositis, are associated with proximal but not distal or bulbar (lower cranial nerves) weakness. Patients with proximal muscle weakness have difficulty walking up or down steps, or they exhibit weakness with overhead lifting or other activities requiring the upper arm and shoulder musculature. Fluoroquinolone use is associated with tendinopathy that may also cause distal rather than proximal muscle weakness. Frailty syndrome is associated with loss of muscle mass and patients exhibit a gradually slowing gait, usually combined proximal and distal muscle weakness, falls, and diminished grip strength. Guillain-Barré syndrome develops fairly rapidly, in a matter of days, and weakness begins distally, then progresses proximally (ascending paralysis), and is also associated with respiratory muscle and bulbar involvement. The neuromuscular manifestations of multiple sclerosis are widely varied, have a gradual onset, are most often unilateral, and usually include bulbar and/or sensory involvement.

A 10-year-old female is brought to your office for a sports preparticipation examination. You note thoracic rib asymmetry during the Adams forward bend test. Radiographs confirm rightward thoracolumbar scoliosis with a Cobb angle of 32°. Which one of the following would be most appropriate at this time? A) Genetic testing (ScoliScore) B) MRI of the thoracic and lumbar spine without contrast C) Scoliosis radiography in 1 year D) Referral to physical therapy E) Referral to a pediatric orthopedist

ANSWER: E This patient is a skeletally immature female with a Cobb angle that puts her at increased risk for progression (>29°). Referral to a spine specialist for consideration of bracing and appropriate follow-up is recommended. Genetic testing is available to help determine the risk of progression, but it is not a widely validated tool at this time. MRI does not provide any additional information to help with decision-making. Because of the patient's increased risk of progression, simple follow-up in 1 year is not recommended. Physical therapy is not indicated for the primary treatment of scoliosis.

A 12-year-old male is brought to your office by his guardian. In the last 12 months he was suspended from school multiple times for bullying and fighting. He kicks the family dog and goes out at night without permission. He ran away from home 2 weeks ago and he lies constantly. He skips school and does not care about his grades. Attention-deficit/hyperactivity disorder was previously ruled out. Two days ago he was grounded and retaliated by destroying his guardian's computer. During the office visit he says school is "boring and stupid." Which one of the following would be the most appropriate next step in managing this patient? A) Carbamazepine (Tegretol) B) Haloperidol C) Lithium D) Quetiapine (Seroquel) E) Referral for psychosocial intervention

ANSWER: E This patient meets the criteria for conduct disorder, which is a psychiatric syndrome occurring in childhood and adolescence. Aggression toward people or animals, deceitfulness, theft, destruction of property, and serious violations of rules are characteristic symptoms. Risk factors include poverty in childhood, male sex, exposure to sexual or physical abuse, and domestic violence. The DSM-5 specifies that at least 3 of 15 criteria should have been present in the past 12 months for a formal diagnosis. The treatment is multifaceted. Psychosocial interventions are recommended as the first-line treatment (C evidence rating). The FDA has not approved medications for the treatment of conduct disorder unless they are indicated for concurrent attention-deficit/hyperactivity disorder (ADHD). The FDA recommends considering risperidone as a short-term treatment for explosive anger or severe aggression after comorbid ADHD is treated. Carbamazepine, haloperidol, lithium, and quetiapine would not be appropriate treatments for this patient's conduct disorder.

A 53-year-old female who typically jogs 8-10 miles per week presents to your office with pain on the anterior part of her left kneecap that increases with running and when she goes up or down stairs. She has been taking ibuprofen, 600 mg three times daily, with partial relief. Plain film radiographs are negative. Which one of the following is the most likely diagnosis? A) Iliotibial band syndrome B) Osgood-Schlatter disease C) Osteochondritis dissecans D) Patellofemoral osteoarthritis E) Patellofemoral pain syndrome

ANSWER: E This patient presents with the classic symptoms of patellofemoral pain syndrome. The pain is in the anterior knee and increases during weight-bearing activities when the knee is flexed, as well as with prolonged sitting and descending stairs. Iliotibial band syndrome usually involves lateral pain and tenderness over the lateral femoral condyle. Osgood-Schlatter disease occurs in adolescents and is characterized by tenderness and swelling over the patellar tendon insertion at the tibial tubercle. Both osteochondritis dissecans and patellofemoral osteoarthritis would show abnormal findings on plain film radiographs.

A 56-year-old male comes to your office because of right shoulder pain for the past week. It started when he lifted a heavy piece of furniture while helping a friend move. He felt immediate sharp pain in his shoulder, which has since radiated down the biceps toward the right radial forearm. On examination there is no deformity of the shoulder or arm. He has increased pain with palpation in the anterior shoulder near the bicipital groove of the humerus. You suspect biceps tendinitis. Anterior shoulder pain with which one of the following examination maneuvers of the right arm would be most consistent with this diagnosis? A) Active or passive cross adduction of the arm at the shoulder B) Shooting pain to the thumb with axial compression of the head with the neck flexed toward the right shoulder C) Resisted extension of the elbow with the shoulder in a neutral position D) Resisted internal rotation of the shoulder with the elbow flexed to 90° E) Resisted supination of the hand with the elbow flexed to 90°

ANSWER: E This patient's symptoms are consistent with bicipital tendinitis, which causes pain with abduction and external rotation of the arm, and tenderness of the bicipital groove with palpation. Resisted supination of the hand with the elbow flexed to 90° is the Yergason test, and anterior shoulder pain with this maneuver is consistent with bicipital tendinitis. Anterior shoulder pain with cross adduction of the arm is more consistent with acromioclavicular arthritis. Axial compression of the slightly flexed neck is the Spurling test for cervical radiculopathy. Extension of the elbow would activate the triceps, and internal rotation of the shoulder with the elbow flexed would result in less activation of the biceps than resisted supination.

An 18-year-old male is brought to your clinic by his parents. He reports daily use of alcohol and opioids that he has been getting illegally from his friends. This habit started 3 years ago when he started high school. In addition to counseling, which one of the following pharmacologic agents is indicated to treat both of his substance use disorders? A) Acamprosate B) Bupropion (Wellbutrin) C) Disulfiram (Antabuse) D) Methadone E) Naltrexone

ANSWER: E Treatment of substance abuse disorders is multifactorial and should address the needs of the whole person. Evidence-based treatment modalities range from school- and parent-based interventions to medication-assisted treatment. Of the options listed, only naltrexone can be used for both alcohol and opioid use disorders. Acamprosate and disulfiram are used for alcohol use disorder, bupropion is used for nicotine use disorder, and methadone is used for opioid use disorder (SOR C).

In a patient on chronic warfarin (Coumadin) therapy who has a stable INR in the therapeutic range, which one of the following antibiotics would be most likely to elevate the INR? A) Cephalexin (Keflex) B) Clindamycin (Cleocin) C) Penicillin G D) Rifampin (Rifadin) E) Trimethoprim/sulfamethoxazole (Bactrim)

ANSWER: E Trimethoprim/sulfamethoxazole is one of the antimicrobials most likely to increase the INR of a patient taking warfarin. If trimethoprim/sulfamethoxazole is used in a patient on warfarin, reducing the warfarin dosage by 25%-40% is recommended, with close monitoring of the INR. The patient's INR should be checked within 3-5 days of starting or stopping any antimicrobial. First generation cephalosporins such as cephalexin, fourth generation cephalosporins, clindamycin, and penicillin G have a lower likelihood of affecting the INR. Rifampin decreases the INR and the warfarin dosage should be increased if rifampin is started. Other antimicrobials that significantly affect the INR include metronidazole and fluconazole. Azithromycin, ciprofloxacin, clarithromycin, and levofloxacin may impact the INR with a variable patient-specific effect.

Which one of the following types of hallucinations is associated with Parkinson's disease? A) Auditory only B) Auditory and visual C) Hypnagogic D) Hypnopompic E) Visual only

ANSWER: E Visual hallucinations are associated with Parkinson's disease, and are seen with Parkinson's dementia, which is a type of Lewy body dementia. The combination of auditory and visual hallucinations is associated with schizophrenia. Hypnagogic and hypnopompic hallucinations are associated with sleep disorders.


संबंधित स्टडी सेट्स

Business Communications Final Exam

View Set

Domain 1 (CISA Review Questions, Answers & Explanations Manual, 12th Edition | Print | English)

View Set

PrepU Management of Patient with Upper Respiratory Tract Disorders

View Set

15b. Evaluating Psychotherapies

View Set

AS Psychology: 1.8 - Social Influence: Resistance to Social Influence

View Set

SAYING NUMBERS: ORDINAL NUMBERS 21-30

View Set

Chapter 6: Price Ceiling and Price Floor

View Set

Computer Forensics - Mod 14 Quiz

View Set